Neuro Tests- Silvestri 336 Questions

Pataasin ang iyong marka sa homework at exams ngayon gamit ang Quizwiz!

The nurse is preparing to care for a client who had a supratentorial craniotomy. The nurse should plan to place the client in which position? 1. Prone 2. Supine 3. Side-lying 4. Semi-Fowler's

Rationale: After supratentorial surgery (surgery above the tentorium of the brain), the head of the client's bed usually is elevated 30 degrees to promote venous outflow through the jugular veins. Options 1, 2, and 3 denote incorrect positions after this surgery and these positions could result in edema at the surgical site and increased intracranial pressure.

A client admitted to the nursing unit from the hospital emergency department has a C4 spinal cord injury. In conducting the admission assessment, what is the nurse's priority action? 1. Take the temperature. 2. Listen to breath sounds. 3. Observe for dyskinesias. 4. Assess extremity muscle strength.

Rationale: Because compromise of respiration is a leading cause of death in cervical cord injury, respiratory assessment is the highest priority. Assessment of temperature and strength can be done after adequate oxygenation is ensured. Because dyskinesias occur in cerebellar disorders, this is not as important a concern as in cord-injured clients unless head injury is suspected.

The nurse is conducting home visits with a head-injured client with residual cognitive deficits. The client has problems with memory, has a shortened attention span, is easily distracted, and processes information slowly. The nurse plans to talk with the primary health care provider about referring the client to which professional? 1. A psychologist 2. A social worker 3. A neuropsychologist 4. A vocational rehabilitation specialist

Rationale: Clients with cognitive deficits after head injury may benefit from referral to a neuropsychologist, who specializes in evaluating and treating cognitive problems. The neuropsychologist plans an individual program of therapy and initiates counseling to help the client reach maximal potential. The neuropsychologist works in collaboration with other disciplines that are involved in the client's care and rehabilitation. Options 1, 2, and 4 are incorrect because these health care workers do not specialize in evaluating and treating cognitive problems.

The nurse has given instructions for taking codeine sulfate to a client with a severe headache. Which statement by the client indicates the teaching has been effective? 1. "I should increase fluid intake." 2. "I should maintain a low-fiber diet." 3. "I should avoid all exercise to prevent lightheadedness." 4. "I should avoid the use of stool softeners to prevent diarrhea."

Rationale: Codeine sulfate can cause constipation, so the client is instructed to increase fluid intake to prevent this occurrence. A high-fiber diet and stool softeners may be prescribed to prevent constipation. All exercise is not avoided.

The nurse is caring for a client receiving mannitol (Osmitrol) via intravenous (IV) infusion. A vial is sent from the pharmacy, and in preparing the medication the nurse notes that the vial contains crystals. What is the most appropriate nursing action? 1. Discard the vial. 2. Place the vial in warm water. 3. Send the vial back to the pharmacy. 4. Shake the vial to dissolve the crystals.

Rationale: Crystals form in a mannitol solution if the solution is cooled, but they will quickly dissolve if the container is placed in warm water and then cooled to body temperature before administration. However, if crystals remain after the warming procedure, the medication should not be used and should be returned to the pharmacy. The nurse would not discard the medication. The medication is not initially returned to the pharmacy because it is not defective. Shaking the vial should not be done and will not dissolve the crystals.

A nurse is reviewing the record of a child with increased intracranial pressure and notes that the child has exhibited signs of decerebrate posturing. Which assessment finding should the nurse expect if this type of posturing were present? 1. Flexion of the upper extremities and extension of the lower extremities 2. Unilateral or bilateral postural change in which the extremities are rigid 3. Abnormal extension of the upper and lower extremities with some internal rotation 4. Arms are adducted with fists clenched and the legs are flaccid with external rotation

Rationale: Decerebrate (extension) posturing is an abnormal extension of the upper extremities, with internal rotation of the upper arm and wrist and extension of the lower extremities with some internal rotation. Option 1 describes decorticate posturing. Options 2 and 4 are incorrect and not characteristics of decerebrate posturing.

The nurse is preparing a plan of care for a child with a head injury. On review of the records, the nurse notes that the health care provider has documented decorticate posturing. The nurse plans care knowing that this type of posturing indicates which finding? 1. Damage to the pons 2. Damage to the midbrain 3. Damage to the diencephalon 4. A lesion in the cerebral hemisphere

Rationale: Decorticate posturing indicates a lesion in the cerebral hemisphere or disruption of the corticospinal tracts. Decerebrate posturing indicates damage in the diencephalon, midbrain, or pons.

The nurse has a prescription to give dexamethasone (Decadron) by the intravenous (IV) route to a client with cerebral edema. How should the nurse prepare this medication? 1. Diluting the medication in 500 mL of 5% dextrose 2. Preparing an undiluted direct injection of the medication 3. Diluting the medication in 1 mL of lactated Ringer's solution for direct injection 4. Diluting the medication in 10% dextrose in water and administering it as a direct injection

Rationale: Dexamethasone may be given by direct IV injection or IV infusion. For IV infusion, it may be mixed with 50 to 100 mL of 0.9% sodium chloride or 5% dextrose in water. It is not mixed with lactated Ringer's solution or 10% dextrose in water.

Dexamethasone (Decadron) intravenously is prescribed for the client with cerebral edema. The nurse prepares the medication for administration and plans to perform which action? 1. Mix the medication in 1000 mL of 5% dextrose. 2. Prepare an undiluted direct injection of the medication. 3. Mix the medication in 100 mL of lactated Ringer's solution. 4. Dilute the medication in lactated Ringer's solution and administer as a direct injection.

Rationale: Dexamethasone may be given by direct intravenous injection or intravenous infusion. Dexamethasone may be mixed with 0.9% sodium chloride or 5% dextrose. If administered as an infusion, a minimum amount of diluting solution is needed.

A nurse preparing to administer carbamazepine (Tegretol) notices each of the following items on the client's breakfast tray. Which item should be a cause for concern and should be removed from the tray? 1. Carton of milk 2. Scrambled eggs 3. Grapefruit juice 4. Toast with honey

Rationale: Grapefruit juice can increase peak and trough levels of carbamazepine by 40%. Accordingly, clients taking the medication should be advised to avoid grapefruit juice. The other foods can be taken with this medication.

A nurse is caring for a client who has been taking hydrocodone for the last 3 months. Which side/adverse effects of this medication should the nurse assess the client for? 1. Tachycardia and hypertension 2. Diarrhea and abdominal cramping 3. Psychological and physical dependence 4. Increased respiratory rate and bronchospasm

Rationale: Hydrocodone is an opioid analgesic that also has antitussive properties. Side /adverse effects of this medication include physical and psychological dependence, bradycardia and hypotension, respiratory depression, nausea, vomiting, constipation, sedation, and confusion.

A client is experiencing delirium. The nurse concludes that which areas of the nervous system are affected? 1. Temporal lobe and frontal lobe 2. Hippocampus and frontal lobe 3. Limbic system and cerebral hemispheres 4. Reticular activating system and cerebral hemispheres

Rationale: Insomnia, agitation, mania, and delirium are caused by excessive arousal of the reticular activating system in conjunction with the cerebral hemispheres. The temporal lobe, hippocampus, and frontal lobe are responsible for memory. The limbic system is responsible for feelings and affect.

A child is admitted to the hospital with a diagnosis of acute bacterial meningitis. In reviewing the health care provider's prescriptions, which would the nurse question as appropriate for a child with this diagnosis? 1. Administer an oral antibiotic. 2. Maintain strict intake and output. 3. Draw blood for a culture and sensitivity. 4. Place the child on droplet precautions in a private room.

Rationale: Medication to treat acute bacterial meningitis is administered intravenously, not orally. A culture and sensitivity is done to determine if the diagnosis is bacterial or viral. Until meningitis is ruled out, the child is placed in isolation on droplet precautions because the disease is spread by airborne means. Strict intake and output should be maintained.

A nurse is caring for a client with an intracranial pressure (ICP) monitoring device. The nurse should become most concerned if the ICP readings drifted to and stayed in the vicinity of which finding? 1. 5 mm Hg 2. 8 mm Hg 3. 14 mm Hg 4. 22 mm Hg

Rationale: Normal ICP readings range from 5 to 15 mm Hg pressure. Pressures greater than 20 mm Hg are considered to represent increased ICP, which seriously impairs cerebral perfusion.

Propofol (Diprivan) is prescribed to induce sedation in a client who is intubated and is being mechanically ventilated. The nurse prepares the medication and selects which solution to dilute the medication? Refer to Figure. 1. A 2. B 3. C 4. D

Rationale: Propofol is an anesthetic agent that is used to provide continuous sedation in a client receiving mechanical ventilation. It is diluted only with 5% dextrose (D5W).

A client has suffered a head injury affecting the occipital lobe of the brain. The nurse anticipates that the client may experience difficulty with which sense? 1. Smell 2. Taste 3. Vision 4. Hearing

Rationale: The occipital lobe is responsible for reception of vision and contains visual association areas. This area of the brain helps the individual to visually recognize and understand the surroundings. The other senses listed are not a function of the occipital lobe.

A nurse is testing the spinal reflexes of a client during neurological assessment. Which reflex will assist in determining that the client has an adequate spinal reflex? 1. Cough reflex 2. Withdrawal reflex 3. Accommodation reflex 4. Munroe-Kellie reflex

Rationale: The withdrawal reflex is one of the spinal reflexes. It is an abrupt withdrawal of a body part in response to painful or injurious stimuli. The cough reflex is a brainstem-associated reflex. Accommodation reflex is associated with cranial nerve III and is part of the ocular motor system. Munroe-Kellie is not a reflex; it is a doctrine or a hypothesis addressing the cerebral volume relationships among the brain, the cerebrospinal fluid, and intracranial blood and their cumulative impact on intracranial pressure.

A client has a prescription for valproic acid (Depakene) 250 mg once daily. To maximize the client's safety, the nurse should plan to schedule the medication at what time? 1. At bedtime 2. With lunch 3. After breakfast 4. Before breakfast

Rationale: Valproic acid is an anticonvulsant that causes central nervous system (CNS) depression. For this reason, the side effects of the medication include sedation, dizziness, ataxia, and confusion. When the client is taking this medication as a single daily dose, administering it at bedtime negates the risk of injury from sedation and enhances client safety. The medication should be taken at the same time each day.

The nurse is caring for a client who has undergone a craniotomy and has a supratentorial incision. The nurse should place the client in which position postoperatively? 1. Head of bed flat, head and neck midline 2. Head of bed flat, head turned to the nonoperative side 3. Head of bed elevated 30 to 45 degrees, head and neck midline 4. Head of bed elevated 30 to 45 degrees, head turned to the operative side

After supratentorial surgery, the head is kept at a 30- to 45-degree angle. The head and neck should not be angled either anteriorly or laterally, but rather should be kept in a neutral (midline) position. This promotes venous return through the jugular veins, which will help prevent a rise in intracranial pressure.

A client has a medication prescription for phenytoin (Dilantin) to be administered by the intravenous route. After drawing up the medication, the nurse notes the presence of precipitate in the syringe. Which action should the nurse take? 1. Discard the syringe and begin again. 2. Add sterile water to dissolve the precipitate. 3. Draw up an additional 1 mL of normal saline into the syringe. 4. Chart the medication as "not given," and write a note in the medical record.

Home History Help Calculator Study ModeQuestion 101 of 336 Previous 101 ▲ ▼ Go Next Stop Bookmark Rationale Strategy Reference(s) Submit A client has a medication prescription for phenytoin (Dilantin) to be administered by the intravenous route. After drawing up the medication, the nurse notes the presence of precipitate in the syringe. Which action should the nurse take?Rationale: If the injectable solution is not clear or if precipitate is present, the medication should not be used and should be discarded. The nurse may have to call the pharmacy department to obtain another vial of the medication. Options 2, 3, and 4 are inaccurate actions.

The nurse is assessing a client's gait, which is characterized by unsteadiness and staggering steps. The nurse determines the presence of which type of gait? 1. Spastic 2. Ataxic 3. Festinating 4. Dystrophic or broad-based

Home History Help Calculator Study ModeQuestion 76 of 336 Previous 76 ▲ ▼ Go Next Stop Bookmark Rationale Strategy Reference(s) Submit The nurse is assessing a client's gait, which is characterized by unsteadiness and staggering steps. The nurse determines the presence of which type of gait?Rationale: An ataxic gait is characterized by unsteadiness and staggering. A spastic gait is characterized by stiff, short steps with the legs held together, hip and knees flexed, and toes that catch and drag. A festinating gait is best described as walking on the toes with an accelerating pace. A dystrophic or broad-based gait is seen as waddling, with the weight shifting from side to side and the legs far apart.

The nurse is assigned to care for an 8-year-old child with a diagnosis of a basilar skull fracture. The nurse reviews the health care provider's (HCPs) prescriptions and should contact the HCP to question which prescription? 1. Suction as needed. 2. Obtain daily weight. 3. Provide clear liquid intake. 4. Maintain a patent intravenous line.

Rationale: A basilar skull fracture is a type of head injury. Nasotracheal suctioning is contraindicated in a child with a basilar skull fracture: Because of the nature of the injury, there is a possibility that the catheter will enter the brain through the fracture, creating a high risk of secondary infection. Fluid balance is monitored closely by daily weight determination, intake and output measurement, and serum osmolality determination to detect early signs of water retention, excessive dehydration, and states of hypertonicity or hypotonicity. The child is maintained on NPO status or restricted to clear liquids until it is determined that vomiting will not occur. An intravenous line is maintained to administer fluids or medications if necessary.

A client with a neurological impairment experiences urinary incontinence. Which nursing action would be most helpful in assisting the client to adapt to this alteration? 1. Using adult diapers 2. Inserting a Foley catheter 3. Establishing a toileting schedule 4. Padding the bed with an absorbent cotton pad

Rationale: A bladder retraining program, such as use of a toileting schedule, may be helpful to clients experiencing urinary incontinence. A Foley catheter should be used only when necessary because of the associated risk of infection. Use of diapers or pads is the least acceptable alternative because of the risk of skin breakdown.

The nurse is developing a plan of care for a newborn infant with spina bifida (myelomeningocele type). The nurse includes assessment measures in the plan to monitor for increased intracranial pressure. Which assessment technique should be performed that will best detect the presence of an increase in intracranial pressure? 1. Check urine for specific gravity. 2. Monitor for signs of dehydration. 3. Assess anterior fontanel for bulging. 4. Assess blood pressure for signs of hypotension.

Rationale: A bulging or taut anterior fontanel would indicate the presence of increased intracranial pressure. Urine concentrating ability is not well developed at the newborn stage of development. Monitoring for signs of dehydration will not provide data related to increased intracranial pressure. Blood pressure is difficult to assess during the newborn period and is not the best indicator of intracranial pressure.

The nurse is caring for the client with increased intracranial pressure. The nurse would note which trend in vital signs if the intracranial pressure is rising? 1. Increasing temperature, increasing pulse, increasing respirations, decreasing blood pressure 2. Increasing temperature, decreasing pulse, decreasing respirations, increasing blood pressure 3. Decreasing temperature, decreasing pulse, increasing respirations, decreasing blood pressure 4. Decreasing temperature, increasing pulse, decreasing respirations, increasing blood pressure

Rationale: A change in vital signs may be a late sign of increased intracranial pressure. Trends include increasing temperature and blood pressure and decreasing pulse and respirations. Respiratory irregularities also may occur.

A client is about to undergo a lumbar puncture. The nurse describes to the client that which position will be used during the procedure? 1. Side-lying with a pillow under the hip 2. Prone with a pillow under the abdomen 3. Prone in slight Trendelenburg's position 4. Side-lying with the legs pulled up and the head bent down onto the chest

Rationale: A client undergoing lumbar puncture is positioned lying on the side, with the legs pulled up to the abdomen and the head bent down onto the chest. This position helps open the spaces between the vertebrae and allows for easier needle insertion by the health care provider. The nurse remains with the client during the procedure to help the client maintain this position. The other options identify incorrect positions for this procedure.

The nurse is preparing a plan of care for a client with a diagnosis of amyotrophic lateral sclerosis (ALS). On assessment, the nurse notes that the client is severely dysphagic. Which intervention should be included in the care plan for this client? Select all that apply. 1. Provide oral hygiene after each meal. 2. Assess swallowing ability frequently. 3. Allow the client sufficient time to eat. 4. Maintain a suction machine at the bedside. 5. Provide a full liquid diet for ease in swallowing.

Rationale: A client who is severely dysphagic is at risk for aspiration. Swallowing is assessed frequently. The client should be given a sufficient amount of time to eat. Semisoft foods are easiest to swallow and require less chewing. Oral hygiene is necessary after each meal. Suctioning should be available for clients who experience dysphagia and are at risk for aspiration.

A client was seen and treated in the hospital emergency department for treatment of a concussion. The nurse determines that the family needs reinforcement of the discharge instructions if they verbalize to call the health care provider (HCP) for which client sign or symptom? 1. Vomiting 2. Minor headache 3. Difficulty speaking 4. Difficulty awakening

Rationale: A concussion after head injury is a temporary loss of consciousness (from a few seconds to a few minutes) without evidence of structural damage. After concussion, the family is taught to monitor the client and call the HCP or return the client to the emergency department for signs and symptoms such as confusion, difficulty awakening or speaking, one-sided weakness, vomiting, and severe headache. Minor headache is expected.

The nurse is preparing for the admission of a client with a suspected diagnosis of Guillain-Barré syndrome. When the client arrives at the nursing unit, the nurse reviews the health care provider's documentation. The nurse expects to note documentation of which hallmark clinical manifestation of this syndrome? 1. Multifocal seizures 2. Altered level of consciousness 3. Abrupt onset of a fever and headache 4. Development of progressive muscle weakness

Rationale: A hallmark clinical manifestation of Guillain-Barré syndrome is progressive muscle weakness that develops rapidly. Seizures are not normally associated with this disorder. The client does not have symptoms such as a fever or headache. Cerebral function, level of consciousness, and pupillary responses are normal.

A nurse is transcribing a prescription for antibiotic therapy for a client with bacterial meningitis. The nurse understands that the medication that will be prescribed for the client will have which characteristic? 1. Metabolizes slowly 2. Acts within minutes to hours 3. Crosses the blood-brain barrier 4. Excretes readily and easily in the urine

Rationale: A key consideration in prescribing medications that will affect the brain is the ability of the medication to cross the blood-brain barrier. If the medication cannot pass into the brain, it will not be effective. The duration of action, onset, and excretion (options 1, 2, and 4) apply to most medications and most uses but are not specific to those needed to treat meningitis.

The nurse assists a health care provider in performing a lumbar puncture on a 3-year-old child with leukemia in whom central nervous system disease is suspected. In which position will the nurse place the child during this procedure? 1. Lithotomy position 2. Modified Sims position 3. Prone with knees flexed to the abdomen and head bent with chin resting on the chest 4. Lateral recumbent position with the knees flexed to the abdomen and head bent with the chin resting on chest

Rationale: A lateral recumbent position with the knees flexed to the abdomen and the head bent with the chin resting on the chest is assumed for a lumbar puncture. This position separates the spinal processes and facilitates needle insertion into the subarachnoid space. Options 1, 2, and 3 are incorrect positions.

The nurse receives a telephone call from the admissions office and is told that a child with acute bacterial meningitis will be admitted to the pediatric unit. The nurse prepares for the child's arrival and plans to implement which type of precautions? 1. Enteric 2. Contact 3. Droplet 4. Neutropenic

Rationale: A major priority in nursing care for a child with suspected meningitis is to administer the appropriate antibiotic as soon as it is prescribed. The child will be placed in a private room, with droplet transmission precautions, for at least 24 hours after antibiotics are given. Enteric, contact, and neutropenic precautions are not associated with the mode of transmission of meningitis. Enteric precautions are instituted when the mode of transmission is through the gastrointestinal tract. Contact precautions are instituted when contact with infectious items or materials is likely. Neutropenic precautions are instituted when the client has a low neutrophil count.

The nurse is caring for a newborn infant with spina bifida (myelomeningocele) who is scheduled for surgical closure of the sac. In the preoperative period, which is the priority problem? 1. Infection 2. Choking 3. Inability to tolerate stimulation 4. Delayed growth and development

Rationale: A myelomeningocele is a type of spina bifida that results from failure of the neural tube to close during embryonic development. With a myelomeningocele, protrusion of the meninges, cerebrospinal fluid, nerve roots, and a portion of the spinal cord occurs. The newborn with spina bifida is at risk for infection before the closure of the sac, which is done soon after birth. Initial care of the newborn with myelomeningocele involves prevention of infection. A sterile normal saline dressing is placed over the sac to maintain moisture of the sac and its contents and to prevent tearing or breakdown of the skin integrity at the site. Any opening in the sac greatly increases the risk of infection of the central nervous system. Choking and inability to tolerate stimulation are not priority problems with this defect. Delayed growth and development is a problem for the infant with myelomeningocele, but preventing infection has priority in the preoperative period.

A nurse is preparing to administer a prescribed antibiotic to a client with bacterial meningitis. The nurse understands that the selection of an antibiotic to treat meningitis is based on which fact? 1. It has a long half-life. 2. It acts within minutes to hours. 3. It can be easily excreted in the urine. 4. It is able to cross the blood-brain barrier.

Rationale: A primary consideration regarding medications to treat bacterial meningitis is the ability of the medication to cross the blood-brain barrier. If the medication cannot cross, it will not be effective. The duration, onset, and excretion of the medication(options 1, 2, and 3) are also of general concern but apply to all medications and not specifically to those that are used to treat meningitis.

The nurse is developing a plan of care for a child who is at risk for seizures. Which interventions apply if the child has a seizure? Select all that apply. 1. Time the seizure. 2. Restrain the child. 3. Stay with the child. 4. Place the child in a prone position. 5. Move furniture away from the child. 6. Insert a padded tongue blade in the child's mouth.

Rationale: A seizure is a disorder that occurs as a result of excessive and unorganized neuronal discharges in the brain that activate associated motor and sensory organs. During a seizure, the child is placed on his or her side in a lateral position. Positioning on the side prevents aspiration because saliva drains out the corner of the child's mouth. The child is not restrained because this could cause injury to the child. The nurse would loosen clothing around the child's neck and ensure a patent airway. Nothing is placed into the child's mouth during a seizure because this action may cause injury to the child's mouth, gums, or teeth. The nurse would stay with the child to reduce the risk of injury and allow for observation and timing of the seizure.

The nurse develops a plan of care for a child at risk for tonic-clonic seizures. In the plan of care, the nurse identifies seizure precautions and documents that which item(s) need to be placed at the child's bedside? 1. Emergency cart 2. Tracheotomy set 3. Padded tongue blade 4. Suctioning equipment and oxygen

Rationale: A seizure results from the excessive and unorganized neuronal discharges in the brain that activate associated motor and sensory organs. A type of generalized seizure is a tonic-clonic seizure. This type of seizure causes rigidity of all body muscles, followed by intense jerking movements. Because increased oral secretions and apnea can occur during and after the seizure, oxygen and suctioning equipment are placed at the bedside. A tracheotomy is not performed during a seizure. No object, including a padded tongue blade, is placed into the child's mouth during a seizure. An emergency cart would not be left at the bedside, but would be available in the treatment room or nearby on the nursing unit.

A thymectomy accomplished via a median sternotomy approach is performed in a client with a diagnosis of myasthenia gravis. The nurse develops a postoperative plan of care for the client that should include which intervention? 1. Monitor the chest tube drainage. 2. Restrict visitors for 24 hours postoperatively. 3. Maintain intravenous infusion of lactated Ringer's solution. 4. Avoid administering pain medication to prevent respiratory depression.

Rationale: A thymectomy may be used for management of clients with myasthenia gravis. The procedure is performed through a median sternotomy or a transcervical approach. Postoperatively the client will have a chest tube in the mediastinum. Lactated intravenous solutions usually are avoided because they can increase weakness. Pain medication is administered as needed, but the client is monitored closely for respiratory depression. There is no reason to restrict visitors.

The nurse is performing an assessment of a 7-year-old child who is suspected of having episodes of absence seizures. Which assessment question to the mother will assist in providing information that will identify the symptoms associated with this type of seizure? 1. "Does twitching occur in the face and neck?" 2. "Does the muscle twitching occur on one side of the body?" 3. "Does the muscle twitching occur on both sides of the body?" 4. "Does the child have a blank expression during these episodes?"

Rationale: Absence seizures are brief episodes of altered awareness or momentary loss of consciousness. No muscle activity occurs except eyelid fluttering or twitching. The child has a blank facial expression. These seizures last only 5 to 10 seconds, but they may occur one after another several times a day. Simple partial seizures consist of twitching of an extremity, face, or neck, or the sensation of twitching or numbness in an extremity or face or neck. Myoclonic seizures are brief random contractions of a muscle group that can occur on one or both sides of the body.

A nurse is collecting data on a 7-year-old child who is suspected of having episodes of absence seizures. Which question should the nurse ask the parents to identify the symptoms associated with these types of seizures? 1. "Does the muscle twitching occur on one side of the body?" 2. "Does the muscle twitching occur on both sides of the body?" 3. "Does the sensation of twitching occur in the face and neck?" 4. "Does the child have a blank expression during these episodes?"

Rationale: Absence seizures are very brief episodes of altered awareness. There is no muscle activity except eyelid fluttering or twitching. The child has a blank facial expression. These seizures last only 5 to 10 seconds but may occur one after another several times a day. Myoclonic seizures are brief, random contractions of a muscle group that can occur on one or both sides of the body. Simple partial seizures consist of twitching of an extremity, the face, or the neck, or the sensation of twitching or numbness in an extremity, the face, or the neck.

Acetazolamide (Diamox) is prescribed for a client hospitalized with a diagnosis of a supratentorial lesion. The nurse understands that which is the primary action of the medication? 1. Prevention of hypertension 2. Prevention of hyperthermia 3. Decrease in cerebrospinal fluid production 4. Maintenance of blood pressure adequate for cerebral perfusion

Rationale: Acetazolamide (Diamox) is a carbonic anhydrase inhibitor. It is used in the client with or at risk for increased intracranial pressure to decrease cerebrospinal fluid production. Options 1, 2, and 4 are not actions of this medication.

A client recovering from a head injury is participating in care. The nurse determines that the client understands measures to prevent elevations in intracranial pressure if the nurse observes the client doing which activity? 1. Blowing the nose 2. Isometric exercises 3. Coughing vigorously 4. Exhaling during repositioning

Rationale: Activities that increase intrathoracic and intraabdominal pressures cause an indirect elevation of the intracranial pressure. Some of these activities include isometric exercises, Valsalva's maneuver, coughing, sneezing, and blowing the nose. Exhaling during activities such as repositioning or pulling up in bed, opens the glottis, which prevents intrathoracic pressure from rising.

A client with a spinal cord injury becomes angry and belligerent whenever the nurse tries to administer care. The nurse should perform which action? 1. Ask the family to deliver the care. 2. Leave the client alone until ready to participate. 3. Advise the client that rehabilitation progresses more quickly with cooperation. 4. Acknowledge the client's anger and continue to encourage participation in care.

Rationale: Adjusting to paralysis is physically and psychosocially difficult for the client and family. The nurse recognizes that the client goes through the grieving process in adjusting to the loss and may move back and forth among the stages of grief. The nurse acknowledges the client's feelings while continuing to meet the client's physical needs and encouraging independence. The family also is in crisis and needs the nurse's support and should not be relied on to provide care. The nurse cannot simply neglect the client until the client is ready to participate. Option 3 represents a factual but noncaring approach to the client and is not therapeutic.

A nurse is reading the laboratory results for a client being treated with carbamazepine (Tegretol) for prophylaxis of complex partial seizures. When evaluating the client's laboratory data, the nurse determines that which value is consistent with an adverse effect of this medication? 1. Sodium level, 136 mEq/L 2. Platelet count, 350,000 cells/mm3 3. White blood cell count, 3200 cells/mm3 4. Blood urea nitrogen (BUN), 19 mg/dL

Rationale: Adverse effects of carbamazepine appear as blood dyscrasias, including aplastic anemia, agranulocytosis, thrombocytopenia, and leukopenia. Other adverse effects include cardiovascular disturbances, thrombophlebitis, dysrhythmias, and dermatological effects.

A client with trigeminal neuralgia is being treated with carbamazepine (Tegretol), 400 mg orally daily. Which value indicates that the client is experiencing an adverse effect to the medication? 1. Uric acid level, 5 mg/dL 2. Sodium level, 140 mEq/L 3. Blood urea nitrogen level, 15 mg/dL 4. White blood cell count, 3000 cells/mm3

Rationale: Adverse effects of carbamazepine appear as blood dyscrasias, including aplastic anemia, agranulocytosis, thrombocytopenia, and leukopenia; cardiovascular disturbances including thrombophlebitis and dysrhythmias; and dermatological effects. The low white blood cell count reflects agranulocytosis. The laboratory values in options 1, 2, and 3 are normal values.

A nurse is evaluating the neurological status of a client. To assess the function of the limbic system, the nurse should gather data about which item? 1. Experience of pain 2. Affect or emotions 3. Response to verbal stimuli 4. Insight, judgment, and planning

Rationale: Affect and emotions are part of the role of the limbic system and involve both hemispheres of the brain. Pain is a complex experience involving several areas of the central nervous system. The response to verbal stimuli is part of the level of consciousness, which is under the control of the reticular activating system and both cerebral hemispheres. Insight, judgment, and planning are part of the functions of the frontal lobes of the brain in conjunction with association fibers connecting to other areas of the cerebrum.

A nurse is caring for a child who sustained a head injury after falling from a tree. On assessment of the child, the nurse notes the presence of a watery discharge from the child's nose. The nurse should immediately test the discharge for the presence of which substance? 1. Protein 2. Glucose 3. Neutrophils 4. White blood cells

Rationale: After a head injury, bleeding from the nose or ears necessitates further evaluation. A watery discharge from the nose (rhinorrhea) that tests positive for glucose is likely to be cerebrospinal fluid (CSF) leaking from a skull fracture. On noting watery discharge from the child's nose, the nurse should test the drainage for glucose using reagent strips such as Dextrostix. If the results are positive, the nurse will contact the health care provider. The items in options 1, 3, and 4 are not normally found in CSF.

The nurse is assessing fluid balance in a client who has undergone a craniotomy. The nurse should assess for which finding as a sign of overhydration, which would aggravate cerebral edema? 1. Unchanged weight 2. Shift intake 950 mL, output 900 mL 3. Blood urea nitrogen (BUN) 10 mg/dL 4. Serum osmolality 280 mOsm/kg H2O

Rationale: After craniotomy the goal is to keep the serum osmolality on the high side of normal to minimize excess body water and control cerebral edema. The normal serum osmolality is 285 to 295 mOsm/kg H2O. A higher value indicates dehydration; a lower value indicates overhydration. Stable weight indicates that there is neither fluid excess nor fluid deficit. A difference of 50 mL in intake and output for an 8-hour shift is insignificant. The BUN of 10 mg/dL is within normal range and does not indicate overhydration or underhydration.

The nurse has made a judgment that a client who had a craniotomy is experiencing a problem with body image. The nurse develops goals for the client but determines that the client has not met the outcome criteria by discharge if the client performs which action? 1. Wears a turban to cover the incision 2. Indicates that facial puffiness will be a permanent problem 3. Verbalizes that periorbital bruising will disappear over time 4. States an intention to purchase a hairpiece until hair has grown back

Rationale: After craniotomy, clients may experience difficulty with altered personal appearance. The nurse can help by listening to the client's concerns and by clarifying any misconceptions about facial edema, periorbital bruising, and hair loss (all of which are temporary). The nurse can encourage the client to participate in self-grooming and use personal articles of clothing. Finally, the nurse can suggest the use of a turban, followed by a hairpiece, to help the client adapt to the temporary change in appearance.

The student nurse develops a plan of care for a client after a lumbar puncture. The nursing instructor corrects the student if the student documents which incorrect intervention in the plan? 1. Maintain the client in a flat position. 2. Restrict fluid intake for a period of 2 hours. 3. Assess the client's ability to void and move the extremities. 4. Inspect the puncture site for swelling, redness, and drainage.

Rationale: After the lumbar puncture the client remains flat in bed for at least 2 hours, depending on the health care provider's prescriptions. A liberal fluid intake is encouraged to replace the cerebrospinal fluid removed during the procedure, unless contraindicated by the client's condition. The nurse checks the puncture site for redness and drainage and assesses the client's ability to void and move the extremities.

The nurse is preparing to care for a client after a lumbar puncture. The nurse should plan to place the client in which best position immediately after the procedure? 1. Prone in semi-Fowler's position 2. Supine in semi-Fowler's position 3. Prone with a small pillow under the abdomen 4. Lateral with the head slightly lower than the rest of the body

Rationale: After the procedure, the client assumes a flat position. If the client is able, a prone position with a pillow under the abdomen is the best position. This position helps reduce cerebrospinal fluid leakage and decreases the likelihood of post-lumbar puncture headache. Options 1, 2, and 4 are incorrect.

A client with a diagnosis of trigeminal neuralgia is started on a regimen of carbamazepine (Tegretol). The nurse provides instructions to the client about the side/adverse effects of the medication. Which client statement indicates an understanding of the side/adverse effects of the medication? 1. "I will report a fever or sore throat to my health care provider." 2. "I must brush my teeth frequently to avoid damage to my gums." 3. "If I notice ringing in my ears that doesn't stop, I'll seek medical attention." 4. "If I notice a pink color to my urine, I will stop the medication and call my health care provider (HCP)."

Rationale: Agranulocytosis is adverse effect of carbamazepine and places the client at risk for infection. If a fever or a sore throat develops, the HCP should be notified. Gum damage, ringing in the ears, and pink-colored urine are not effects associated with this medication.

A client with a spinal cord injury at the level of C5 has a weakened respiratory effort and ineffective cough and is using accessory neck muscles in breathing. The nurse carefully monitors the client and suspects the presence of which complication? 1. Altered breathing pattern 2. Increased likelihood of injury 3. Ineffective oxygen consumption 4. Increased susceptibility to aspiration

Rationale: Altered breathing pattern indicates that the respiratory rate, depth, rhythm, timing, or chest wall movements are insufficient for optimal ventilation of the client. This is a risk for clients with spinal cord injury in the lower cervical area. Ineffective oxygen consumption occurs when oxygenation or carbon dioxide elimination is altered at the alveolar-capillary membrane. Increased susceptibility to aspiration and increased likelihood of injury are unrelated to the focus of the question.

An 84-year-old client in an acute state of disorientation is brought to the hospital emergency department by his or her daughter. The daughter states that the client was "clear as a bell this morning." The nurse determines from this piece of information that which is an unlikely cause of the disorientation? 1. Hypoglycemia 2. Alzheimer's disease 3. Medication dosage error 4. Impaired circulation to the brain

Rationale: Alzheimer's disease is a chronic disease with progression of memory deficits over time. The situation presented in the question represents an acute problem. Evaluation is necessary to determine whether hypoglycemia, medication use, or impaired cerebral circulation has had a role in causing the client's current symptoms.

A client has been diagnosed with Alzheimer's disease. The nurse concludes that the client has a pathological condition of which components of the nervous system? 1. Glia 2. Peripheral nerves 3. Neuronal dendrites 4. Monoamine oxidase

Rationale: Alzheimer's disease is characterized by changes in the dendrites of the neurons. The decrease in the number and composition of the dendrites is responsible for the symptoms of the disease. The components in the other options are not related to the pathology of Alzheimer's disease.

A client with a diagnosis of Parkinson's disease began taking amantadine (Symmetrel) approximately 2 weeks ago. The client reports to the clinic for a follow-up evaluation. The nurse determines that the client is experiencing an adverse effect related to the use of this medication if which is noted? 1. Decreased rigidity 2. Decreased akinesia 3. A blood pressure of 118/74 mm Hg 4. Client complaints of urinary retention

Rationale: Amantadine is an antiparkinson agent that potentiates the action of dopamine in the central nervous system. The expected effect of therapy is a decrease in akinesia and rigidity. Leukopenia, urinary retention, and hypotension all are adverse effects of the medication.

A client began taking amantadine (Symmetrel) approximately 2 weeks ago. The nurse determines that the medication is having a therapeutic effect if the client exhibits which finding? 1. Decreased voiding 2. Decreased blood pressure 3. Decreased rigidity and akinesia 4. Decreased white blood cell count

Rationale: Amantadine is an antiparkinson agent that potentiates the action of dopamine in the central nervous system. The expected effect of therapy is a decrease in akinesia and rigidity. Urinary retention, hypotension, and leukopenia are adverse effects of the medication.

The nurse who is caring for a client with myasthenia gravis has a prescription to perform an Enlon (Tensilon) test. After obtaining edrophonium (Enlon), the nurse should be certain that which also is available at the bedside? 1. Atropine sulfate 2. Protamine sulfate 3. Calcium gluconate 4. Magnesium sulfate

Rationale: An Enlon test is performed to distinguish between myasthenic and cholinergic crisis. After administration of the Enlon, if symptoms intensify, the crisis is cholinergic. Because the symptoms of cholinergic crisis will worsen with the administration of Enlon, atropine sulfate should be available because it is the antidote.

A nurse caring for a child who has sustained a head injury in an automobile crash is monitoring the child for signs of increased intracranial pressure (ICP). For which early sign of increased ICP should the nurse monitor? 1. Increased systolic blood pressure 2. Abnormal posturing of extremities 3. Significant widening pulse pressure 4. Changes in level of consciousness (LOC)

Rationale: An altered level of consciousness is an early sign of increased ICP. Late signs of increased ICP include tachycardia leading to bradycardia, apnea, systolic hypertension, widening pulse pressure, and posturing.

A client with myasthenia gravis has become increasingly weaker. The health care provider prepares to identify whether the client is reacting to an overdose of the medication (cholinergic crisis) or an increasing severity of the disease (myasthenic crisis). An injection of edrophonium is administered. Which finding would indicate that the client is in cholinergic crisis? 1. No change in the condition 2. Complaints of muscle spasms 3. An improvement of the weakness 4. A temporary worsening of the condition

Rationale: An edrophonium injection makes the client in cholinergic crisis temporarily worse. An improvement in the weakness indicates myasthenia crisis. Muscle spasms are not associated with this test.

A client with myasthenia gravis becomes increasingly weaker. The health care provider (HCP) injects a dose of edrophonium (Enlon) to determine whether the client is experiencing a myasthenic crisis or a cholinergic crisis. The nurse expects that the client will have which reaction if in cholinergic crisis? 1. No change in the condition 2. Complaints of muscle spasms 3. An improvement of the weakness 4. A temporary worsening of the condition

Rationale: An edrophonium injection makes the client in cholinergic crisis temporarily worse. An improvement of the condition indicates myasthenic crisis. The other two options are unrelated to the test.

A nurse caring for an infant with a diagnosis of hydrocephalus should monitor the infant for which sign of increased intracranial pressure (ICP)? 1. Proteinuria 2. Bradycardia 3. A drop in blood pressure 4. A bulging anterior fontanel

Rationale: An elevated or bulging anterior fontanel indicates an increase in cerebrospinal fluid collection in the cerebral ventricle. Proteinuria, bradycardia, and a drop in blood pressure are not specific signs of increased ICP. Changes in the level of consciousness and a widened pulse pressure are additional signs of increased ICP.

The nurse is planning to put aneurysm precautions in place for a client with a cerebral aneurysm. Which nursing measure would be a potentially unsafe component of the precautions? 1. Provide physical aspects of care. 2. Prevent pushing or straining activities. 3. Maintain the head of the bed at 15 degrees. 4. Limit caffeinated coffee to one cup per day.

Rationale: Aneurysm precautions include placing the client on bed rest (as prescribed) in a quiet setting. Stimulants such as caffeine and nicotine are prohibited; decaffeinated coffee or tea may be used. Lights are kept dim to minimize environmental stimulation. Any activity that increases the blood pressure or impedes venous return from the brain is prohibited, such as pushing, pulling, sneezing, coughing, or straining. The nurse provides physical care to minimize increases in blood pressure. For the same reason, visitors, radio, television, and reading materials are prohibited or limited.

The nurse develops a plan of care for a client with a brain aneurysm who will be placed on aneurysm precautions. Which interventions should be included in the plan? Select all that apply. 1. Leave the lights on in the client's room at night. 2. Place a blood pressure cuff at the client's bedside. 3. Close the shades in the client's room during the day. 4. Allow the client to drink one cup of caffeinated coffee a day. 5. Allow the client to ambulate four times a day with assistance.

Rationale: Aneurysm precautions include placing the client on bed rest in a quiet setting. The use of lights is kept to a minimum to prevent environmental stimulation. The nurse should monitor the blood pressure and note any changes that could indicate rupture. Any activity, such as pushing, pulling, sneezing, or straining, that increases the blood pressure or impedes venous return from the brain is prohibited. The nurse provides physical care to minimize increases in blood pressure. Visitors, radio, television, and reading materials are restricted or limited. Stimulants, such as nicotine and coffee and other caffeine-containing products, are prohibited. Decaffeinated coffee or tea may be used.

The nurse is providing home care instructions to the parents of a child with a seizure disorder. Which statement indicates to the nurse that the teaching regarding seizure disorders has been effective? 1. "We're glad we only have to give our child the medication for 30 days." 2. "We will make appointments for follow-up blood work and care as directed." 3. "We're glad there are no side effects from taking the antiseizure medications." 4. "After our child has been seizure free for 1 month, we can discontinue the medication."

Rationale: Antiseizure medications are continued for a prolonged time even if seizures are controlled. Periodic reevaluation of the child is important to assess the continued effectiveness of the medication, check serum medication levels, and determine the need to alter the dosage if indicated. Antiseizure medications have potential side effects, and parents should be informed of such effects specific to the medication the child will be taking. Withdrawal of medication follows a predesigned protocol, usually begun when the child has been seizure free for at least 2 years. When a medication is discontinued, the dosage should be reduced gradually over 1 to 2 weeks.

A student nurse is assisting with an assessment of a client's level of consciousness using the Glasgow Coma Scale. The student understands that which categories of client functioning are included in this assessment? Select all that apply. 1. Eye opening 2. Reflex response 3. Best verbal response 4. Best motor response 5. Pupil size and reaction

Rationale: Assessment of pupil size and reaction and reflex response are not part of the Glasgow Coma Scale. The three categories included are eye opening, best verbal response, and best motor response. Pupil assessment and reflex response is a necessary part of a total assessment of the neurological status of a client but is not part of this particular scale.

The nurse is assessing a client with a brainstem injury. In addition to obtaining the client's vital signs and determining the Glasgow Coma Scale score, what priority intervention should the nurse plan to implement? 1. Check cranial nerve functioning. 2. Determine the cause of the accident. 3. Draw blood for arterial blood gas analysis. 4. Perform a pulmonary wedge pressure measurement.

Rationale: Assessment should be specific to the area of the brain involved. The respiratory center is located in the brainstem. Assessing the respiratory status is the priority for a client with a brainstem injury. Options 1, 2, and 4 are not priorities although they may be a component in the assessment process, depending on the injury and client condition.

The nurse is performing an assessment on a client admitted to the nursing unit with a diagnosis of stroke (brain attack). On assessment, the nurse notes that the client is unable to understand spoken language. The nurse plans care, understanding that the client is experiencing impairment of which areas? 1. The occipital lobe 2. The auditory association areas 3. The frontal lobe and optic nerve tracts 4. Concept formation and abstraction areas

Rationale: Auditory association and storage areas are located in the temporal lobe and relate to understanding spoken language. The occipital lobe contains areas related to vision. The frontal lobe controls voluntary muscle activity, including speech, and an impairment can result in expressive aphasia. The parietal lobe contains association areas for concept formation, abstraction, spatial orientation, body and object size and shape, and tactile sensation.

A client who has a spinal cord injury that resulted in paraplegia experiences a sudden onset of severe headache and nausea. The client is diaphoretic with piloerection and has flushing of the skin. The client's systolic blood pressure (BP) is 210 mm Hg. What should the nurse immediately suspect? 1. Return of spinal shock 2. Malignant hypertension 3. Impending brain attack (stroke) 4. Autonomic dysreflexia (hyperreflexia)

Rationale: Autonomic dysreflexia (hyperreflexia) results from sudden strong discharge of the sympathetic nervous system in response to a noxious stimulus. Signs and symptoms include pounding headache, nausea, nasal stuffiness, flushed skin, piloerection, and diaphoresis. Severe hypertension can occur, with a systolic BP rising potentially as high as 300 mm Hg. It often is triggered by thermal or mechanical events such as a kinking of catheter tubing, constipation, urinary tract infection, or any variety of cutaneous stimuli. The nurse must recognize this situation immediately and take corrective action to remove the stimulus. If untreated, this medical emergency could result in stroke, status epilepticus, or possibly death.

The nurse is caring for a client who sustained a spinal cord injury. During administration of morning care, the client begins to exhibit signs and symptoms of autonomic dysreflexia. Which initial nursing action should the nurse take? 1. Elevate the head of the bed. 2. Examine the rectum digitally. 3. Assess the client's blood pressure. 4. Place the client in the prone position.

Rationale: Autonomic dysreflexia is a serious complication that can occur in the spinal cord-injured client. Once the syndrome is identified, the nurse elevates the head of the client's bed and then examines the client for the source of noxious stimuli. The nurse also assesses the client's blood pressure, but the initial action would be to elevate the head of the bed. The client would not be placed in the prone position.

A postoperative craniotomy client who sustained a severe head injury is admitted to the neurological unit. What important nursing intervention is necessary for this client? 1. Take and record vital signs every 4 to 8 hours. 2. Prophylactically hyperventilate during the first 20 hours. 3. Treat a central fever with the administration of antipyretic medications such as acetaminophen (Tylenol). 4. Keep the head of the bed elevated at least 30 degrees, and position the client to avoid extreme flexion or extension of the neck and head.

Rationale: Avoiding extreme flexion and extension of the neck can enhance venous drainage and help prevent increased intracranial pressure. As a general rule, hyperventilation is avoided during the first 20 hours postoperatively because it may produce ischemia caused by cerebral vasoconstriction. Vital signs need to be taken and recorded at least every 1 to 2 hours. Central fevers caused by hypothalamic damage respond better to cooling (hypothermia blankets, sponge baths) than to the administration of antipyretic medications.

The nurse is caring for a client with trigeminal neuralgia (tic douloureux). The client asks for a snack and something to drink. The nurse should offer which best snack to the client? 1. Cocoa with honey and toast 2. Hot herbal tea with graham crackers 3. Iced coffee and peanut butter and crackers 4. Vanilla wafers and room-temperature water

Rationale: Because mild tactile stimulation of the face can trigger pain in trigeminal neuralgia, the client needs to eat or drink lukewarm, nutritious foods that are soft and easy to chew. Extremes of temperature will cause trigeminal nerve pain. Therefore, options 1, 2, and 3 are incorrect.

The nurse is planning care for a client with intracranial pressure (ICP) monitoring. Which intervention is appropriate to include in the plan of care? 1. Place the client in Sims position. 2. Change the drainage tubing every 48 hours. 3. Level the transducer at the lowest point of the ear. 4. Use strict aseptic technique when touching the monitoring system.

Rationale: Because there is a foreign body embedded in the client's brain, vigilant aseptic technique should be implemented. Sims is side-lying, flat position. With a client who has increased ICP, the head of the bed should be elevated at least 30% to improve jugular outflow. The drainage tubing should not be routinely changed. It should remain for the duration of the monitoring. To obtain accurate ICP pressure readings, the transducer is zeroed at the level of the foramen of Monro, which is approximated by placing the transducer 1 inch above the level of the ear. Serial ICP readings should be done with the client's head in the same position.

The home care nurse is performing an assessment on a client with a diagnosis of Bell's palsy. Which assessment question will elicit the most specific information regarding this client's disorder? 1. "Do your eyes feel dry?" 2. "Do you have any spasms in your throat?" 3. "Are you having any difficulty chewing food?" 4. "Do you have any tingling sensations around your mouth?"

Rationale: Bell's palsy is a one-sided facial paralysis caused by compression of the facial nerve. Manifestations include facial droop from paralysis of the facial muscles; increased lacrimation; painful sensations in the eye, face, or behind the ear; and speech or chewing difficulties.

The nurse is performing an assessment on a client with a diagnosis of Bell's palsy. The nurse should expect to observe which finding in the client? 1. Facial drooping 2. Periorbital edema 3. Ptosis of the eyelid 4. Twitching on the affected side of the face

Rationale: Bell's palsy is a one-sided facial paralysis caused by the compression of the facial nerve (cranial nerve VII). Assessment findings include facial droop from paralysis of the facial muscles; increased lacrimation; painful sensations in the eye, face, or behind the ear; and speech or chewing difficulty. Options 2, 3, and 4 are not associated findings in Bell's palsy.

The home care nurse is visiting a client with a diagnosis of Parkinson's disease. The client is taking benztropine mesylate (Cogentin) orally daily. The nurse provides information to the spouse regarding the side effects of this medication and should tell the spouse to report which side effect if it occurs? 1. Shuffling gait 2. Inability to urinate 3. Decreased appetite 4. Irregular bowel movements

Rationale: Benztropine mesylate is an anticholinergic, which causes urinary retention as a side effect. The nurse would instruct the client or spouse about the need to monitor for difficulty with urinating, a distended abdomen, infrequent voiding in small amounts, and overflow incontinence. Options 1, 3, and 4 are unrelated to the use of this medication.

A client with Parkinson's disease is taking benztropine mesylate (Cogentin) orally daily. In monitoring this client for medication side effects, the nurse should plan to focus the assessment on which item? 1. Pupil response 2. Voiding pattern 3. Prothrombin time 4. Respiratory status

Rationale: Benztropine mesylate is an anticholinergic. Because urinary retention is a side effect of benztropine mesylate, the nurse must assess for dysuria, distended abdomen, infrequent voiding in small amounts, and overflow incontinence. Monitoring of the other options is not necessary with this medication.

Benztropine mesylate (Cogentin) is prescribed for a client with a diagnosis of Parkinson's disease. What statement by the client indicates that the client needs additional information about the medication? 1. "I will avoid driving if I get drowsy or dizzy." 2. "I'll watch my urinary output and look for signs of constipation." 3. "I will sit in the sun for an hour a day to enhance medication effectiveness." 4. "I will call the health care provider (HCP) if I have difficulty swallowing or if I start vomiting."

Rationale: Benztropine mesylate is an anticholinergic. The client taking benztropine mesylate may have decreased tolerance to heat as a result of diminished ability to sweat and should plan rest periods in cool places during the day. The client is instructed to avoid driving or operating hazardous equipment if drowsy or dizzy. The client is also instructed to monitor urinary output and watch for signs of constipation. The client should be instructed to contact the HCP if difficulty swallowing or speaking develops, vomiting occurs, or central nervous system effects occur. Additionally, the use of anticholinergic drugs should be avoided in older adults because they can cause confusion, urinary retention, constipation, dry mouth, and blurred vision.

The nurse in the neurological unit is caring for a client who was in a motor vehicle crash and sustained a blunt head injury. On assessment of the client, the nurse notes the presence of bloody drainage from the nose. Which nursing action is most appropriate? 1. Insert nasal packing. 2. Document the findings. 3. Contact the health care provider (HCP). 4. Monitor the client's blood pressure and check for signs of increased intracranial pressure.

Rationale: Bloody or clear drainage from either the nasal or the auditory canal after head trauma could indicate a cerebrospinal fluid leak. The appropriate nursing action is to notify the HCP, because this finding requires immediate intervention. Options 1, 2, and 4 are inappropriate nursing actions in this situation.

A client has suffered damage to Broca's area of the brain. The nurse providing care for this client anticipates that which area will be affected? 1. Speech 2.Hearing 3.Balance 4. Level of consciousness

Rationale: Broca's area in the brain is responsible for the motor aspects of speech, through coordination of the muscular activity of the tongue, mouth, and larynx. The term assigned to damage in this area is aphasia. The items listed in the other options are not the responsibility of Broca's area.

The nurse is performing an assessment on a client with the diagnosis of Brown-Séquard syndrome. The nurse would expect to note which assessment finding? 1. Bilateral loss of pain and temperature sensation 2. Ipsilateral paralysis and loss of touch and vibration 3. Contralateral paralysis and loss of touch, pressure, and vibration 4. Complete paraplegia or quadriplegia, depending on the level of injury

Rationale: Brown-Séquard syndrome results from hemisection of the spinal cord, resulting in ipsilateral paralysis and loss of touch, pressure, vibration, and proprioception. Contralaterally, pain and temperature sensation are lost because these fibers decussate after entering the cord. Options 1, 3, and 4 are not assessment findings in this syndrome.

A client has a high level of carbon dioxide (CO2) in the bloodstream, as measured by arterial blood gases. A nurse reviewing the client's record plans care, knowing that a high CO2 level will have which effect on circulation to the brain? 1. It will cause arteriovenous shunting. 2. It will cause vasodilation of blood vessels in the brain. 3. It will cause blood vessels in the circle of Willis to collapse. 4. It will cause hyperresponsiveness of blood vessels in the brain.

Rationale: CO2 is one of the metabolic end products that can alter the tone of the blood vessels in the brain. High CO2 levels cause vasodilation, which may cause headache, whereas low CO2 levels cause vasoconstriction, which may cause lightheadedness. The statements included in the other options are incorrect effects.

A client had a transsphenoidal resection of the pituitary gland. The nurse notes drainage on the nasal dressing. Suspecting cerebrospinal fluid (CSF) leakage, the nurse should look for drainage that is of which characteristic? 1. Serosanguineous only 2. Bloody with very small clots 3. Sanguineous only with no clot formation 4. Serosanguineous, surrounded by clear to straw-colored fluid

Rationale: CSF leakage after cranial surgery may be detected by noting drainage that is serosanguineous (from the surgery) and surrounded by an area of clear or straw-colored drainage. The typical appearance of CSF drainage is that of a "halo." The nurse also would further verify actual CSF drainage by testing the drainage for glucose, which would be positive.

The nurse is assisting with caloric testing of the oculovestibular reflex in an unconscious client. Cold water is injected into the left auditory canal. The client exhibits eye conjugate movements toward the left, followed by eye movement back to midline. The nurse understands that this finding indicates which situation? 1. Brain death 2. A cerebral lesion 3. A temporal lesion 4. An intact brainstem

Rationale: Caloric testing provides information about differentiating between cerebellar and brainstem lesions. After determining patency of the ear canal, cold or warm water is injected into the auditory canal. A normal response that indicates intact function of cranial nerves III, VI, and VIII is conjugate eye movements toward the side being irrigated, followed by eye movement back to midline. Absent or dysconjugate eye movements indicate brainstem damage.

A client is scheduled to begin therapy with carbamazepine (Tegretol). The nurse should assess the results of which test(s) before administering the first dose of this medication to the client? 1. Liver function tests 2. Renal function tests 3. Pancreatic enzyme studies 4. Complete blood cell count

Rationale: Carbamazepine (Tegretol) can cause leukopenia, anemia, thrombocytopenia, and, very rarely, fatal aplastic anemia. To reduce the risk of serious hematological effects, a complete blood cell count should be done before treatment and periodically thereafter. This medication should be avoided in clients with preexisting hematological abnormalities. The client also is told to report the occurrence of fever, sore throat, pallor, weakness, infection, easy bruising, and petechiae. Options 1, 2, and 3 are not associated with the use of this medication.

A client taking carbamazepine (Tegretol) asks the nurse what to do if a dose is inadvertently missed. The nurse responds that which action should be taken? 1. Withhold until the next scheduled dose. 2. Withhold and call the health care provider (HCP). 3. Take the dose as long as it is not close to the time for the next dose. 4. Withhold until the next scheduled dose, which should then be doubled.

Rationale: Carbamazepine is an anticonvulsant that should be taken around the clock, precisely as directed. If a dose is omitted, the client should take the dose as soon as it is remembered, as long as it is not close to the time that the next dose is due. The medication should not be double-dosed. If more than one dose is omitted, the client should call the HCP.

A nurse is collecting data from a client and notes that the client is taking carbamazepine (Tegretol). The nurse determines that this medication has been prescribed to treat which condition? 1. Glaucoma 2. Diabetes mellitus 3. Parkinson's disease 4. Trigeminal neuralgia

Rationale: Carbamazepine is classified as an iminostilbene derivative and is used as an anticonvulsant, antineuralgic, antimanic, and antipsychotic. It is not used to treat any of the conditions noted in options 1, 2, or 3.

The health care provider (HCP) writes a prescription for carbamazepine (Tegretol) for a client who was admitted to the hospital. The nurse contacts the HCP to verify the prescription if which condition is noted in the assessment data? 1. Hypertension 2. Tonic-clonic seizures 3. Trigeminal neuralgia 4. Bone marrow depression

Rationale: Carbamazepine is classified as an iminostilbene derivative and is used as an anticonvulsant, antineuralgic, antimanic, and antipsychotic. It is used to treat seizure disorders, trigeminal neuralgia, and diabetic neuropathy. The medication can cause blood dyscrasias as an adverse effect and is contraindicated if the client has a history of bone marrow depression or hypersensitivity to tricyclic antidepressants or concurrent use of monoamine oxidase inhibitors.

Carbamazepine (Tegretol) has been prescribed for a client, and the client asks the nurse about the action of the medication. The nurse's response should incorporate which information as a primary effect of this medication? 1. Anticonvulsant effect 2. Prevention of cellular division 3. Decrease in intraocular pressure 4. Interference with DNA production

Rationale: Carbamazepine is classified as an iminostilbene derivative and is used as an anticonvulsant, antineuralgic, antimanic, and antipsychotic. Options 2, 3, and 4 are not actions or effects of this medication.

Carbamazepine (Tegretol) has been prescribed for a client. The nurse should tell the client that which blood test will be done periodically while the client is taking this medication? 1. Lipase level 2. Amylase level 3. Ammonia level 4. Complete blood cell (CBC) count

Rationale: Carbamazepine is classified as an iminostilbene derivative and is used as an anticonvulsant, antineuralgic, antimanic, and antipsychotic. The medication can cause blood dyscrasias as an adverse effect, and the client should have a CBC done before therapy and periodically during therapy. Additional laboratory tests that should be done include a serum iron determination, urinalysis, blood urea nitrogen determination, and a carbamazepine level. The tests identified in options 1, 2, and 3 are unnecessary.

The parents of a child recently diagnosed with cerebral palsy ask the nurse about the disorder. The nurse bases the response on the understanding that cerebral palsy is which type of condition? 1. An infectious disease of the central nervous system 2. An inflammation of the brain as a result of a viral illness 3. A congenital condition that results in moderate to severe retardation 4. A chronic disability characterized by impaired muscle movement and posture

Rationale: Cerebral palsy is a chronic disability characterized by impaired movement and posture resulting from an abnormality in the extrapyramidal or pyramidal motor system. Meningitis is an infectious process of the central nervous system. Encephalitis is an inflammation of the brain that occurs as a result of viral illness or central nervous system infection. Down syndrome is an example of a congenital condition that results in moderate to severe retardation.

The nurse is performing an assessment on a client with a diagnosis of thrombotic brain attack (stroke). Which assessment question would elicit data specific to this type of stroke? 1. "Have you had any headaches in the past few days?" 2. "Have you recently been having difficulty with seeing at nighttime?" 3. "Have you had any sudden episodes of passing out in the past few days?" 4. "Have you had any numbness or tingling or paralysis-type feelings in any of your extremities recently?"

Rationale: Cerebral thrombosis (thrombotic stroke) does not occur suddenly. In the few days or hours preceding the thrombotic stroke, the client may experience a transient loss of speech, hemiparesis, or paresthesias on one side of the body. Signs and symptoms of this type of stroke vary but may also include dizziness, cognitive changes, or seizures. Headache is rare, but some clients with stroke experience signs and symptoms similar to those of cerebral embolism or intracranial hemorrhage. The client does not complain of difficulty with night vision as part of this clinical problem. In addition, most clients do not have repeated episodes of loss of consciousness.

The nurse is reviewing the medical records of a client admitted to the nursing unit with a diagnosis of a thrombotic brain attack (stroke). The nurse would expect to note that which is documented in the assessment data section of the record? 1. Sudden loss of consciousness occurred. 2. Signs and symptoms occurred suddenly. 3. The client experienced paresthesias a few days before admission to the hospital. 4. The client complained of a severe headache, which was followed by sudden onset of paralysis.

Rationale: Cerebral thrombosis does not occur suddenly. In the few hours or days preceding a thrombotic brain attack (stroke), the client may experience a transient loss of speech, hemiplegia, or paresthesias on one side of the body. Signs and symptoms of thrombotic brain attack (stroke) vary but may include dizziness, cognitive changes, or seizures. Headache is rare, but some clients with brain attack (stroke) experience signs and symptoms similar to those of cerebral embolism or intracranial hemorrhage.

The nurse is assessing the nasal dressing on a client who had a transsphenoidal resection of the pituitary gland. The nurse notes a small amount of serosanguineous drainage that is surrounded by clear fluid on the nasal dressing. Which nursing action is most appropriate? 1. Document the findings. 2. Reinforce the dressing. 3. Notify the health care provider (HCP). 4. Mark the area of drainage with a pen and monitor for further drainage.

Rationale: Cerebrospinal fluid (CSF) leakage after cranial surgery may be detected by noting drainage that is serosanguineous surrounded by an area of straw-colored or pale drainage. The physical appearance of CSF drainage is that of a halo. If the nurse notes the presence of this type of drainage, the HCP needs to be notified. Options 1, 2, and 4 are inappropriate nursing actions.

The client with a cervical spine injury has cervical tongs applied in the emergency department. What should the nurse avoid when planning care for this client? 1. Using a Roto-Rest bed 2. Removing the weights to reposition the client 3. Assessment of the integrity of the weights and pulleys 4. Comparing the amount of prescribed traction with the amount in use

Rationale: Cervical tongs are applied after drilling holes in the client's skull under local anesthesia. Weights are attached to the tongs, which exert pulling pressure on the longitudinal axis of the cervical spine. Serial x-rays of the cervical spine are taken, with weights being added gradually until the x-ray reveals that the vertebral column is realigned. After that, weights may be reduced gradually to a point that maintains alignment. The client with cervical tongs is placed on a Stryker frame or Roto-Rest bed. The nurse ensures that weights hang freely, and the amount of weight matches the current prescription. The nurse also inspects the integrity and position of the ropes and pulleys. The nurse does not remove the weights to administer care.

The nurse is providing home care instructions to the mother of a child who is recovering from Reye's syndrome. Which instruction should the nurse provide to the mother? 1. Increase stimuli in the home environment. 2. Avoid daytime naps so that the child will sleep at night 3. Give the child frequent small meals, if vomiting occurs. 4. Check the skin and eyes every day for a yellow discoloration.

Rationale: Checking for jaundice will assist in identifying the presence of liver complications, which are characteristic of Reye's syndrome. Decreasing stimuli and providing rest decrease stress on the brain tissue. If vomiting occurs in Reye's syndrome, it is caused by cerebral edema and is a sign of intracranial pressure.

A school-age child with Down syndrome is brought to the ambulatory care center by the mother. The child has bruising all over the body. To work most effectively with this child, the nurse first addresses which complication associated with Down syndrome? 1. Children with Down syndrome are more likely to develop acute leukemia than the average child. 2. Children with Down syndrome fall down easily as a result of hyperflexibility and muscle weakness. 3. Children with Down syndrome are at risk for physical abuse because of their low intellectual functioning. 4. Children with Down syndrome scratch themselves a lot because of dry, cracked, and frequently fissuring skin.

Rationale: Children with Down syndrome have an increased risk for developing leukemia compared with the average child. The other statements also could be true, but the nurse should first gather baseline data to determine the cause of the bruising before making other assumptions.

A nurse has a prescription to administer a medication to a client who is experiencing shivering as a result of hyperthermia. Which medication should the nurse anticipate to be prescribed? 1. Buspirone (BuSpar) 2. Chlorpromazine (Thorazine) 3. Prochlorperazine (Compazine) 4. Fluphenazine (Prolixin Decanoate)

Rationale: Chlorpromazine is used to control shivering in hyperthermic states. It is a phenothiazine and has antiemetic and antipsychotic uses, especially when psychosis is accompanied by increased psychomotor activity. Buspirone is an anxiolytic. Prochlorperazine is a phenothiazine that is an antiemetic and antipsychotic. Fluphenazine is a phenothiazine that is used as an antipsychotic.

The nurse is assessing the adaptation of a client to changes in functional status after a stroke (brain attack). Which observation indicates to the nurse that the client is adapting most successfully? 1. Gets angry with family if they interrupt a task 2. Experiences bouts of depression and irritability 3. Has difficulty with using modified feeding utensils 4. Consistently uses adaptive equipment in dressing self

Rationale: Clients are evaluated as coping successfully with lifestyle changes after a brain attack (stroke) if they make appropriate lifestyle alterations, use the assistance of others, and have appropriate social interactions. Options 1 and 2 are not adaptive behaviors; option 3 indicates a not yet successful attempt to adapt.

A home care nurse visits a client at home. Clonazepam (Klonopin) has been prescribed for the client, and the nurse teaches the client about the medication. Which client statement indicates that further teaching is necessary? 1. "My drowsiness will decrease over time with continued treatment." 2. "I should take my medicine with food to avoid any stomach problems." 3. "I can take my medicine at bedtime if it tends to make me feel drowsy." 4. "If I experience slurred speech, this problem will disappear in about 8 weeks."

Rationale: Clients who are experiencing signs and symptoms of toxicity with the administration of clonazepam exhibit slurred speech, sedation, confusion, respiratory depression, hypotension, and eventually coma. The medication may be taken with food to decrease gastrointestinal irritation. Some drowsiness may occur but will decrease with continued use. Options 1, 2, and 3 all are correct and represent an accurate understanding of the medication.

A client with recent-onset Bell's palsy is upset and crying about the change in facial appearance. The nurse plans to support the client emotionally by making which statement to the client? 1. This is caused by a small tumor, which can be removed easily. 2. This is not a brain attack (stroke), and many clients recover in 3 to 5 weeks. 3. This is a temporary problem, with treatment similar to that for migraine headaches. 4. This is similar to a brain attack (stroke), but all symptoms will reverse without treatment.

Rationale: Clients with Bell's palsy should be reassured that they have not experienced a brain attack (stroke) and that symptoms often disappear spontaneously in 3 to 5 weeks. The client is given supportive treatment for symptoms. Bell's palsy usually is not caused by a tumor, and the treatment is not similar to that for migraine headaches.

The nurse reviews the health care provider's (HCP) prescriptions for a client with Guillain-Barré syndrome. Which prescription written by the HCP should the nurse question? 1. Clear liquid diet 2. Bilateral calf measure 3. Monitor vital signs frequently 4. Passive range-of-motion (ROM) exercises

Rationale: Clients with Guillain-Barré syndrome have dysphagia. Clients with dysphagia are more likely to aspirate clear liquids than thick or semisolid foods. Passive ROM exercises can help prevent contractures, and assessing calf measurements can help detect deep vein thrombosis, for which these clients are at risk. Because clients with Guillain-Barré syndrome are at risk for hypotension or hypertension, bradycardia, and respiratory depression, frequent monitoring of vital signs is required.

The nurse is preparing to ambulate a client with Parkinson's disease who has recently been started on methyldopa (levodopa). Before performing this activity with the client, the nurse should include which most important assessment in the client's plan of care? 1. History of falls 2. Use of assistive devices 3. Postural (orthostatic) vital signs 4. Degree of exhibited intention tremor

Rationale: Clients with Parkinson's disease are at risk for postural (orthostatic) hypotension from the disease. This problem is exacerbated with the introduction of levodopa, which also can cause postural hypotension and increase the client's risk for falls. Although knowledge of the client's use of assistive devices and history of falls is helpful, neither of these options is the most important element of the assessment, based on the wording of this question. Clients with Parkinson's disease generally have resting tremor, not intention tremor.

The nurse is trying to communicate with a client with brain attack and aphasia. Which action by the nurse would be least helpful to the client? 1. Speaking to the client at a slower rate 2. Allowing plenty of time for the client to respond 3. Completing the sentences that the client cannot finish 4. Looking directly at the client during attempts at speech

Rationale: Clients with aphasia after brain attack often fatigue easily and have a short attention span. General guidelines when trying to communicate with the aphasic client include speaking more slowly and allowing adequate response time, listening to and watching attempts to communicate, and trying to put the client at ease with a caring and understanding manner. The nurse would avoid shouting (because the client is not deaf), appearing rushed for a response, and letting family members provide all the responses for the client.

A client with myasthenia gravis arrives at the hospital emergency department in suspected crisis. The health care provider plans to administer edrophonium to differentiate between myasthenic and cholinergic crises. The nurse ensures that which medication is available in the event that the client is in cholinergic crisis? 1. Atropine sulfate 2. Morphine sulfate 3. Protamine sulfate 4. Pyridostigmine bromide

Rationale: Clients with cholinergic crisis have experienced overdosage of medication. Edrophonium will exacerbate symptoms in cholinergic crisis to the point at which the client may need intubation and mechanical ventilation. Intravenous atropine sulfate is used to reverse the effects of these anticholinesterase medications. Morphine sulfate and pyridostigmine bromide would worsen the symptoms of cholinergic crisis. Protamine sulfate is the antidote for heparin.

The nurse is planning care for a client who displays confusion secondary to a neurological problem. Which approach by the nurse would be least helpful in assisting this client? 1. Providing sensory cues 2. Giving simple, clear directions 3. Providing a stable environment 4. Encouraging multiple visitors at one time

Rationale: Clients with cognitive impairment from neurological dysfunction respond best to a stable environment that is limited in amount and type of sensory input. The nurse can provide sensory cues and give clear, simple directions in a positive manner. Confusion can be minimized by reducing environmental stimuli (such as television or multiple visitors) and by keeping familiar personal articles (such as family pictures) at the bedside.

The home care nurse is making extended follow-up visits to a client discharged from the hospital after a moderately severe head injury. The family states that the client is behaving differently than before the accident. The client is more fatigued and irritable and has some memory problems. The client, who was previously very even-tempered, is prone to outbursts of temper now. The nurse counsels the family on the basis of an understanding that these behaviors are indicative of which condition? 1. Indicate a worsening of the original injury 2. Will probably be a long-term sequela of the injury 3. Will come and go as intracranial pressure changes 4. Are short-term problems that will resolve in about 1 month

Rationale: Clients with moderate to severe head injury usually have residual physical and cognitive disabilities; these include personality changes, increased fatigue and irritability, mood alterations, and memory changes. The client also may require frequent to constant supervision. The nurse assesses the family's ability to cope and makes appropriate referrals to respite services, support groups, and state or local chapters of the National Head Injury Foundation.

The nurse is teaching a client with myasthenia gravis about the prevention of myasthenic and cholinergic crises. Which client activity suggests that teaching is most effective? 1. Eating large, well-balanced meals 2. Doing muscle-strengthening exercises 3. Doing all chores early in the day while less fatigued 4. Taking medications on time to maintain therapeutic blood levels

Rationale: Clients with myasthenia gravis are taught to space out activities over the day to conserve energy and restore muscle strength. Taking medications correctly to maintain blood levels that are not too low or too high is important. Muscle-strengthening exercises are not helpful and can fatigue the client. Overeating is a cause of exacerbation of symptoms, as is exposure to heat, crowds, erratic sleep habits, and emotional stress.

A client is having the dosage of clonazepam (Klonopin) adjusted. The nurse should plan to perform which action? 1. Weigh the client daily. 2. Monitor blood glucose levels. 3. Institute seizure precautions. 4. Observe for areas of ecchymosis.

Rationale: Clonazepam is a benzodiazepine that is used as an anticonvulsant. During initial therapy and during periods of dosage adjustment, the nurse should initiate seizure precautions for the client. Weight, glucose levels, and ecchymosis are unrelated to this medication.

A client who was started on anticonvulsant therapy with clonazepam (Klonopin) tells the nurse of increasing clumsiness and unsteadiness since starting the medication. The client is visibly upset by these manifestations and asks the nurse what to do. The nurse's response is based on which understanding? 1. These symptoms probably result from interaction with another medication. 2. These symptoms usually occur when the client takes the medication with food. 3. These symptoms are most severe during initial therapy and decrease or disappear with long-term use. 4. These symptoms indicate that the client is experiencing a severe adverse reaction to the medication.

Rationale: Clonazepam is classified as a benzodiazepine and is used as an anticonvulsant and antianxiety agent. Drowsiness, unsteadiness, and clumsiness are expected effects of the medication during early therapy. They are dose related and usually diminish or disappear altogether with continued use of the medication. They are unrelated to an interaction with another medication. The client is encouraged to take this medication with food to minimize gastrointestinal upset. These symptoms do not indicate that an adverse effect is occurring.

A client with a history of simple partial seizures is taking clorazepate (Tranxene). The client asks the nurse if there is a risk of addiction with this medication. The nurse's response is based on which information about this medication? 1.Is not habit forming either physically or psychologically 2.Leads to physical tolerance, but only after 10 or more years of therapy 3.Leads to physical and psychological dependence with prolonged high-dose therapy 4.Can result in psychological dependence only because of the nature of the medication

Rationale: Clorazepate is classified as an anticonvulsant, an anxiolytic (antianxiety agent), and a sedative-hypnotic (benzodiazepine). One of the nursing implications of clorazepate therapy is that the medication can lead to physical or psychological dependence with prolonged therapy at high doses. For this reason, the amount of medication that is readily available to the client at any one time is restricted.

The nurse is preparing to give a postcraniotomy client medication for incisional pain. The family asks the nurse why the client is receiving codeine sulfate and not "something stronger." In formulating a response, the nurse incorporates which information about codeine sulfate? 1. Is one of the strongest opioid analgesics available 2. Cannot lead to physical or psychological dependence 3. Does not alter respirations or mask neurological signs as do other opioids 4. Does not cause gastrointestinal (GI) upset or constipation as do other opioids

Rationale: Codeine sulfate is an opioid analgesic used for clients after craniotomy. It often is combined with a nonopioid analgesic such as acetaminophen (Tylenol) for added effect. It does not alter the respiratory rate or mask neurological signs as do other opioids. Side effects of codeine sulfate include GI upset and constipation. Chronic use of the medication can lead to physical and psychological dependence.

A home health nurse visits a client who suffered a back injury. On review of the health care provider's prescriptions, the nurse notes that codeine sulfate has been prescribed for the client, and the nurse provides instructions to the client regarding the medication. Which statement, if made by the client, indicates an understanding of health measures related to the medication? 1. "The medication is not addicting." 2. "I should watch out for diarrhea as a side effect." 3. "I should increase my fluid intake while taking this medication." 4. "I need to be sure to eat foods that are low in fiber to prevent diarrhea."

Rationale: Codeine sulfate is an opioid analgesic used to treat pain and can cause constipation. Because it is an opioid analgesic, codeine sulfate can be addicting. The client is instructed to increase fluid intake to prevent constipation. The client also should consume foods high in fiber and should take a stool softener.

The client has an impairment of cranial nerve II. Specific to this impairment, what should the nurse should plan to do to ensure client safety? 1. Speak loudly to the client. 2. Test the temperature of the shower water. 3. Check the temperature of the food on the dietary tray. 4. Provide a clear path for ambulation without obstacles.

Rationale: Cranial nerve II is the optic nerve, which governs vision. The nurse can provide safety for the visually impaired client by clearing the path of obstacles when ambulating. Speaking loudly may help overcome a deficit of cranial nerve VIII (vestibulocochlear). Testing the shower water temperature would be useful if there were an impairment of peripheral nerves. Cranial nerve VII (facial) and IX (glossopharyngeal) control taste from the anterior two thirds and posterior third of the tongue, respectively.

The nurse in the neurological unit is monitoring a client for signs of increased intracranial pressure (ICP). The nurse reviews the assessment findings for the client and notes documentation of the presence of Cushing's reflex. The nurse determines that the presence of this reflex is obtained by assessing which item? 1. Blood pressure 2. Motor response 3. Pupillary response 4. Level of consciousness

Rationale: Cushing's reflex is a late sign of increased ICP and consists of a widening pulse pressure (systolic pressure rises faster than diastolic pressure) and bradycardia. Options 2, 3, and 4 are unrelated to monitoring for Cushing's reflex.

The nurse is monitoring a child with a brain tumor for complications associated with increased intracranial pressure (ICP). Which finding, if noted by the nurse, would indicate the presence of diabetes insipidus (DI)? 1. Weight gain 2. Hypertension 3. High urine output 4. Urine specific gravity greater than 1.020

Rationale: DI can occur in a child with increased ICP. Weight gain, hypertension and a urine specific gravity greater than 1.020 are indications of the syndrome of inappropriate antidiuretic hormone (SIADH) secretion, not DI. A high urine output would be indicative of DI.

Dantrolene sodium (Dantrium) has been administered to a client with a spinal cord injury. The nurse determines that the client is experiencing an adverse effect of the medication if which is noted? 1. Dizziness 2. Drowsiness 3. Abdominal pain 4. Lightheadedness

Rationale: Dantrium is hepatotoxic. The nurse observes for indications of liver dysfunction, which include jaundice, abdominal pain, and malaise. The nurse notifies the health care provider if these occur. The items in options 1, 2, and 4 are expected side effects due to the central nervous system (CNS)-depressant effects of the medication.

A client experiencing spasticity as a result of spinal cord injury has a new prescription for dantrolene (Dantrium). Before administering the first dose, the nurse checks to see if which baseline study has been done? 1. Liver function studies 2. Renal function studies 3. Otoscopic examination 4. Blood glucose measurements

Rationale: Dantrolene is a skeletal muscle relaxant and can cause liver damage; therefore the nurse should monitor the results of liver function studies. They should be done before therapy starts and periodically throughout therapy. Dantrolene is discontinued if no relief of spasticity is achieved in 6 weeks. The incorrect options are not specifically related to the administration of this medication.

The nurse is reviewing the record of a child with increased intracranial pressure and notes that the child has exhibited signs of decerebrate posturing. On assessment of the child, the nurse expects to note which characteristic of this type of posturing? 1. Flaccid paralysis of all extremities 2. Adduction of the arms at the shoulders 3. Rigid extension and pronation of the arms and legs 4. Abnormal flexion of the upper extremities and extension and adduction of the lower extremities

Rationale: Decerebrate (extension) posturing is characterized by the rigid extension and pronation of the arms and legs. Option 1 is incorrect. Options 2 and 4 describe decorticate (flexion) posturing.

The nurse is performing an assessment on a child with a head injury. The nurse notes an abnormal flexion of the upper extremities and an extension of the lower extremities. What should the nurse document that the child is experiencing? 1. Decorticate posturing 2. Decerebrate posturing 3. Flexion of the arms and legs 4. Normal expected positioning after head injury

Rationale: Decorticate posturing is an abnormal flexion of the upper extremities and an extension of the lower extremities with possible plantar flexion of the feet. Decerebrate posturing is an abnormal extension of the upper extremities with internal rotation of the upper arms and wrists and an extension of the lower extremities with some internal rotation.

A client with a spinal cord injury expresses little interest in food and is very particular about the choice of meals that are actually eaten. How should the nurse interpret this information? 1. Anorexia is a sign of clinical depression, and a referral to a psychologist is needed. 2. The client has compulsive habits that should be ignored so long as they are not harmful. 3. The client probably has a naturally slow metabolism, and the decreased nutritional intake will not matter. 4. Meal choices represent an area of client control and should be encouraged as much as is nutritionally reasonable.

Rationale: Depression frequently may be seen in the client with spinal cord injury and may be exhibited as a loss of appetite. However, the client should be allowed to choose the types of food eaten and when they are eaten as much as is feasible because it is one of the few areas of control that the client has left. There is no information in the query of the question that would indicate that the client is anorexic, obsessive-compulsive, or has a slow metabolism.

The nurse is developing a plan of care for an older client that addresses interventions to prevent cold discomfort and the development of accidental hypothermia. The nurse should document which desired outcome in the plan of care? 1. The client's body temperature is 98° F. 2. The client's fingers and toes are cool to touch. 3. The client remains in a fetal position when in bed. 4. The client complains of coolness in the hands and feet only.

Rationale: Desired outcomes for nursing interventions to prevent cold discomfort and the development of accidental hypothermia include the following: hands and limbs are warm; body is relaxed and not curled; body temperature is greater than 97° F; the client is not shivering; and the client has no complaints of feeling cold.

The nurse is caring for the client who suffered a spinal cord injury 48 hours ago. What should the nurse assess for when monitoring for gastrointestinal complications? 1. A history of diarrhea 2. A flattened abdomen 3. Hyperactive bowel sounds 4. Hematest-positive nasogastric tube drainage

Rationale: Development of a stress ulcer also can occur after spinal cord injury and can be detected by Hematest-positive nasogastric tube aspirate or stool. The client is also at risk for paralytic ileus, which is characterized by the absence of bowel sounds and abdominal distention. A history of diarrhea is irrelevant.

The nurse has the following prescription for a postcraniotomy client, "dexamethasone (Decadron) 4 mg by the intravenous (IV) route now." How does the nurse administer the medication? 1. IV push over 1 minute 2. IV push over 4 minutes 3. IV piggyback in 50 mL of normal saline over 10 minutes 4. IV piggyback in 50 mL of normal saline over 30 minutes

Rationale: Dexamethasone (Decadron) is an adrenocorticosteroid administered after craniotomy to control cerebral edema. It is given by IV push, and single doses are administered over 1 minute. Dexamethasone IV doses are changed to the oral route after 24 to 72 hours and are tapered until discontinued. Additionally, IV fluids are administered cautiously after craniotomy to prevent increased cerebral edema.

A client with narcolepsy has been prescribed dextroamphetamine (DextroStat). The client complains to the nurse that he cannot sleep well at night and does not want to take the medication any longer. Before making any specific comment, the nurse plans to investigate whether the client takes the medication at which proper time schedule? 1. 2 hours before bedtime 2. After supper each night 3. Just before going to sleep 4. At least 6 hours before bedtime

Rationale: Dextroamphetamine is a central nervous system (CNS) stimulant that acts by releasing norepinephrine from nerve endings. The client should take the medication at least 6 hours before going to bed at night to prevent sleep disturbance.

A client on the nursing unit has a prescription for dextroamphetamine (DextroStat) daily. The unit nurse collaborates with the dietitian to limit the amount of which item on the client's dietary trays? 1. Fat 2. Starch 3. Caffeine 4. Protein

Rationale: Dextroamphetamine is a central nervous system (CNS) stimulant. Caffeine is a stimulant also, so caffeine intake should be limited in the client taking this medication. He or she should be taught to limit his or her own caffeine intake as well. It is not necessary to limit fat, starch, or protein while taking this medication.

The nurse is providing diet instructions to a client with Ménière's disease who is being discharged from the hospital after admission for an acute attack. Which statement, if made by the client, indicates an understanding of the dietary measures to take to help prevent further attacks? 1. "I need to restrict my carbohydrate intake." 2. "I need to drink at least 3 L of fluid per day." 3. "I need to maintain a low-fat and low-cholesterol diet." 4. "I need to be sure to consume foods that are low in sodium."

Rationale: Dietary changes, such as salt and fluid restrictions, that reduce the amount of endolymphatic fluid are sometimes prescribed for the client with Ménière's disease. The client should be instructed to consume a low-sodium diet and restrict fluids as prescribed. Although helpful to treat other disorders, low-fat, low-carbohydrate, and low-cholesterol diets are not specifically prescribed for the client with Ménière's disease.

A client has received a dose of dimenhydrinate (Dramamine). The nurse evaluates the effect of the medication by noting whether the client obtained relief from what symptom? 1. Chills 2. Headache 3. Ringing in the ears 4. Nausea and vomiting

Rationale: Dimenhydrinate is an antiemetic and antihistamine used to treat and prevent the signs and symptoms of dizziness, vertigo, and nausea and vomiting that accompany motion sickness. The other options are incorrect; the medication is not used to treat these conditions.

The nurse in the health care clinic is providing medication instructions to a client with a seizure disorder who will be taking divalproex sodium (Depakote). The nurse should instruct the client about the importance of returning to the clinic for monitoring of which laboratory study? 1. Electrolyte panel 2. Liver function studies 3. Renal function studies 4. Blood glucose level determination

Rationale: Divalproex sodium, an anticonvulsant, can cause fatal hepatotoxicity. The nurse should instruct the client about the importance of monitoring the results of liver function studies and ammonia level determinations. Options 1, 3, and 4 are not studies that are required with the use of this medication.

The nurse is caring for a child diagnosed with Down's syndrome. In describing the disorder to the parents, what characteristics are most closely associated with the syndrome and serve as the basis for the nurse's explanation? 1. Subaverage intellectual functioning with a congenial nature 2. Above-average intellectual functioning with deficits in adaptive behavior 3. Moderate to severe retardation and linkage to an extra chromosome 21, group G 4. Average intellectual functioning and the absence of deficits in adaptive behavior

Rationale: Down's syndrome is a form of mental retardation and is a congenital condition that results in moderate to severe mental retardation. Most cases are attributable to an extra chromosome (group G)-hence the name trisomy 21. Options 1, 2, and 4 are incorrect characteristics of this syndrome.

A client is receiving phenobarbital sodium (Luminal) for the treatment of a seizure disorder. Which finding on the nursing assessment would indicate that the client is experiencing a common side effect of this medication? 1. Drowsiness 2. Hypocalcemia 3. Blurred vision 4. Seizure activity

Rationale: Drowsiness is a common side effect of phenobarbital, which is a barbiturate and antiseizure medication. Hypocalcemia is a rare toxic reaction. Blurred vision is not an associated side effect of this medication. Seizure activity could occur from abrupt withdrawal of this medication therapy or as a toxic reaction.

The nurse enters a child's room and discovers that the child is having a seizure. Which actions should the nurse take? Select all that apply. 1. Call a code. 2. Run to get the crash cart. 3. Turn the child on her side. 4. Loosen any restrictive clothing. 5. Check the child's respiratory status. 6. Place an airway into the child's mouth.

Rationale: During a seizure the child is placed on his or her side in a lateral position. Positioning on the side will prevent aspiration because saliva will drain out the corner of the child's mouth. The child is not restrained because this could cause injury to the child. The nurse would loosen clothing around the child's neck and ensure a patent airway by checking respiratory status. A code would be called if the child was not breathing or the heart is not beating. There are no data in the question indicating that this is the case. The nurse would stay with the child to reduce the risk of injury and allow for observation and timing of the seizure. Nothing is placed into the child's mouth during a seizure because this could injure the child's mouth, gums, or teeth.

Which nursing actions apply to the care of a child who is having a seizure? Select all that apply. 1. Time the seizure. 2. Restrain the child. 3. Stay with the child. 4. Insert an oral airway. 5. Place the child in a lateral side-lying position. 6. Loosen clothing around the child's neck.

Rationale: During a seizure, the child is placed on his or her side in a lateral position. Positioning on the side prevents aspiration because saliva drains out the corner of the child's mouth. The child is not restrained because this could cause injury to the child. The nurse should loosen clothing around the child's neck and ensure a patent airway. Nothing is placed in the child's mouth during a seizure because this could injure the child's mouth, gums, or teeth. The nurse should stay with the child to reduce the risk of injury and allow for observation and timing of the seizure.

Carbidopa-levodopa (Sinemet) is prescribed for a client with Parkinson's disease. The nurse monitors the client for side/adverse effects to the medication. Which finding indicates that the client is experiencing an adverse effect? 1. Pruritus 2. Tachycardia 3. Hypertension 4. Impaired voluntary movements

Rationale: Dyskinesia and impaired voluntary movement may occur with high levodopa dosages. Nausea, anorexia, dizziness, orthostatic hypotension, bradycardia, and akinesia are frequent side effects of the medication.

The nurse is caring for a client with an intracranial aneurysm who was previously alert. Which sign is an early indication that the level of consciousness (LOC) is deteriorating? Select all that apply. 1. Mild drowsiness 2. Drooping eyelids 3. Ptosis of the left eyelid 4. Slight slurring of speech 5. Less frequent spontaneous speech

Rationale: Early changes in LOC relate to orientation, alertness and verbal responsiveness. Mild drowsiness, slight slurring of speech, and less frequent spontaneous speech are early signs of decreasing LOC. Ptosis (drooping) of the eyelid is caused by pressure on and dysfunction of cranial nerve III. Once ptosis occurs, it is ongoing; it does not relate to LOC.

The nurse is caring for a client after a craniotomy and monitors the client for signs of increased intracranial pressure (ICP). Which finding, if noted in the client, would indicate an early sign of increased ICP? 1. Confusion 2. Bradycardia 3. Sluggish pupils 4. A widened pulse pressure

Rationale: Early manifestations of increased ICP are subtle and often may be transient, lasting for only a few minutes in some cases. These early clinical manifestations include episodes of confusion, drowsiness, and slight pupillary and breathing changes. Later manifestations include a further decrease in the level of consciousness, a widened pulse pressure, and bradycardia. Cheyne-Stokes respiratory pattern, or a hyperventilation respiratory pattern; pupillary sluggishness and dilatation appear in the late stages.

The nurse is reviewing the record of a client with a suspected diagnosis of Huntington's disease. The nurse should expect to note documentation of which early symptom of this disease? 1. Aphasia 2. Agnosia 3. Difficulty with swallowing 4. Balance and coordination problems

Rationale: Early symptoms of Huntington's disease include restlessness, forgetfulness, clumsiness, falls, balance and coordination problems, altered speech, and altered handwriting. Difficulty with swallowing occurs in the later stages. Aphasia and agnosia do not occur.

The nurse is assisting in the care of a client who is being evaluated for possible myasthenia gravis. The health care provider gives a test dose of edrophonium (Enlon). Evaluation of the results indicates that the test is positive. Which would be the expected response noted by the nurse? 1. Joint pain for the next 15 minutes 2. An immediate increase in blood pressure 3. An increase in muscle strength within 1 to 3 minutes 4. Feelings of faintness or dizziness for 5 to 10 minutes

Rationale: Edrophonium (Enlon) is a short-acting acetylcholinesterase inhibitor used to diagnose myasthenia gravis. An increase in muscle strength should be seen in 1 to 3 minutes following the test dose if the client does have the disease. If no response occurs, another dose is given over the next 2 minutes and muscle strength is tested again. If no increase in muscle strength occurs with this higher dose, the muscle weakness is not caused by myasthenia gravis. Clients who receive injections of this medication commonly demonstrate a drop of blood pressure, feel faint and dizzy, and are flushed.

The nurse is caring for a client with myasthenia gravis who has received edrophonium (Enlon) by the intravenous route to test for myasthenic crisis. The client asks the nurse how long the improvement in muscle strength will last. The nurse's response is based on the understanding that the effects have a duration of approximately how long? 1. 5 minutes 2. 10 minutes 3. 30 minutes 4. 60 minutes

Rationale: Edrophonium commonly is given to test for myasthenic crisis. If the client is in myasthenic crisis, muscle strength improves after administration of the medication, and the improvement lasts for about 30 minutes. Options 1, 2, and 4 are incorrect.

A client is suspected of having myasthenia gravis. Edrophonium (Enlon) is administered intravenously to determine the diagnosis. Which indicates that the client may have myasthenia gravis? 1. Joint pain following administration of the medication 2. Feelings of faintness, dizziness, hypotension, and signs of flushing in the client 3. A decrease in muscle strength within 30 to 60 seconds following administration of the medication 4. An increase in muscle strength within 30 to 60 seconds following administration of the medication

Rationale: Edrophonium is a short-acting acetylcholinesterase inhibitor used as a diagnostic agent. When a client with suspected myasthenia gravis the health care provider will administer a edrophonium test. When a dose is administered intravenously, an increase in muscle strength should be seen in 30 to 60 seconds. If no response occurs, another dose of edrophonium is given over the next 2 minutes, and muscle strength is tested again. If no increase in muscle strength occurs with this higher dose, the muscle weakness is not caused by myasthenia gravis. Clients receiving injections of this medication commonly demonstrate a drop in blood pressure, feel faint and dizzy, and are flushed.

The health care provider is preparing to administer edrophonium (Enlon) to the client with myasthenia gravis. In planning care, the nurse understands which about the administration of edrophonium? Select all that apply. 1. Edrophonium is a long-acting cholinesterase inhibitor. 2. Atropine is used to reverse the effects of edrophonium. 3. If symptoms worsen following administration of edrophonium, the crisis is cholinergic. 4. Edrophonium is used to distinguish between a myasthenic crisis and a cholinergic crisis. 5.An improvement in symptoms following administration of edrophonium indicates worsening of myasthenia gravis.

Rationale: Edrophonium is an ultra-short-acting cholinesterase inhibitor that can be used to distinguish between a cholinergic and a myasthenic crisis. To distinguish between overtreatment (cholinergic crisis) and undertreatment (myasthenic crisis), edrophonium, an ultra-short-acting reversible cholinesterase inhibitor is administered; this is often referred to as a Tensilon test. Overtreatment of myasthenia gravis with reversible cholinesterase inhibitors results in a cholinergic crisis. Undertreatment, or disease worsening, can result in a myasthenic crisis. Both cholinergic and myasthenic crises result in increased muscle weakness or paralysis. If symptoms improve after the administration of edrophonium, the crisis is myasthenic; if symptoms worsen, the crisis is cholinergic. Atropine must be readily available so that edrophonium can be reversed if the symptoms worsen.

The nurse is caring for an infant with spina bifida (myelomeningocele type) who had the sac on the back containing cerebrospinal fluid, the meninges, and the nerves (gibbus) surgically removed. The nursing plan of care for the postoperative period should include which nursing action to maintain the infant's safety? 1. Covering the back dressing with a binder 2. Placing the infant in a head-down position 3. Strapping the infant in a baby seat sitting up 4. Elevating the head with the infant in the prone position

Rationale: Elevating the head will decrease the chance that cerebrospinal fluid will accumulate in the cranial cavity. The infant needs to be prone or side-lying to decrease the pressure on the surgical site on the back. Binders and a baby seat should not be used because of the pressure they would exert on the surgical site.

The nurse has a prescription to begin aneurysm precautions for a client with a subarachnoid hemorrhage secondary to aneurysm rupture. The nurse would plan to incorporate which intervention in controlling the environment for this client? 1. Keep the window blinds open. 2. Turn on a small spotlight above the client's head. 3. Make sure the door to the room is open at all times. 4. Prohibit or limit the use of a radio or television and reading.

Rationale: Environmental stimuli are kept to a minimum with subarachnoid precautions to prevent or minimize increases in intracranial pressure. For this reason, lighting is reduced by closing window blinds and keeping the door to the client's room shut. Overhead lighting also is avoided for the same reason. The nurse prohibits television, radio, and reading unless this is so stressful for the client that it would be counterproductive. In that instance, minimal amounts of stimuli by these means are allowed with approval of the health care provider.

The nurse is admitting a client to the hospital emergency department from a nursing home. The client is unconscious with an apparent frontal head injury. A medical diagnosis of epidural hematoma is suspected. Which question is of the highest priority for the emergency department nurse to ask of the transferring nurse at the nursing home? 1. "When did the injury occur?" 2. "Was the client awake and talking right after the injury?" 3. "What medications has the client received since the fall?" 4. "What was the client's level of consciousness before the injury?"

Rationale: Epidural hematomas frequently are characterized by a "lucid interval" that lasts for minutes to hours, during which the client is awake and talking. After this lucid interval, signs and symptoms progress rapidly, with potentially catastrophic intracranial pressure increase. Epidural hematomas are medical emergencies. It is important for the nurse to assist in the differentiation between epidural hematoma and other types of head injuries.

A client with vascular headaches is taking ergotamine (Cafergot). The home health nurse should periodically assess him or her for which finding? 1. Hypotension 2. Constipation 3. Dependent edema 4. Cool, numb fingers and toes

Rationale: Ergotamine produces vasoconstriction by stimulating α-adrenergic receptors, which suppresses vascular headaches when the medication is given in the therapeutic dose range. The nurse periodically assesses for hypertension; cool, numb fingers and toes; muscle pain; and nausea and vomiting. This medication does not cause hypotension, constipation, or dependent edema.

The nurse notes that a client taking ergotamine tartrate (Cafergot) is having the intended effects of therapy if the client states relief from which symptom? 1. Cough 2. Diarrhea 3. Backache 4. Headaches

Rationale: Ergotamine tartrate is used to stop an ongoing migraine attack; it also is used to treat cluster headaches. The other options are unrelated to the use of this medication.

The nurse has given the client with Bell's palsy instructions on preserving muscle tone in the face and preventing denervation. The nurse determines that the client needs additional teaching if the client makes which statements? 1. "I will perform facial exercises." 2. "I will expose my face to cold to decrease the pain." 3. "I will massage my face with a gentle upward motion." 4. "I will wrinkle my forehead, blow out my cheeks, and whistle frequently."

Rationale: Exposure to cold or drafts is avoided in Bell's palsy because it can cause discomfort. Prevention of muscle atrophy with Bell's palsy is accomplished with facial massage, facial exercises, and electrical nerves stimulation. Local application of heat to the face may improve blood flow and provide comfort.

The nurse is performing a neurological assessment on a client and is assessing the function of cranial nerves III, IV, and VI. Assessment of which aspect of function by the nurse will yield the best information about these cranial nerves? 1. Eye movements 2. Response to verbal stimuli 3. Affect, feelings, or emotions 4. Insight, judgment, and planning

Rationale: Eye movements are under the control of cranial nerves III, IV, and VI. Level of consciousness (response to verbal stimuli) is controlled by the reticular activating system and both cerebral hemispheres. Feelings are part of the role of the limbic system and involve both hemispheres. Insight, judgment, and planning are part of the function of the frontal lobe in conjunction with association fibers that connect to other areas of the cerebrum.

The nurse has given suggestions to a client with trigeminal neuralgia about strategies to minimize episodes of pain. The nurse determines that the client needs further education if the client makes which statement? 1. "I will wash my face with cotton pads." 2. "I'll have to start chewing on my unaffected side." 3. "I'll try to eat my food either very warm or very cold." 4. "I should rinse my mouth if toothbrushing is painful."

Rationale: Facial pain can be minimized by using cotton pads to wash the face and using room temperature water. The client should chew on the unaffected side of the mouth, eat a soft diet, and take in foods and beverages at room temperature. If toothbrushing triggers pain, an oral rinse after meals may be helpful instead.

The nurse is trying to help the family of an unconscious client cope with the situation. Which intervention should the nurse plan to incorporate into the care routine for the client and family? 1. Discouraging the family from touching the client 2. Explaining equipment and procedures on an ongoing basis 3. Ensuring adherence to visiting hours to ensure the client's rest 4. Encouraging the family not to "give in" to their feelings of grief

Rationale: Families often need assistance to cope with the illness of a loved one. The nurse should explain all equipment, treatments, and procedures and should supplement or reinforce information given by the health care provider. Family members should be encouraged to touch and speak to the client and to become involved in the client's care to the extent they are comfortable. The nurse should allow the family to stay with the client to the extent possible and should encourage them to eat and sleep adequately to maintain strength. The nurse can help family members of an unconscious client by assisting them to work through their feelings of grief.

A client who is taking phenytoin (Dilantin) for a seizure disorder is being admitted to the hospital because of an increase in seizure activity. The client reports severe vomiting for the last 24 hours and inability to take phenytoin during that time. The nurse anticipates that the health care provider will most likely prescribe which medication? 1. Phenobarbital (Luminal) 2. Clonazepam (Klonopin) 3. Valproic acid (Depakene) 4. Fosphenytoin sodium (Cerebyx)

Rationale: Fosphenytoin sodium is used for short-term parenteral (intravenous) infusion. A client who is not tolerating medications orally and has a seizure disorder would need an anticonvulsant administered by the parenteral route. Phenobarbital is an antiseizure medication that is given orally or parenterally. However, the medication of choice in this case would be fosphenytoin since its use if for short-term. Valproic acid and clonazepam usually are administered orally.

The nurse is assessing a client with fragile X syndrome. The nurse anticipates to note which physical assessment finding? 1. Low, straight palate 2. Short, narrow protruding ears 3. Long, narrow face with a prominent jaw 4. Short, rounded face with an indiscernible jaw

Rationale: Fragile X syndrome is a genetic condition that causes developmental problems including learning disabilities and cognitive impairment. Physical assessment findings of fragile X syndrome include long, wide, and/or protruding ears; a long, narrow face with a prominent jaw; large protruding ears; and large testes. Therefore, options 1, 2, and 4 are incorrect.

The nurse is preparing for the admission to the unit of a client with a diagnosis of seizures and is preparing to institute full seizure precautions. Which item is contraindicated for use if a seizure occurs? 1. Oxygen source 2. Suction machine 3. Padded tongue blade 4. Padding for the side rails

Rationale: Full seizure precautions include bed rest with padded side rails in a raised position, a suction machine at the bedside, having diazepam (Valium) or lorazepam (Ativan) available, and providing an oxygen source. Objects such as tongue blades are contraindicated and should never be placed in the client's mouth during a seizure.

A client is scheduled to begin medication therapy with valproic acid (Depakene). The nurse looks for the results of which laboratory test(s) before administering the first dose? 1. Liver function tests 2. Renal function tests 3. Pulmonary function test 4. Pancreatic enzyme studies

Rationale: Gastrointestinal effects from valproic acid (Depakene) are common and typically mild, but hepatotoxicity, although rare, is serious. To minimize the risk of fatal liver injury, liver function is evaluated before initiation of treatment and periodically thereafter. The other options are unrelated to the use of this medication.

The nurse is preparing a plan of care for a client with a brain attack (stroke) who has global aphasia. The nurse should incorporate communication strategies into the plan of care because the client's speech will be characteristic of which finding? 1. Intact 2. Rambling 3. Characterized by literal paraphasia 4. Associated with poor comprehension

Rationale: Global aphasia is a condition in which the affected person has few language skills as a result of extensive damage to the left hemisphere. The speech is nonfluent and is associated with poor comprehension and limited ability to name objects or repeat words. The client with conduction aphasia has difficulty repeating words spoken by another, and speech is characterized by literal paraphasia with intact comprehension. The client with Wernicke's aphasia may exhibit a rambling type of speech.

The client is admitted to the hospital with a diagnosis of Guillain-Barré syndrome. Which past medical history finding makes the client most at risk for this disease? 1. Meningitis or encephalitis during the last 5 years 2. Seizures or trauma to the brain within the last year 3. Back injury or trauma to the spinal cord during the last 2 years 4. Respiratory or gastrointestinal infection during the previous month

Rationale: Guillain-Barré syndrome is a clinical syndrome of unknown origin that involves cranial and peripheral nerves. Many clients report a history of respiratory or gastrointestinal infection in the 1 to 4 weeks before the onset of neurological deficits. On occasion, the syndrome can be triggered by vaccination or surgery.

A mother arrives at an emergency department with her 5-year-old child and states that the child fell off a bunk bed. A head injury is suspected, and the nurse checks the child's airway status and assesses the child for early and late signs of increased intracranial pressure (ICP). Which is a late sign of increased ICP? 1. Nausea 2. Irritability 3. Headache 4. Bradycardia

Rationale: Head injury is the pathological result of any mechanical force to the skull, scalp, meninges, or brain. A head injury can cause bleeding in the brain and result in increased intracranial pressure (ICP). In a child, early signs include a slight change in level of consciousness, headache, nausea, vomiting, visual disturbances (diplopia), seizures. Late signs of increased ICP include a significant decrease in level of consciousness, bradycardia, decreased motor and sensory responses, alterations in pupil size and reactivity, posturing, Cheyne-Stokes respirations, and coma.

The nurse is monitoring a client who has returned to the nursing unit after a myelogram. Which client complaint would indicate the need to notify the health care provider (HCP)? 1. Backache 2. Headache 3. Neck stiffness 4. Feelings of fatigue

Rationale: Headache is relatively common after the procedure, but neck stiffness, especially on flexion, and pain should be reported because they signal meningeal irritation. The client also is monitored for evidence of allergic reactions to the dye such as confusion, dizziness, tremors, and hallucinations. Feelings of fatigue may be normal, and back discomfort may be owing to the positions required for the procedure.

A client who had a brain attack (stroke) has right-sided hemianopsia. What should the nurse plan to do to help the client adapt to this problem? 1. Teach the client to scan the environment. 2. Place all objects within the left visual field. 3. Place all objects within the right visual field. 4. Ensure that the family brings the client's eyeglasses to hospital.

Rationale: Hemianopsia is blindness in half the visual field. The client with hemianopsia is taught to scan the environment. This allows the client to take in the entirety of the visual field, which is necessary for proper functioning within the environment and helps to prevent injury to the client. Options 2 and 3 will not help the client adapt to this visual impairment. Eyeglasses are useful if the client already wears them, but they will not correct this visual-field deficit.

The nurse is assigned to care for a client with complete right-sided hemiparesis. Which characteristics are associated with this condition? Select all that apply. 1. The client is aphasic. 2. The client has weakness in the face and tongue. 3. The client has weakness on the right side of the body. 4. The client has complete bilateral paralysis of the arms and legs. 5. The client has lost the ability to move the right arm but is able to walk independently. 6. The client has lost the ability to ambulate independently but is able to feed and bathe himself or herself without assistance.

Rationale: Hemiparesis is a weakness of one side of the body that may occur after a stroke. It involves weakness of the face and tongue, arm, and leg on one side. These clients are also aphasic: unable to discriminate words and letters. They are generally very cautious and get anxious when attempting a new task. Complete bilateral paralysis does not occur in this hemiparesis. The client with right-sided hemiparesis has weakness of the right arm and leg and needs assistance with feeding, bathing, and ambulating.

The nurse is developing a plan of care for a client with a stroke (brain attack) who has right homonymous hemianopsia. Which should the nurse include in the plan of care for the client? 1. Place an eye patch on the left eye. 2. Place personal articles on the client's right side. 3. Approach the client from the right field of vision. 4. Instruct the client to turn the head to scan the right visual field.

Rationale: Homonymous hemianopsia is a loss of half of the visual field. The nurse instructs the client to scan the environment and stands within the client's intact field of vision. The nurse should not patch the eye because the client does not have double vision. The client should have objects placed in the intact fields of vision, and the nurse should approach the client from the intact side.

The nurse has instructed the family of a client with stroke (brain attack) who has homonymous hemianopsia about measures to help the client overcome the deficit. Which statement suggests that the family understands the measures to use when caring for the client? 1. "We need to discourage him from wearing eyeglasses." 2. "We need to place objects in his impaired field of vision." 3. "We need to approach him from the impaired field of vision." 4. "We need to remind him to turn his head to scan the lost visual field."

Rationale: Homonymous hemianopsia is loss of half of the visual field. The client with homonymous hemianopsia should have objects placed in the intact field of vision, and the nurse also should approach the client from the intact side. The nurse instructs the client to scan the environment to overcome the visual deficit and does client teaching from within the intact field of vision. The nurse encourages the use of personal eyeglasses, if they are available.

A nurse notes that an infant with the diagnosis of hydrocephalus has a head that is heavier than that of the average infant. The nurse should determine that special safety precautions are needed when moving the infant with hydrocephalus. Which statement should the nurse plan to include in the discharge teaching with the parents to reflect this safety need? 1. "Feed your infant in a side-lying position." 2. "Place a helmet on your infant when in bed." 3. "Hyperextend your infant's head with a rolled blanket under the neck area." 4. "When picking up your infant, support the infant's neck and head with the open palm of your hand."

Rationale: Hydrocephalus is a condition characterized by an enlargement of the cranium because of an abnormal accumulation of cerebrospinal fluid in the cerebral ventricular system. This characteristic causes the increase in the weight of the infant's head. The infant may experience significant head enlargement. Care must be exercised so that the head is well supported when the infant is fed or moved to prevent extra strain on the infant's neck, and measures must be taken to prevent the development of pressure areas. Supporting the infant's head and neck when picking up the infant will prevent the hyperextension of the neck area and the infant from falling backward. The infant should be fed with the head elevated for proper motility of food processing. A helmet could suffocate an unattended infant during rest and sleep times, and hyperextension of the infant's head could put pressure on the neck vertebrae, causing injury.

An infant with a diagnosis of hydrocephalus is scheduled for surgery. Which is the priority nursing intervention in the preoperative period? 1. Test the urine for protein. 2. Reposition the infant frequently. 3. Provide a stimulating environment. 4. Assess blood pressure every 15 minutes.

Rationale: Hydrocephalus occurs as a result of an imbalance of cerebrospinal fluid absorption or production that is caused by malformations, tumors, hemorrhage, infections, or trauma. It results in head enlargement and increased intracranial pressure. In infants with hydrocephalus, the head grows at an abnormal rate, and if the infant is not repositioned frequently, pressure ulcers can occur on the back and side of the head. An egg crate mattress under the head is also a nursing intervention that can help prevent skin breakdown. Proteinuria is not specific to hydrocephalus. Stimulus should be kept at a minimum because of the increase in intracranial pressure. It is not necessary to check the blood pressure every 15 minutes.

A client is being hyperventilated by a mechanical ventilator to decrease the client's intracranial pressure (ICP). On monitoring arterial blood gas results, the nurse should expect values that are within which ranges? 1. Pao2 60 to 100 mm Hg, Paco2 25 to 30 mm Hg 2. Pao2 60 to 100 mm Hg, Paco2 30 to 35 mm Hg 3. Pao2 80 to 100 mm Hg, Paco2 25 to 30 mm Hg 4. Pao2 80 to 100 mm Hg, Paco2 35 to 40 mm Hg

Rationale: Hyperventilation with a Paco2 of 25 to 30 mm Hg causes cerebral vasoconstriction, which decreases intracranial blood volume and ICP. The Pao2 is not allowed to fall below 80 mm Hg, to prevent cerebral vasodilation from hypoxemia. Therefore, the remaining options are incorrect.

The nurse is caring for an unconscious client who is experiencing persistent hyperthermia with no signs of infection. The nurse interprets that the hyperthermia may be related to damage to the client's thermoregulatory center in which structure? 1. Cerebrum 2. Cerebellum 3. Hippocampus 4. Hypothalamus

Rationale: Hypothalamic damage causes persistent hyperthermia, which also may be called central fever. It is characterized by a persistent high fever with no diurnal variation. Another characteristic feature is absence of sweating. Hyperthermia would not result from damage to the cerebrum, cerebellum, or hippocampus.

A client with status epilepticus has been prescribed phenytoin (Dilantin) to be given by the intravenous (IV) route. The nurse administering the medication is careful not to exceed which recommended infusion rate? 1. 50 mg/min 2. 60 mg/min 3. 100 mg/min 4. 750 mg/min

Rationale: IV administration of phenytoin is performed slowly (no faster than 50 mg/min) because rapid administration can cause cardiovascular collapse. It should not be added to any existing IV infusion because this is likely to produce a precipitate in the solution. Solutions are highly alkaline and can cause local venous irritation.

The nurse should place a child who had a medulloblastoma brain tumor (infratentorial) removed in which position postoperatively? 1. Trendelenburg's 2. Flat, on either side 3. With the head of the bed elevated above heart level 4. With the head of the bed elevated in low Fowler's position

Rationale: If an infratentorial tumor has been removed, the child is positioned flat on either side. The pillow is placed behind the child's back for comfort and to maintain the position. The pillow is not placed behind the head because when the pillow is behind the head, proper alignment is not maintained, and this misalignment can impair circulation. The child should never be placed in a Trendelenburg's position (head down) because this position increases intracranial pressure. The head is elevated when the tumor is a supratentorial one.

The nurse is assisting in the care of a client with myasthenia gravis who is receiving pyridostigmine (Mestinon). Which medication should the nurse plan to have readily available should the client develop cholinergic crisis because of excessive medication dosage? 1. Vitamin K 2. Atropine sulfate 3. Protamine sulfate 4. Acetylcysteine (Mucomyst)

Rationale: If the client is in cholinergic crisis, the antidote for the medication would be a medication that is an anticholinergic. Thus, the antidote for cholinergic crisis is atropine sulfate. Vitamin K is the antidote for warfarin (Coumadin). Protamine sulfate is the antidote for heparin, and acetylcysteine (Mucomyst) is the antidote for acetaminophen (Tylenol).

The nurse is providing instructions to the parents of an infant with a ventriculoperitoneal shunt. The nurse should include which instruction? 1. Expect an increased urine output from the shunt. 2. Notify the health care provider if the infant is fussy. 3. Call the health care provider if the infant has a high-pitched cry. 4. Position the infant on the side of the shunt when the infant is put to bed.

Rationale: If the shunt is malfunctioning, the fluid from the ventricle part of the brain will not be diverted to the peritoneal cavity. The cerebrospinal fluid will build up in the cranial area. The result is increased intracranial pressure, which then causes a high-pitched cry in the infant. The infant should not be positioned on the side of the shunt because this will cause pressure on the shunt and skin breakdown. This type of shunt affects the gastrointestinal system, not the genitourinary system, and an increased urinary output is not expected. Being fussy is a concern only if other signs indicative of a complication are occurring.

The nurse is assessing the function of cranial nerve XII in a client who sustained a stroke. To assess function of this nerve, which action should the nurse ask the client to perform? 1. Extend the arms. 2. Extend the tongue. 3. Turn the head toward the nurse's arm. 4. Focus the eyes on the object held by the nurse.

Rationale: Impairment of cranial nerve XII can occur with a stroke. To assess the function of cranial nerve XII (the hypoglossal nerve), the nurse would assess the client's ability to extend the tongue. Options 1, 3, and 4 do not test the function of cranial nerve XII.

A client reports frequent use of acetaminophen (Tylenol) for relief of headaches and other discomforts. The nurse should evaluate which diagnostic data to determine if the client is at risk for toxicity? 1. Chest x-ray 2. The electrocardiogram 3. The liver function studies 4. The upper gastrointestinal x-ray results

Rationale: In adults, overdose of acetaminophen causes liver damage. In addition, clients with liver disorders are at higher risk of experiencing hepatotoxicity with chronic acetaminophen use. Options 1, 2, and 4 are not associated with acetaminophen overdose.

A client with trigeminal neuralgia tells the nurse that acetaminophen (Tylenol) is taken daily for the relief of generalized discomfort. Which laboratory value would indicate toxicity associated with the medication? 1. Sodium level of 140 mEq/L 2. Prothrombin time of 11.8 seconds 3. Direct bilirubin level of 2 mg/dL 4. Platelet count of 400,000 cells/mm3

Rationale: In adults, overdose of acetaminophen causes liver damage. The correct option is an indicator of liver function and is the only option that indicates an abnormal laboratory value. The normal direct bilirubin level is 0 to 0.3 mg/dL. The normal sodium level is 135 to 145 mEq/L. The normal prothrombin time is 9.5 to 11.3 seconds. The normal platelet count is 150,000 to 400,000 cells/mm3.

The nurse is performing the oculocephalic response (doll's-eyes maneuver) test on an unconscious client. The nurse turns the client's head and notes movement of the eyes in the same direction as for the head. How should the nurse document these findings? 1. Normal 2. Abnormal 3. Insignificant 4. Inconclusive

Rationale: In an unconscious client, eye movements are an indication of brainstem activity and are tested by the oculocephalic response. When the doll's-eyes maneuver is intact, the eyes move in the opposite direction when the head is turned. Abnormal responses include movement of the eyes in the same direction as for the head and maintenance of a midline position of the eyes when the head is turned. An abnormal response indicates a disruption in the processing of information through the brainstem.

The nurse is assisting the neurologist in performing an assessment on a client who is unconscious after sustaining a head injury. The nurse understands that the neurologist would avoid performing the oculocephalic response (doll's-eyes maneuver) if which condition is present in the client? 1. Dilated pupils 2. Lumbar trauma 3. A cervical cord injury 4. Altered level of consciousness

Rationale: In an unconscious client, eye movements are an indication of brainstem activity and are tested by the oculocephalic response. When the doll's-eyes maneuver is intact, the eyes move in the opposite direction when the head is turned. Abnormal responses include movement of the eyes in the same direction as that for the head and maintenance of a midline position of the eyes when the head is turned. An abnormal response indicates a disruption in the processing of information through the brainstem. Contraindications to performing this test include cervical-level spinal cord injuries and severely increased intracranial pressure.

The nurse is developing a plan of care for a client with a diagnosis of brain attack (stroke) with anosognosia. To meet the needs of the client with this deficit, the nurse should include activities that will achieve which outcome? 1. Encourage communication. 2. Provide a consistent daily routine. 3. Promote adequate bowel elimination. 4. Increase the client's awareness of the affected side.

Rationale: In anosognosia, the client exhibits neglect of the affected side of the body. The nurse will plan care activities that remind the client to perform actions that require looking at the affected arm or leg, as well as activities that will increase the client's awareness of the affected side. Options 1, 2, and 3 are not associated with this deficit.

The nurse is preparing a plan of care for a client with a diagnosis of brain attack (stroke). On reviewing the client's record, the nurse notes an assessment finding of anosognosia. The nursing care plan should address which manifestation related to this finding? 1. The client will be easily fatigued. 2. The client will have difficulty speaking. 3. The client will have difficulty swallowing. 4. The client will exhibit neglect of the affected side.

Rationale: In anosognosia, the client neglects the affected side of the body. The client either may ignore the presence of the affected side (often creating a safety hazard as a result of potential injuries) or may state that the involved arm or leg belongs to someone else. Options 1, 2, and 3 are not associated with anosognosia.

A nurse is performing an assessment on a client with a head injury and notes that the client is assuming this posture. The nurse contacts the health care provider and reports that the client is exhibiting which posture? Refer to Figure. 1. Opisthotonos 2. Decorticate rigidity 3. Decerebrate rigidity 4. Flaccid quadriplegia

Rationale: In decorticate rigidity, the upper extremities (arms, wrists, and fingers) are flexed with adduction of the arms. The lower extremities are extended with internal rotation and plantar flexion. Decorticate rigidity indicates a hemispheric lesion of the cerebral cortex. Opisthotonos is prolonged arching of the back with the head and heels bent backward. Opisthotonos indicates meningeal irritation. In decerebrate rigidity, the upper extremities are stiffly extended and adducted with internal rotation and pronation of the palms. The lower extremities are stiffly extended with plantar flexion. The teeth are clenched, and the back is hyperextended. Decerebrate rigidity indicates a lesion in the brainstem at the midbrain or upper pons. Flaccid quadriplegia is complete loss of muscle tone and paralysis of all four extremities, indicating a completely nonfunctional brainstem.

A nurse caring for a client following craniotomy who has a supratentorial incision understands that the client should most likely be maintained in which position? 1. Prone position 2. Supine position 3. Semi-Fowler's position 4. Dorsal recumbent position

Rationale: In supratentorial surgery (surgery above the brain's tentorium), the client's head is usually elevated 30 degrees to promote venous outflow through the jugular veins. The client's head or the head of the bed is not lowered in the acute phase of care after supratentorial surgery. An exception to this position is the client who has undergone evacuation of a chronic subdural hematoma, but a health care provider's (HCP) prescription is required for positions other than those involving head elevation. Additionally, the health care provider's (HCP) prescription regarding positioning is always checked and agency procedures are always followed.

The nurse assigned to the care of an unconscious client is making initial daily rounds. On entering the client's room, the nurse observes that the client is lying supine in bed, with the head of the bed elevated approximately 5 degrees. The nasogastric tube feeding is running at 70 mL/hr, as prescribed. The nurse assesses the client and auscultates adventitious breath sounds. Which judgment should the nurse formulate for the client? 1. Impaired nutritional intake 2. Increased risk for aspiration 3. Increased likelihood for injury 4. Susceptibility to fluid volume deficit

Rationale: Increased risk for aspiration is a condition in which an individual is at risk for entry of gastrointestinal (GI) secretions, oropharyngeal secretions, or solids or fluids into tracheobronchial passages. Conditions that place the client at risk for aspiration include reduced level of consciousness, depressed cough and gag reflexes, and feeding via a GI tube. There is no information in the question indicating that option 1, 3, or 4 is a concern.

The nurse is observing a new nursing graduate who is preparing an intermittent intravenous (IV) infusion of phenytoin (Dilantin) for a client with a diagnosis of seizures. Which solution used by the nursing graduate should indicate to the nurse an understanding of proper preparation of this medication? 1. 5% dextrose in water 2. Lactated Ringer's solution 3. 0.9% sodium chloride (normal saline) 4. 5% dextrose and 0.45% sodium chloride

Rationale: Intermittent IV infusion of phenytoin is administered by injection into a large vein, using normal saline solution. Dextrose solutions are avoided because the medication will precipitate in these solutions. Options 1, 2, and 4 identify incorrect solutions for IV administration with this medication.

A nurse reviews the plan of care for a child with Reye's syndrome. Which priority intervention should the nurse include in the plan of care? 1. Monitor for signs of increased intracranial pressure. 2. Immediately check the presence of protein in the urine. 3. Reassure the parents hyperglycemia is a common symptom. 4. Teach the parents signs and symptoms of a bacterial infection.

Rationale: Intracranial pressure and encephalopathy are major symptoms of Reye's syndrome. Protein is not present in the urine. Reye's syndrome is related to a history of viral infections, and hypoglycemia is a symptom of this disease.

The nurse is preparing an intravenous infusion of phenytoin (Dilantin) as prescribed by the health care provider for the client with seizures. Which solution should the nurse plan to use to dilute this medication? 1. Dextrose 5% 2. Normal saline solution 3. Lactated Ringer's solution 4. Dextrose 5% and half-normal saline (0.45%)

Rationale: Intravenous infusion of phenytoin should be administered by injection into a large vein. The medication may be diluted in normal saline solution; however, dextrose solution should be avoided because of medication precipitation. The medication is administered as intermittent doses. Continuous intravenous infusions should not be used. Infusion rates of more than 50 mg/minute may cause hypotension or cardiac dysrhythmias, especially in older and debilitated clients.

The nurse is caring for a client with an exacerbation of multiple sclerosis. Which medication(s) will the nurse expect to be prescribed to hasten recovery from the exacerbation? 1. Methylprednisolone (Solu-Medrol) intravenously 2. Carbamazepine (Tegretol) and phenytoin (Dilantin) by mouth 3. Phenytoin (Dilantin) intravenously, then tapered to oral route 4. Lioresal (Baclofen) by mouth and diazepam (Valium) intravenously

Rationale: Intravenous methylprednisolone or adrenocorticotropic hormone may be prescribed to accelerate recovery from an exacerbation of multiple sclerosis. Carbamazepine may be prescribed for trigeminal neuralgia, and phenytoin may be prescribed to control seizures. Lioresal and diazepam are used to treat muscle spasticity.

A client has clear fluid leaking from the nose following a basilar skull fracture. Which finding would alert the nurse that cerebrospinal fluid is present? 1. Fluid is clear and tests negative for glucose. 2. Fluid is grossly bloody in appearance and has a pH of 6. 3. Fluid clumps together on the dressing and has a pH of 7. 4. Fluid separates into concentric rings and tests positive for glucose.

Rationale: Leakage of cerebrospinal fluid (CSF) from the ears or nose may accompany basilar skull fracture. CSF can be distinguished from other body fluids because the drainage will separate into bloody and yellow concentric rings on dressing material, called a halo sign. The fluid also tests positive for glucose.

The nurse in the neurological unit is caring for a client with a supratentorial lesion. The nurse assesses which measurement as the most critical index of central nervous system (CNS) dysfunction? 1. Temperature 2. Blood pressure 3. Ability to speak 4. Level of consciousness

Rationale: Level of consciousness is the most critical index of CNS dysfunction. Changes in level of consciousness can indicate clinical improvement or deterioration. Although blood pressure, temperature, and ability to speak may be components of the assessment, the client's level of consciousness is the most critical index of CNS dysfunction.

The nurse is developing a plan of care for a client with dysphagia following a stroke (brain attack). Which should the nurse include in the plan? Select all that apply. 1. Thicken liquids. 2. Assist the client with eating. 3. Assess for the presence of a swallow reflex. 4. Place the food on the affected side of the mouth. 5. Provide ample time for the client to chew and swallow.

Rationale: Liquids are thickened to prevent aspiration. The nurse should assist the client with eating and place food on the unaffected side of the mouth. The nurse should assess for gag and swallowing reflexes before the client with dysphagia is started on a diet. The client should be allowed ample time to chew and swallow to prevent choking.

The client with a head injury is experiencing signs of increased intracranial pressure (ICP), and mannitol (Osmitrol) is prescribed. The nurse administering this medication expects which as an intended effects of this medication? Select all that apply. 1. Increased diuresis 2. Reduced intracranial pressure 3. Increased osmotic pressure of glomerular filtrate 4. Reduced tubular reabsorption of water and solutes 5. Reabsorption of sodium and water in the loop of Henle

Rationale: Mannitol is an osmotic diuretic that induces diuresis by raising the osmotic pressure of glomerular filtrate, thereby inhibiting tubular reabsorption of water and solutes. It is used to reduce intracranial pressure in the client with head trauma. The incorrect option would cause fluid retention through reabsorption, thereby increasing the intracranial pressure.

The nurse is caring for a client who is at risk for increased intracranial pressure (ICP) after a stroke. Which activities performed by the nurse will assist with preventing increases in ICP? Select all that apply. 1. Clustering nursing activities 2. Hyperoxygenating before suctioning 3. Maintaining 20 degree flexion of the knees 4. Maintaining the head and neck in midline position 5. Maintaining the head of the bed (HOB) at 30 degrees elevation

Rationale: Measures aimed at preventing increased ICP in the post-stroke client include hyperoxgenating before suctioning to avoid transient hypoxemia and resultant ICP elevation from dilation of cerebral arteries; maintaining the head in a midline, neutral position to help promote venous drainage from the brain; and keeping the HOB elevated to between 25 and 30 degrees to prevent a decreased blood flow to the brain. Clustering activities can be stressful for the client and increase ICP. Maintaining 20 degree flexion of the knees increases intra-abdominal pressure and consequently ICP.

The nurse notes documentation that a child with meningitis is exhibiting a positive Kernig's sign. Which observation is characteristic of this sign? 1. The child complains of muscle and joint pain. 2. Petechial and purpuric rashes are noted on the child's trunk. 3. Neck flexion causes adduction and flexion movements of the lower extremities. 4. The child is not able to extend the leg when the thigh is flexed anteriorly at the hip.

Rationale: Meningitis is an infectious process of the central nervous system caused by bacteria and viruses. The inability to extend the leg when the thigh is flexed anteriorly at the hip is a positive Kernig's sign, noted in meningitis. Muscle and joint pain is characteristic of meningococcal infection and H. influenzae infection. A petechial or purpuric rash is characteristic of meningococcal infection. A positive Brudzinski's sign is noted when neck flexion causes adduction and flexion movements of the lower extremities in children and adolescents. This is also a characteristic of meningitis.

The nurse is planning care for a child with acute bacterial meningitis. Based on the mode of transmission of this infection, which precautionary intervention should be included in the plan of care? 1. Maintain enteric precautions. 2. Maintain neutropenic precautions. 3. No precautions are required as long as antibiotics have been started. 4. Maintain respiratory isolation precautions for at least 24 hours after the initiation of antibiotics.

Rationale: Meningitis is an infectious process of the central nervous system caused by bacteria and viruses; it may be acquired as a primary disease or as a result of complications of neurosurgery, trauma, infection of the sinus or ears, or systemic infections. A major priority of nursing care for a child suspected to have meningitis is to administer the antibiotic as soon as it is prescribed. The child also is placed on respiratory isolation precautions for at least 24 hours while culture results are obtained and the antibiotic is having an effect. Enteric precautions and neutropenic precautions are not associated with the mode of transmission of meningitis. Enteric precautions are instituted when the mode of transmission is through the gastrointestinal tract. Neutropenic precautions are instituted when a child has a low neutrophil count.

A lumbar puncture is performed on a child suspected to have bacterial meningitis, and cerebrospinal fluid (CSF) is obtained for analysis. The nurse reviews the results of the CSF analysis and determines that which results would verify the diagnosis? 1. Clear CSF, decreased pressure, and elevated protein level 2. Clear CSF, elevated protein, and decreased glucose levels 3. Cloudy CSF, elevated protein, and decreased glucose levels 4. Cloudy CSF, decreased protein, and decreased glucose levels

Rationale: Meningitis is an infectious process of the central nervous system caused by bacteria and viruses; it may be acquired as a primary disease or as a result of complications of neurosurgery, trauma, infection of the sinus or ears, or systemic infections. Meningitis is diagnosed by testing cerebrospinal fluid obtained by lumbar puncture. In the case of bacterial meningitis, findings usually include an elevated pressure; turbid or cloudy cerebrospinal fluid; and elevated leukocyte, elevated protein, and decreased glucose levels.

Meperidine hydrochloride (Demerol) has been prescribed for a client to treat pain. Which are side/adverse effects of this medication? Select all that apply. 1. Diarrhea 2. Tremors 3. Drowsiness 4. Hypotension 5. Urinary frequency 6. Increased respiratory rate

Rationale: Meperidine hydrochloride is an opioid analgesic. Side/adverse effects include respiratory depression, drowsiness, hypotension, constipation, urinary retention, nausea, vomiting, and tremors.

The client arrives at the emergency department complaining of back spasms. The client states, "I have been taking two to three aspirin every 4 hours for the last week, and it hasn't helped my back." Since aspirin intoxication is suspected, the nurse should assess the client for which manifestation? 1. Tinnitus 2. Diarrhea 3. Constipation 4. Photosensitivity

Rationale: Mild intoxication with acetylsalicylic acid (aspirin) is called salicylism and is experienced commonly when the daily dosage is higher than 4 g. Tinnitus (ringing in the ears) is the most frequent effect noted with intoxication. Hyperventilation may occur because salicylate stimulates the respiratory center. Fever may result, because salicylate interferes with the metabolic pathways coupling oxygen consumption and heat production. Options 2, 3, and 4 are not associated specifically with toxicity.

The nurse is caring for a client receiving morphine sulfate for pain. Because this medication has been prescribed for this client, which nursing action should be included in the plan of care? 1. Encourage fluids. 2. Monitor the client's temperature. 3. Maintain the client in a supine position. 4. Encourage the client to cough and deep breathe.

Rationale: Morphine sulfate suppresses the cough reflex. Clients need to be encouraged to cough and deep breathe to prevent pneumonia. The remaining options are not associated specifically with the use of this medication.

The nurse has instructed a client with myasthenia gravis about strategies for self-management at home. The nurse determines a need for more information if the client makes which statement? 1. "Here's the Medic-Alert bracelet I obtained." 2. "I should take my medications an hour before mealtime." 3. "Going to the beach will be a nice, relaxing form of activity." 4. "I've made arrangements to get a portable resuscitation bag and home suction equipment.

Rationale: Most ongoing treatment for myasthenia gravis is done in outpatient settings, and the client must be aware of the lifestyle changes needed to maintain independence. The client should carry medical identification about the presence of the condition. Taking medications an hour before mealtime gives greater muscle strength for chewing and is indicated. The client should have portable suction equipment and a portable resuscitation bag available in case of respiratory distress. The client should avoid situations and other factors, including stress, infection, heat, surgery, and alcohol, that could worsen the symptoms.

A client arrives in the hospital emergency department with a closed head injury to the right side of the head caused by an assault with a baseball bat. The nurse assesses the client neurologically, looking primarily for motor response deficits that involve which area? 1. The left side of the body 2. The right side of the body 3. Both sides of the body equally 4. Cranial nerves only, such as speech and pupillary response

Rationale: Motor responses such as weakness and decreased movement will be seen on the side of the body that is opposite an area of head injury. Contralateral deficits result from compression of the cortex of the brain or the pyramidal tracts. Depending on the severity of the injury, the client may have a variety of neurological deficits.

The nurse is assessing the motor function of an unconscious client. The nurse should plan to use which technique to test the client's peripheral response to pain? 1. Sternal rub 2. Nail bed pressure 3. Pressure on the orbital rim 4. Squeezing of the sternocleidomastoid muscle

Rationale: Motor testing in the unconscious client can be done only by testing response to painful stimuli. Nail bed pressure tests a basic peripheral response. Cerebral responses to pain are tested using a sternal rub, placing upward pressure on the orbital rim, or squeezing the clavicle or sternocleidomastoid muscle.

A client with multiple sclerosis is experiencing muscle weakness, spasticity, and an ataxic gait. On the basis of this information, the nurse should include which client problem in the plan of care? 1. Inability to care for self 2. Interruption in skin integrity 3. Interruption in physical mobility 4. Inability to perform daily activities

Rationale: Multiple sclerosis is a chronic, nonprogressive, noncontagious degenerative disease of the central nervous system characterized by demyelinization of the neurons. Interruption in physical mobility is most appropriate for the client with multiple sclerosis experiencing muscle weakness, spasticity, and ataxic gait. Options 1, 2, and 4 are not related to the data in the question.

A client with a history of myasthenic gravis presents at a clinic with bilateral ptosis and is drooling, and myasthenia crisis is suspected. The nurse assesses the client for which precipitating factor? 1. Getting too little exercise 2. Taking excess medication 3. Omitting doses of medication 4. Increasing intake of fatty foods

Rationale: Myasthenic crisis often is caused by undermedication and responds to the administration of cholinergic medications. Cholinergic crisis (the opposite problem) is caused by excess medication and responds to withholding of medications. Too little exercise and excessive fatty food intake are incorrect. Overexertion and overeating possibly could trigger myasthenic crisis.

A nurse is monitoring a 7-year-old child who sustained a head injury in a motor vehicle crash for signs of increased intracranial pressure (ICP). The nurse should assess the child frequently for which early sign of increased ICP? 1. Nausea 2. Papilledema 3. Decerebrate posturing 4. Alterations in pupil size

Rationale: Nausea is an early sign of increased ICP. Late signs of increased ICP include a significant decrease in level of consciousness, Cushing's triad (increased systolic blood pressure and widened pulse pressure, bradycardia, and irregular respirations), and fixed and dilated pupils. Other late signs include decreased motor response to command, decreased sensory response to painful stimuli, posturing, Cheyne-Stokes respirations, and papilledema.

The nurse is caring for a client who begins to experience seizure activity while in bed. Which action by the nurse is contraindicated? 1. Loosening restrictive clothing 2. Restraining the client's limbs 3. Removing the pillow and raising padded side rails 4. Positioning the client to the side, if possible, with the head flexed forward

Rationale: Nursing actions during a seizure include providing for privacy, loosening restrictive clothing, removing the pillow and raising padded side rails in the bed, and placing the client on one side with the head flexed forward, if possible, to allow the tongue to fall forward and facilitate drainage. The limbs are never restrained because the strong muscle contractions could cause the client harm. If the client is not in bed when seizure activity begins, the nurse lowers the client to the floor, if possible, protects the head from injury, and moves furniture that may injure the client.

The nurse is providing care to a client with increased intracranial pressure (ICP). Which approach is beneficial in controlling the client's ICP from an environmental viewpoint? 1. Reduce environmental noise. 2. Allow visitors as desired by the client and family. 3. Cluster nursing activities to reduce the number of interruptions. 4. Awaken the client every 2 to 3 hours to monitor mental status.

Rationale: Nursing interventions to control the ICP include maintaining a calm, quiet, and restful environment. Environmental noise should be kept at a minimum. Visiting should be monitored to avoid emotional stress and interruption of sleep. Interventions should be spaced out over the shift to minimize the risk of a sustained rise in ICP.

A client with multiple sclerosis tells a home health care nurse that she is having increasing difficulty in transferring from the bed to a chair. What is the initial nursing action? 1. Observe the client demonstrating the transfer technique. 2. Start a restorative nursing program before an injury occurs. 3. Seize the opportunity to discuss potential nursing home placement. 4. Determine the number of falls that the client has had in recent weeks.

Rationale: Observation of the client's transfer technique is the initial intervention. Starting a restorative program is important but not unless an assessment has been completed first. Discussing nursing home placement would be inappropriate in view of the information provided in the question. Determining the number of falls is another important intervention, but observing the transfer technique should be done first.

The nurse is reviewing a chart for a child with a head injury. The nurse notes that the level of consciousness has been documented as obtunded. Which finding should the nurse expect to note on assessment of the child? 1. Not easily arousable and limited interaction 2. Loss of the ability to think clearly and rapidly 3. Loss of the ability to recognize place or person 4. Awake, alert, interacting with the environment

Rationale: Obtunded indicates that the child sleeps unless aroused and once aroused has limited interaction with the environment. Confusion indicates that the ability to think clearly and rapidly is lost. Disorientation indicates that the ability to recognize place or person is lost. Full consciousness indicates that the child is alert, awake, orientated, and interacts with the environment.

Cerebral palsy (CP) is a term applied to a disorder that impairs movement and posture. The effects on perception, language, and intellect are determined by the type that is diagnosed. What are the potential warning signs of CP? Select all that apply. 1. The infant's arms or legs are stiff or rigid. 2. A high risk factor for CP is very low birth weight. 3. By 8 months of age, the infant can sit without support. 4. The infant has strong head control but a limp body posture. 5. The infant has feeding difficulties, such as poor sucking and swallowing. 6. If the infant is able to crawl, only one side is used to propel himself or herself.

Rationale: Options 1, 2, 5, and 6 are potential warning signs of CP. By 8 months of age, if the infant cannot sit up without support, this would be considered a potential warning sign, because this developmental task should be completed by this time. The infant with a potential diagnosis of CP has poor head control by 3 months of age, when head control should be strong.

The home health nurse is visiting a client with a diagnosis of multiple sclerosis. The client has been taking oxybutynin (Ditropan XL). The nurse evaluates the effectiveness of the medication by asking the client which assessment question? 1. "Are you consistently fatigued?" 2. "Are you having muscle spasms?" 3. "Are you getting up at night to urinate?" 4. "Are you having normal bowel movements?"

Rationale: Oxybutynin is an antispasmodic used to relieve symptoms of urinary urgency, frequency, nocturia, and incontinence in clients with uninhibited or reflex neurogenic bladder. Expected effects include improved urinary control and decreased urinary frequency, incontinence, and nocturia. Options 1, 2, and 4 are unrelated to the use of this medication.

A nurse is performing an admission assessment on a child with a seizure disorder. The nurse is interviewing the child's parents to determine their adjustment to caring for their child, who has a chronic illness. Which statement, if made by the parents, would indicate a need for further teaching? 1. "Our child sleeps in our bedroom at night." 2. "We worry about injuries when our child has a seizure." 3. "Our child is involved in a swim program with neighbors and friends." 4. "Our babysitter just completed cardiopulmonary resuscitation [CPR] training."

Rationale: Parents are especially concerned about seizures that might go undetected during the night. The nurse needs to decrease parental overprotection and should suggest the use of a baby monitor at night. Options 3 and 4, involvement in a swim program and knowing CPR identify parental understanding of the disorder. Worrying about injuries when a child has a seizure is a common concern. The parents need to be reminded that as the child grows, they cannot always observe their child, but their knowledge of seizure activity and care is appropriate to minimize complications.

The home health nurse visits a client who is taking phenytoin (Dilantin) for control of seizures. During the assessment, the nurse notes that the client is taking birth control pills. Which information should the nurse include in the teaching plan? 1. Pregnancy should be avoided while taking phenytoin. 2. The client may stop the medication if it is causing severe gastrointestinal effects. 3. There is the potential of decreased effectiveness of birth control pills while taking phenytoin. 4. There is the increased risk of thrombophlebitis while taking phenytoin and birth control pills together.

Rationale: Phenytoin enhances the rate of estrogen metabolism, which can decrease the effectiveness of some birth control pills. Options 1, 2, and 4 are inappropriate instructions.

A nurse is speaking with a client taking phenytoin (Dilantin) for seizure control. The client states that she has started using birth control pills to prevent pregnancy. Which would be an important point for the nurse to emphasize to the client? 1. Oral contraceptives decrease the effectiveness of phenytoin. 2. Severe gastrointestinal side effects can occur when phenytoin and oral contraceptives are taken together. 3. There is an increased risk of thrombophlebitis when phenytoin and oral contraceptives are taken at the same time. 4. Phenytoin may decrease effectiveness of birth control pills, and additional measures should be taken to avoid pregnancy.

Rationale: Phenytoin enhances the rate of estrogen metabolism, which can decrease the effectiveness of some birth control pills. The nurse should tell the client to alert the health care provider about the use of birth control pills so that counseling may be provided about alternative birth control methods. The other options are incorrect.

A nurse is assisting in the care of a client being discharged on phenytoin (Dilantin), 100 mg three times daily, for seizure control. When providing client teaching about this medication, the nurse should be sure to include which points? Select all that apply. 1. Break the capsules so they are easier to swallow. 2. Use a soft toothbrush while taking this medication. 3. If a dose is missed, just wait until the next one is due. 4. Alcohol should be avoided while taking this medication. 5. The medication may turn the client's urine pink, red, or brown. 6. Sore throat is a common side effect of the medication and is nothing to worry about.

Rationale: Phenytoin is an anticonvulsant that can cause gingival hyperplasia, as well as bleeding, swelling, and tenderness of the gums. The client should use good oral hygiene, gum massage, and have regular dental checkups. Alcohol interferes with the absorption of phenytoin, so it should be avoided. Change in the color of the urine is a normal reaction. A sore throat, fever, glandular swelling, or any skin reaction indicates hematological toxicity and needs to be reported.

Phenytoin (Dilantin), 100 mg to be given orally three times daily, has been prescribed to a client for seizure control. The home health nurse visits the client and provides instructions regarding the medication. Which statement, if made by the client, would indicate an understanding of the instructions? 1. "I will use a soft toothbrush to brush my teeth." 2. "It's okay to break the capsules to make it easier for me to swallow them." 3. "If I forget to take my medication, I can wait until the next dose and eliminate that dose." 4. "If my throat becomes sore, it's a normal effect of the medication, and it's nothing to be concerned about."

Rationale: Phenytoin is an anticonvulsant. Gingival hyperplasia, bleeding, swelling, and tenderness of the gums can occur with the use of this medication. The client needs to be taught good oral hygiene, gum massage, and the need for regular dentist visits. Capsules should not be chewed or broken. The client should not skip medication doses because inadequate blood levels could precipitate a seizure. The client needs to be instructed to report a sore throat, fever, glandular swelling, or any skin reaction because such findings may indicate hematological toxicity.

To promote optimal cerebral tissue perfusion in the postoperative phase following cranial surgery, the nurse should place the client with an incision in the anterior or middle fossa, in which position? 1. In 15 degrees of Trendelenburg 2. Side-lying with the head of the bed flat 3. With the head of the bed elevated at least 30 degrees 4. With the head of the bed elevated no more than 10 degrees

Rationale: Positioning of the client correctly following cranial surgery is important to avoid increased intracranial pressure and to promote optimal cerebral tissue perfusion. The surgeon's prescription for positioning is always followed. The client with an incision in the anterior or middle fossa should be positioned with the head of bed (HOB) elevated at least 30 degrees. If the incision is in the posterior fossa or burr holes have been made, the client is positioned flat, or with the HOB elevated no more than 10 to 15 degrees. If a craniectomy (bone flap) is performed, the client should not be positioned to the operative side. Trendelenburg position is contraindicated in the postoperative phase following cranial surgery.

The nurse has a prescription to administer phenytoin (Dilantin) 100 mg by the intravenous (IV) route to a client. The nurse administers the medication after preparing it in which solution? 1. Lactated Ringer's solution 2. 5% dextrose in 0.45% normal saline 3. 0.9% normal saline with an in-line filter 4. 5% dextrose in water with an in-line filter

Rationale: Precipitation will occur if phenytoin is mixed with any solution other than 0.9% (normal) saline. This is especially true with solutions containing dextrose. An in-line filter reduces the chance of precipitants entering the bloodstream. Phenytoin is very irritating to the vein wall or other tissues.

A nurse is assessing a client's muscle strength and notes that when asked, the client cannot maintain his or her hands in a supinated position with the arms extended and eyes closed. How should the nurse correctly document this finding on the medical record? 1. Client is demonstrating ataxia. 2. Client is exhibiting pronator drift. 3. Client appears to have nystagmus. 4. Client examination reveals hyperreflexia.

Rationale: Pronator drift occurs when a client cannot maintain his or her hands in a supinated position with the arms extended and eyes closed. This assessment may be done to detect small changes in muscle strength that might not otherwise be noted. Ataxia is a disturbance in gait. Hyperreflexia is an excessive reflex action. Nystagmus is characterized by fine, involuntary eye movements. It can occur with neurological disease or as a side effect of selected medications.

A client with myasthenia gravis has difficulty chewing and has received a prescription for pyridostigmine (Mestinon). The nurse should check to see that the client takes the medication at what time? 1. With meals 2. Between meals 3. Just after meals 4. 30 minutes before meals

Rationale: Pyridostigmine is a cholinergic medication used to increase muscle strength in the client with myasthenia gravis. For the client who has difficulty chewing, the medication should be administered 30 minutes before meals to enhance the client's ability to eat. The times noted in the remaining options will not be helpful to the client.

The nurse teaches the wife of a client who is receiving levodopa/carbidopa (Sinemet) to avoid pyridoxine (vitamin B6) medications. Which statement by the wife indicates an understanding of the instructions? 1. "Vitamin B6 reverses the effectiveness of levodopa, meaning a higher dose is needed." 2. "Vitamin B6 will change perspiration and urine to a dark color, which may stain clothing." 3. "Levodopa competes with vitamin B6 for absorption in the intestine, blocking absorption." 4. "The two medications in combination will cause the blood sugar to drop, causing hypoglycemia."

Rationale: Pyridoxine reverses the therapeutic effects of levodopa. Dietary restrictions are not necessary, but ingredients of multivitamins should be assesses. Many multivitamins contain pyridoxine and should be avoided. Careful reading of over-the-counter vitamin labeling is necessary to avoid increasing pyridoxine in the diet. The statements in the remaining options are incorrect.

A client with neck and upper extremity pain has been diagnosed with cervical radiculitis. The nurse understands that the client's symptoms must be caused by pressure on which structures of the vertebral column? 1. Spinous process 2. Spinal nerve root 3. Central spinal cord 4. Posterior facet joints

Rationale: Radiculitis is a term used to describe spinal nerve root compression at the intervertebral foramen. Radiculitis can be caused by a number of factors, such as whiplash or ruptured intervertebral disk. In many cases, it is caused by malalignment that occurs with degenerative disease or bone spur formation. Options 1, 3, and 4 are not associated with cervical radiculitis.

The nurse is assessing a client with a neurological deficit involving the hippocampus. Which finding is indicative of this deficit? 1. Disoriented to client, place, and time 2. Affect flat, with periods of emotional lability 3. Cannot recall what was eaten for breakfast today 4. Unable to add and subtract; does not know who is president

Rationale: Recall of recent events and the storage of memories are controlled by the hippocampus, which is a limbic system structure. The cerebral hemispheres, with specific regional functions, control orientation. The limbic system, overall, is responsible for feelings, affect, and emotions. Calculation ability and knowledge of current events are under the control of the frontal lobes of the cerebrum.

At the beginning of the work shift, the nurse assesses the status of the client wearing a halo device. The nurse determines that which assessment finding requires intervention? 1. Tightened screws 2. Red skin areas under the jacket 3. Clean and dry lamb's wool jacket lining 4. Finger-width space between the jacket and the skin

Rationale: Red skin areas under the jacket indicate that the jacket is too tight. The resulting pressure could lead to altered skin integrity and needs to be relieved by loosening the jacket. The screws all should be properly tightened. A clean, dry lamb's wool lining should be in place underneath the jacket, and there should be a finger-width space between the jacket and the skin. In addition, the client should wear a clean white cotton T-shirt next to the skin to help prevent itching.

A client is experiencing impotence after taking guanfacine (Tenex). The client states, "I would sooner have a stroke than keep living with the effects of this medication." What is the most appropriate response by the nurse? 1. "I can understand completely." 2. "You wouldn't really want to have a stroke." 3. "You are concerned about the effects of your medication." 4. "The health care provider should change your prescription."

Rationale: Reflection of the client's own comment lets the client know that the nurse hears the concern without judging. The nurse cannot understand what the client is experiencing. To tell the client "you wouldn't really want to have a stroke" is confrontational and unsupportive. The client's prescription may need to be changed, but from the options provided, this is not the most appropriate response.

The nurse is evaluating the neurological signs of a client in spinal shock following spinal cord injury. Which observation indicates that spinal shock persists? 1. Hyperreflexia 2. Positive reflexes 3. Flaccid paralysis 4. Reflex emptying of the bladder

Rationale: Resolution of spinal shock is occurring when there is return of reflexes (especially flexors to noxious cutaneous stimuli), a state of hyperreflexia rather than flaccidity, and reflex emptying of the bladder.

The nurse is performing an assessment on a client with Guillain-Barré syndrome. The nurse determines that which finding would be of most concern? 1. Difficulty articulating words 2. Lung vital capacity of 10 mL/kg 3. Paralysis progressing from the toes to the waist 4. A blood pressure (BP) decrease from 110/78 to 102/70 mm Hg

Rationale: Respiratory compromise is a major concern in clients with Guillain-Barré syndrome. Clients often are intubated and mechanically ventilated when the vital capacity is less than 15 mL/kg. Options 1 and 3 are expected, depending on the degree of paralysis that occurs. Although orthostatic hypotension is a problem with these clients, the BP drop in option 4 is less than 10 mm Hg and is not significant.

A child is diagnosed with Reye's syndrome. The nurse develops a nursing care plan for the child and should include which intervention in the plan? 1. Assessing hearing loss 2. Monitoring urine output 3. Changing body position every 2 hours 4. Providing a quiet atmosphere with dimmed lighting

Rationale: Reye's syndrome is an acute encephalopathy that follows a viral illness and is characterized pathologically by cerebral edema and fatty changes in the liver. A definitive diagnosis is made by liver biopsy. In Reye's syndrome, supportive care is directed toward monitoring and managing cerebral edema. Decreasing stimuli in the environment by providing a quiet environment with dimmed lighting would decrease the stress on the cerebral tissue and neuron responses. Hearing loss and urine output are not affected. Changing the body position every 2 hours would not affect the cerebral edema directly. The child should be positioned with the head elevated to decrease the progression of the cerebral edema and promote drainage of cerebrospinal fluid.

The nurse is caring for a client who is in the chronic phase of stroke (brain attack) and has a right-sided hemiparesis. The nurse identifies that the client is unable to feed self. Which is a priority nursing intervention? 1. Assist the client to eat with the left hand to build strength. 2. Provide a pureed diet that is easy for the client to swallow. 3. Inform the client that a feeding tube will be placed if progress is not made. 4. Provide a variety of foods on the meal tray to stimulate the client's appetite.

Rationale: Right-sided hemiparesis is weakness of the right arm and leg. The nurse should teach the client to use both sides of the body to increase strength and build endurance. Option 2 is incorrect. The question does not mention swallowing difficulty, so there is no need to puree the food. Option 3 is incorrect. That information would come from the health care provider. Option 4 is incorrect. The problem is not the food selection but the client's ability to eat the food independently.

The nurse in a long-term care facility is reviewing the health care provider's (HCP) prescriptions on an assigned client. The nurse notes that the HCP prescribed ropinirole hydrochloride (Requip). The nurse determines that this medication has been prescribed to treat which condition in the client? 1. Depression 2. Diabetes mellitus 3. Coronary artery disease 4. Parkinsonian syndrome

Rationale: Ropinirole hydrochloride is a medication that is used to treat idiopathic parkinsonian syndrome. It normally is administered three times a day to treat the client. This medication is not used to treat depression, diabetes mellitus, or coronary artery disease.

The nurse is evaluating the respiratory outcomes for a client with Guillain-Barré syndrome. The nurse determines that which is the least optimal outcome for the client? 1. Spontaneous breathing 2. Oxygen saturation of 98% 3. Adventitious breath sounds 4. Vital capacity within normal range

Rationale: Satisfactory respiratory outcomes for a client with Guillain-Barré syndrome include clear breath sounds on auscultation, spontaneous breathing, normal vital capacity, normal arterial blood gas levels, and normal pulse oximetry. Adventitious breath sounds are an abnormal finding.

A nursing student is assisting a school nurse in performing scoliosis screening on the children in the school. The nurse assesses the student's preparation for conducting the screening. The nurse determines that the student demonstrates understanding of the disorder when the student states that scoliosis is characterized by which finding? 1. Abnormal lateral curvature of the spine 2. Abnormal anterior curvature of the lumbar spine 3. Excessive posterior curvature of the thoracic spine 4. Abnormal curvature of the spine caused by inflammation

Rationale: Scoliosis is defined as an abnormal lateral curvature in any area of the spine. The region of the spine most commonly affected is the right thoracic area, where it results in rib prominence. Option 2 describes lordosis, which usually is exaggerated during pregnancy, in obesity, or in persons with large tumors. Option 3 describes kyphosis, which also is known as humpback. Scoliosis does not occur as a sequela of inflammation.

The nurse is planning to institute seizure precautions for a client who is being admitted from the emergency department. Which measures should the nurse include in planning for the client's safety? Select all that apply. 1. Padding the side rails of the bed 2. Placing an airway at the bedside 3. Placing the bed in the high position 4. Putting a padded tongue blade at the head of the bed 5. Placing oxygen and suction equipment at the bedside 6. Having intravenous equipment ready for insertion of an intravenous catheter

Rationale: Seizure precautions may vary from agency to agency, but they generally have some common features. Usually, an airway, oxygen, and suctioning equipment are kept available at the bedside. The side rails of the bed are padded, and the bed is kept in the lowest position. The client has an intravenous access in place to have a readily accessible route if anticonvulsant medications must be administered. The use of padded tongue blades is highly controversial, and they should not be kept at the bedside. Forcing a tongue blade into the mouth during a seizure more likely will harm the client who bites down during seizure activity. Risks include blocking the airway from improper placement, chipping the client's teeth, and subsequent risk of aspirating tooth fragments. If the client has an aura before the seizure, it may give the nurse enough time to place an oral airway before seizure activity begins.

A client had a seizure 1 hour ago. Family members were present during the episode and reported that the client's jaw was moving as though grinding food. In helping to determine the origin of this seizure, what should the nurse include in the client's assessment? 1. Presence of diaphoresis 2. Loss of consciousness 3. History of prior trauma 4. Rotating eye movements

Rationale: Seizures that originate with specific motor phenomena are considered focal and are indicative of a focal structural lesion in the brain, often caused by trauma, infection, or drug consumption. Options 1, 2, and 4 address signs, rather than an origin of the seizure.

The clinic nurse is reviewing the record of a client scheduled to be seen in the clinic. The nurse notes that the client is taking selegiline hydrochloride (Eldepryl). The nurse suspects that the client has which disorder? 1. Diabetes mellitus 2. Parkinson's disease 3. Alzheimer's disease 4. Coronary artery disease

Rationale: Selegiline hydrochloride is an antiparkinsonian medication. The medication increases dopaminergic action, assisting in the reduction of tremor, akinesia, and the rigidity of parkinsonism. This medication is not used to treat diabetes mellitus, Alzheimer's disease, or coronary artery disease.

Meperidine hydrochloride (Demerol) is prescribed for a client with pain. What should the nurse monitor for as a side/adverse effect of this medication? 1. Diarrhea 2. Bradycardia 3. Hypertension 4. Urinary retention

Rationale: Side/adverse effects of meperidine include respiratory depression, orthostatic hypotension, tachycardia, drowsiness and mental clouding, constipation, and urinary retention; therefore the remaining options are incorrect.

The nurse is caring for a client who underwent an open reduction internal fixation to the right hip. When administering opioid analgesics for pain, the nurse should instruct the client that which are side/adverse effects of opioid analgesics? Select all that apply. 1. Sedation 2. Diarrhea 3. Constipation 4. Increased pain level 5. Respiratory depression

Rationale: Side/adverse effects of opioids include constipation, sedation, and respiratory depression. Diarrhea is not a side effect. The pain level should decrease not increase.

A client with Parkinson's disease has begun therapy with levodopa. The nurse determines that the client understands the action of the medication if he or she verbalizes that results may not be apparent for how long? 1. 1 week 2. 24 hours 3. 2 to 3 days 4. 2 to 3 weeks

Rationale: Signs and symptoms of Parkinson's disease usually begin to resolve within 2 to 3 weeks of starting therapy, although in some clients marked improvement may not be seen for up to 6 months. The client needs to understand this concept to aid in compliance with medication therapy.

The nurse is evaluating the status of a client who had a craniotomy 3 days ago. Which assessment finding would indicate that the client is developing meningitis as a complication of surgery? 1. A negative Kernig sign 2. Absence of nuchal rigidity 3. A positive Brudzinski sign 4. A Glasgow Coma Scale score of 15

Rationale: Signs of meningeal irritation compatible with meningitis include nuchal rigidity, a positive Brudzinski sign, and positive Kernig sign. Nuchal rigidity is characterized by a stiff neck and soreness, which is especially noticeable when the neck is flexed. Kernig's sign is positive when the client feels pain and spasm of the hamstring muscles when the leg is fully flexed at the knee and hip. Brudzinski's sign is positive when the client flexes the hips and knees in response to the nurse gently flexing the head and neck onto the chest. A Glasgow Coma Scale score of 15 is a perfect score and indicates that the client is awake and alert, with no neurological deficits.

A client with suspected opioid overdose has received a dose of naloxone hydrochloride (Narcan). The client subsequently becomes restless, starts to vomit, and complains of abdominal cramping. The blood pressure increases from 110/72 mm Hg to 160/86 mm Hg. The nurse provides emotional support and reassurance while administering care to the client, knowing which piece of information? 1. The client may next become suicidal. 2. These are signs of opioid withdrawal. 3. These effects will last only a few moments. 4. The client may otherwise sign out against medical advice.

Rationale: Signs of opioid withdrawal include increased temperature and blood pressure, abdominal cramping, vomiting, and restlessness. Time of onset may be anywhere from a few minutes to a few hours after administration of naloxone hydrochloride, depending on the opioid involved, the degree of dependence, and the dose of naloxone. Options 1, 3, and 4 are incorrect interpretations.

Members of the family of an unconscious client with increased intracranial pressure are talking at the client's bedside. They are discussing the client's condition and wondering whether the client will ever recover. The nurse intervenes on the basis of which interpretation? 1. It is possible the client can hear the family. 2. The family needs immediate crisis intervention. 3. The client might have wanted a visit from the hospital chaplain. 4. The family could benefit from a conference with the health care provider.

Rationale: Some clients who have awakened from an unconscious state have remembered hearing specific voices and conversations. Family and staff should assume that the client's sense of hearing is intact and act accordingly. Additionally, positive outcomes are associated with coma stimulation-that is, speaking to and touching the client. Options 2, 3, and 4 are incorrect interpretations.

The nurse is planning care for the client with a neurogenic bladder caused by multiple sclerosis. The nurse plans for fluid administration of at least 2000 mL/day. Which plan would be most helpful to this client? 1. 400 to 500 mL with each meal and 500 to 600 mL in the evening before bedtime 2. 400 to 500 mL with each meal and additional fluids in the morning but not after midday 3. 400 to 500 mL with each meal, with all extra fluid concentrated in the afternoon and evening 4. 400 to 500 mL with each meal and 200 to 250 mL at midmorning, midafternoon, and late afternoon

Rationale: Spacing fluid intake over the day helps the client with a neurogenic bladder to establish regular times for successful voiding. Omitting intake after the evening meal minimizes incontinence or the need to empty the bladder during the night.

The nurse is caring for a client who is brought to the hospital emergency department with a spinal cord injury. The nurse minimizes the risk of compounding the injury most effectively by performing which action? 1. Keeping the client on a stretcher 2. Logrolling the client onto a soft mattress 3. Logrolling the client onto a firm mattress 4. Placing the client on a bed that provides spinal immobilization

Rationale: Spinal immobilization is necessary after spinal cord injury to prevent further damage and insult to the spinal cord. Whenever possible, the client is placed on a special bed, such as a Stryker frame, which allows the nurse to turn the client to prevent complications of immobility while maintaining alignment of the spine. If a Stryker frame is not available, a firm mattress with a bed board under it should be used. Options 1, 2, and 3 are incorrect and potentially harmful interventions.

A client with a neurological problem is experiencing hyperthermia. Which measure would be least appropriate for the nurse to use in trying to lower the client's body temperature? 1. Giving tepid sponge baths 2. Applying a hypothermia blanket 3. Placing ice packs in the axilla and groin areas 4. Administering acetaminophen (Tylenol) per protocol

Rationale: Standard measures to lower body temperature include removing bed covers, providing cool sponge baths, using an electric fan in the room, administering acetaminophen, and placing a hypothermia blanket under the client. Ice packs are not used because they could cause shivering, which increases cellular oxygen demands, with the potential for increased intracranial pressure.

Sumatriptan (Imitrex) is prescribed for a client. Based in this prescription, the nurse suspects that the client has which condition? 1. Sinus headache 2. Simple headache 3. Migraine headache 4. Episodic headache

Rationale: Sumatriptan is used to treat migraine headaches. This medication constricts blood vessels around the brain and reduces substances in the body that can trigger headache pain. Sinus, simple, and episodic headaches are not treated with this medication.

The nursing student is assigned to care for a child with a brain injury who has a temporal lobe herniation. The nursing instructor determines that the student needs to further research this type of injury if the student states that which finding is a characteristic of this type of herniation? 1. It can cause ipsilateral pupil dilation. 2. It produces compression of the sixth cranial nerve. 3. A shifting of the temporal lobe laterally across the tentorial notch occurs. 4. Flaccid paralysis, pupil fixation, and death can occur if the intracranial pressure continues to rise.

Rationale: Temporal lobe herniation or uncal herniation refers to a shifting of the temporal lobe laterally across the tentorial notch. This produces compression of the third cranial nerve and ipsilateral pupil dilation. If pressure continues to rise, flaccid paralysis, pupil fixation, and death will result.

The post-head injury client opens eyes to sound, has no verbal response, and localizes to painful stimuli when applied to each extremity. How should the nurse document the Glasgow Coma Scale (GCS) score? 1. GCS = 3 2. GCS = 6 3. GCS = 9 4. GCS = 11

Rationale: The GCS is a method is assessing neurological status. The highest possible score in the GCS is 15. A score lower than 8 indicates that coma is present. Motor response points are as follows: Obeys a simple response = 6; Localizes painful stimuli = 5; Normal flexion (withdrawal) = 4; Abnormal flexion (decorticate posturing) = 3; Extensor response (decerebrate posturing) = 2; No motor response to pain = 1. Verbal response points are as follows: Oriented = 5; Confused conversation = 4; Inappropriate words = 3; Responds with incomprehensible sounds = 2; No verbal response = 1. Eye opening points are as follows: Spontaneous = 4; In response to sound = 3; In response to pain = 2; No response, even to painful stimuli = 1. Using the GCS, a score of 3 is given when the client opens the eyes to sound. Localization to pain is scored as 5. When there is no verbal response the score is a 1. The total score is then equal to 9.

A client is somewhat nervous about undergoing magnetic resonance imaging (MRI). Which statement by the nurse would provide the most reassurance to the client about the procedure? 1. "The MRI machine is a long, narrow, hollow tube and may make you feel somewhat claustrophobic." 2. "You will be able to eat before the procedure unless you get nauseated easily. If so, you should eat lightly." 3. "Even though you are alone in the scanner, you will be in voice communication with the technologist at all times during the procedure." 4. "It is necessary to remove any metal or metal-containing objects before having the MRI done to avoid the metal being drawn into the magnetic field."

Rationale: The MRI scanner is a hollow tube that gives some clients a feeling of claustrophobia. Metal objects must be removed before the procedure so that they are not drawn into the magnetic field. The client may eat and may take all prescribed medications before the procedure. If a contrast medium is used, the client may wish to eat lightly if he or she has a tendency to become nauseated easily. The client lies supine on a padded table that moves into the imager. The client must lie still during the procedure. The imager makes tapping noises during the scanning. The client is alone in the imager, but the nurse can reassure the client that the technologist will be in voice communication with the client at all times during the procedure.

Transcutaneous electrical nerve stimulation (TENS) is prescribed for a client with pain, and the nurse instructs the client about the purpose of the TENS unit. Which statement by the client indicates the need for further instruction? 1. "The unit relieves pain." 2. "Electrodes are attached to the skin." 3. "The unit will reduce the need for analgesics." 4. "Hospitalization is required because the unit is not portable."

Rationale: The TENS unit is portable and the client controls the system for relieving pain and reducing the need for analgesics. It is attached to the skin of the body by electrodes. Hospitalization is not required.

A client with a neurological deficit is able, with eyes closed, to identify a set of keys placed in his or her hands. A nurse observing the client interprets this to mean that which area of the brain is intact? 1. Frontal lobe 2. Parietal lobe 3. Temporal lobe 4. Occipital lobe

Rationale: The ability to distinguish an object by touch is called stereognosis, which is a function of the right parietal area. The parietal lobe of the brain is responsible for spatial orientation and awareness of sizes and shapes. The left parietal area is responsible for mathematics and right-left orientation. The other lobes of the brain are not responsible for this function.

The nurse is providing instructions to an adolescent prescribed phenytoin (Dilantin) for the control of seizures. Which statement by the adolescent indicates a need for further teaching regarding the medication? 1. "The medication may cause acne or oily skin." 2. "Drinking alcohol may affect the medication." 3." If my gums become sore and swollen I need to stop the medication." 4. "Birth control pills may not be effective when I take this medication."

Rationale: The adolescent should not stop taking antiseizure medications suddenly or without discussing it with a health care provider (HCP) or nurse. Acne or oily skin may be a problem for the adolescent, and the adolescent is advised to call a health care provider (HCP) for skin problems. Alcohol will lower the seizure threshold, and it is best to avoid the use of alcohol. Birth control pills may be less effective when the client is taking antiseizure medication.

A nurse is monitoring an infant for signs of increased intracranial pressure (ICP). On assessment of the fontanelles, the nurse notes that the anterior fontanelle bulges when the infant is sleeping. Based on this finding, which is the priority nursing action? 1. Increase oral fluids. 2. Document the finding. 3. Notify the health care provider. 4. Place the infant supine in a side-lying position.

Rationale: The anterior fontanelle is diamond shaped and is located on the top of the head. It should be soft and flat in a normal infant, and it normally closes by 12 to 18 months of age. A larger-than-normal fontanelle may be a sign of increased ICP within the skull. Although the anterior fontanelle may bulge slightly when the infant cries, bulging at rest may indicate increased ICP. Options 1 and 4 are inaccurate interventions and will not be helpful. Although the nurse would document the finding, the priority action would be to report the finding to the health care provider.

The nurse is administering medications to a client with trigeminal neuralgia. The nurse expects that which medication will be prescribed for pain relief? 1. Oxycodone plus aspirin (Percodan) 2. Acetaminophen (Tylenol) and codeine sulfate 3. Carbamazepine (Tegretol) and gabapentin (Neurontin) 4. Meperidine hydrochloride (Demerol) and hydroxyzine (Vistaril)

Rationale: The anticonvulsant medications carbamazepine and gabapentin (Neurontin) help relieve the pain in many clients with trigeminal neuralgia. They act by inhibiting the reactivity of neurons in the trigeminal nerve. Opioid analgesics (meperidine hydrochloride, codeine sulfate, and oxycodone) are not very effective in controlling pain caused by trigeminal neuralgia.

The nurse is caring for a client in the emergency department who has been diagnosed with Bell's palsy. The client has been taking acetaminophen (Tylenol), and acetaminophen overdose is suspected. Which antidote should the nurse anticipate to be prescribed? 1. Pentostatin (Nipent) 2. Auranofin (Ridaura) 3. Fludarabine (Fludara) 4. Acetylcysteine (Mucomyst)

Rationale: The antidote for acetaminophen is acetylcysteine (Mucomyst). The normal therapeutic serum level of acetaminophen is 10 to 20 mcg/mL. A toxic level is higher than 50 mcg/mL, and levels higher than 200 mcg/mL could indicate hepatotoxicity. Auranofin (Ridaura) is a gold preparation used to treat rheumatoid arthritis. Pentostatin (Nipent) and fludarabine (Fludara) are antineoplastic agents.

A client has a cerebellar lesion. The nurse would plan to obtain which item for use by this client? 1. Walker 2. Slider board 3. Raised toilet seat 4. Adaptive eating utensils

Rationale: The cerebellum is responsible for balance and coordination. A walker provides stability for the client during ambulation. A raised toilet seat is useful if the client has sufficient mobility or ability to flex the hips. A slider board is used in transferring a client with weak or paralyzed legs from a bed to stretcher or wheelchair. Adaptive eating utensils are beneficial if the client has partial paralysis of the hand.

A client has a cerebellar lesion. The nurse determines that the client is adapting successfully to this problem if the client demonstrates proper use of which item? 1. Walker 2. Slider board 3. Raised toilet seat 4. Adaptive eating utensils

Rationale: The cerebellum is responsible for balance and coordination. A walker would provide stability for the client during ambulation. Adaptive eating utensils may be beneficial if the client has partial paralysis of the hand. A raised toilet seat is useful if the client does not have the mobility or ability to flex the hips. A slider board is used in transferring a client from a bed to a stretcher or wheelchair.

A nurse overhears a neurologist saying that a client has an aneurysm located in the circle of Willis. The nurse understands that which blood vessels are parts of the circle of Willis? Select all that apply. 1. Basilar artery 2. Vertebral artery 3. Anterior cerebral artery 4. Posterior cerebral artery 5. Anterior communicating artery

Rationale: The circle of Willis is a ring of blood vessels located at the base of the brain. It is referred to as the anterior circulation to the brain and is composed of the anterior and middle cerebral arteries, posterior cerebral arteries, posterior communicating arteries, internal carotid arteries, and anterior communicating branches. The basilar artery and vertebral artery are not part of the circle of Willis. Rather, they are part of the vertebral-basilar system, which is known as the posterior circulation to the brain. Other parts of the posterior circulation are the posterior inferior cerebellar artery and the spinal arteries.

The community health nurse is providing information to parents of children in a local school regarding the signs of meningitis. The nurse informs the parents that the classic signs/symptoms of meningitis include which findings? 1. Nausea, delirium, and fever 2. Severe headache and back pain 3. Photophobia, fever, and confusion 4. Severe headache, fever, and a change in the level of consciousness

Rationale: The classic signs/symptoms of meningitis include severe headache, fever, stiff neck, and a change in the level of consciousness. Photophobia also may be a prominent early manifestation and is thought to be related to meningeal irritation. Although nausea, confusion, delirium, and back pain may occur in meningitis, these are not the classic signs/symptoms.

The home health nurse is visiting a client with myasthenia gravis and is discussing methods to minimize the risk of aspiration during meals related to decreased muscle strength. Which suggestions should the nurse give to the client? Select all that apply. 1. Chew food thoroughly. 2. Cut food into very small pieces. 3. Sit straight up in the chair while eating. 4. Lift the head while swallowing liquids. 5. Swallow when the chin is tipped slightly downward to the chest.

Rationale: The client avoids swallowing any type of food or drink with the head lifted upward, which could actually cause aspiration by opening the glottis. The client should be advised to sit upright while eating, not to talk with food in the mouth (talking requires opening the glottis), cut food into very small pieces, chew thoroughly, and tip the chin downward to swallow.

The client with a spinal cord injury at the level of T4 is experiencing a severe throbbing headache with a blood pressure of 180/100 mm Hg. What is the priority nursing intervention? 1. Notify the health care provider. 2. Loosen tight clothing on the client. 3. Place the client in a sitting position. 4. Check the urinary catheter tubing for kinks or obstruction.

Rationale: The client is demonstrating clinical manifestations of autonomic dysreflexia, which is a neurological emergency. The first priority is to place the client in a sitting position to prevent hypertensive stroke. Options 2 and 4 can then be done, and option 1 can be completed once initial interventions are done.

A client has been prescribed a cough formula containing codeine sulfate. The nurse has given the client instructions for its use. The nurse concludes that the client understands the instructions if the client verbalizes to self-assess for which side effect? 1. Excitability 2. Rapid pulse 3. Constipation 4. Excessive urination

Rationale: The client is taught about side effects and adverse effects that could occur with the use of codeine sulfate. The most common side effects include drowsiness, confusion, hypotension, nausea and vomiting, and constipation. Adverse effects include bradycardia, respiratory depression, and urinary retention.

The nurse has taught a client with a herniated lumbar disk about proper body mechanics and other items pertinent to low back care. The nurse determines that further instruction is needed if the client states the need to take which action? 1. Bend at the knees to pick up objects. 2. Increase fiber and fluid intake in the diet. 3. Strengthen the back muscles by swimming or walking. 4. Get out of bed by sitting straight up and swinging the legs over the side of the bed.

Rationale: The client is taught to get out of bed by sliding near the edge of the mattress. The client then rolls onto one side and pushes up from the bed using one or both arms. The client keeps the back straight and swings the legs over the side. Proper body mechanics includes bending at the knees, not the waist, to lift objects. Increasing fluid intake and dietary fiber helps prevent straining at stool, thereby preventing increases in intraspinal pressure. Walking and swimming are excellent exercises for strengthening lower back muscles.

Which intervention should the nurse include in a postoperative teaching plan for a client who underwent a spinal fusion and will be wearing a brace? 1. Tell the client to inspect the environment for safety hazards. 2. Inform the client about the importance of sitting as much as possible. 3. Inform the client that lotions and body powders can be used for skin breakdown. 4. Instruct the client to tighten the brace during meals and to loosen it for the first 30 minutes after each meal.

Rationale: The client must inspect the environment for safety hazards. The client is instructed in the importance of avoiding prolonged sitting and standing. Powders and lotions should not be used because they may irritate the skin. The client should be taught to loosen the brace during meals and for 30 minutes after each meal. The client may have difficulty eating if the brace is too tight. Loosening the brace after each meal will allow adequate nutritional intake and promote comfort.

The nurse is teaching a client hospitalized with a seizure disorder and the client's spouse about safety precautions after discharge. The nurse determines that the client needs more information if he or she states an intention to take which action? 1. Refrain from smoking alone. 2. Take all prescribed medications on time. 3. Have the spouse nearby when showering. 4. Drink alcohol in small amounts and only on weekends.

Rationale: The client should avoid the intake of alcohol. Alcohol could interact with the client's seizure medications, or the alcohol could precipitate seizure activity. The client should take all medications on time to avoid decreases in therapeutic drug levels, which could precipitate seizures. The client should not bathe in the shower or tub without someone nearby and should not smoke alone, to minimize the risk of injury if a seizure occurs.

A client has been prescribed cyclobenzaprine (Flexeril) for the treatment of painful muscle spasms accompanying a herniated intervertebral disk. The nurse should withhold the medication and question the prescription if the client has a concurrent prescription for which medication? 1. Furosemide (Lasix) 2. Ibuprofen (Motrin IB) 3. Valproic acid (Depakene) 4. Tranylcypromine (Parnate)

Rationale: The client should not receive cyclobenzaprine if the client has taken monoamine oxidase inhibitors such as tranylcypromine or phenelzine within the last 14 days. Otherwise, the client could experience hyperpyretic crisis, seizures, and possibly death. The medications in the remaining options are not contraindicated.

The nurse is providing instructions to a client with a seizure disorder who will be taking phenytoin (Dilantin). Which statement, if made by the client, would indicate an understanding of the information about this medication? 1. "I need to perform good oral hygiene, including flossing and brushing my teeth." 2. "I should try to avoid alcohol, but if I'm not able to, I can drink alcohol in moderation." 3. "I should take my medication before coming to the laboratory to have a blood level drawn." 4. "I should monitor for side effects and adjust my medication dose depending on how severe the side effects are."

Rationale: The client should perform good oral hygiene, including flossing and brushing the teeth. The client also should see a dentist at regularly scheduled times because gingival hyperplasia is a side effect of this medication. The client should avoid alcohol while taking this medication. The client should also be instructed that follow-up serum blood levels are important and that, on the day of the scheduled laboratory test, the client should avoid taking the medication before the specimen is drawn. The client should not adjust medication dosages.

A client with myasthenia gravis who is taking neostigmine (Prostigmin) is experiencing frequent exacerbations of myasthenic crisis and cholinergic crisis. The nurse teaches the client that it is most important that this medication be taken in which manner? 1. On time 2. On an empty stomach 3. Double-dosed if one dose is missed 4. Titrated for dosage, depending on the symptoms

Rationale: The client should take neostigmine exactly on time. Taking the medication early or late could result in myasthenic or cholinergic crisis. Taking the medication on time is especially important for the client with dysphagia because the client may not be able to swallow the medication if it is given late. These clients are taught to set an alarm clock to remind them of dosage times. The medication should be administered with food or milk to minimize side effects and adverse effects. The client should never skip or double-up on missed doses or titrate the dose, depending on symptoms. The client needs to take the medication exactly as prescribed.

The nurse has given medication instructions to a client beginning anticonvulsant therapy with carbamazepine (Tegretol). The nurse determines that the client understands the use of the medication if he or she makes which statement? 1. Use sunscreen when outdoors. 2. Drive a car as long as it is not at night. 3. Keep tissues handy because of excess salivation. 4. Discontinue the medication if fever or sore throat occurs.

Rationale: The client should use protective clothing and sunscreen to avoid photosensitivity reactions. Carbamazepine acts by depressing synaptic transmission in the central nervous system (CNS). Because of this, the client should avoid driving at any time or doing other activities that require mental alertness until the effect of the medication on the client is known. The medication may cause dry mouth, and he or she should be instructed to provide good oral hygiene and use sugarless candy or gum as needed. The medication should not be abruptly discontinued because this could lead to return of seizures or status epilepticus. Fever and sore throat (leukopenia) should be reported to the health care provider.

A nurse is caring for a client receiving codeine sulfate for pain. The nurse determines that the client is experiencing a side effect of the medication based on which finding? 1. Distended jugular veins 2. Bounding peripheral pulses 3. No bowel movement in 3 days 4. Change in blood pressure from 120/60 to 140/80 mm Hg

Rationale: The client taking codeine sulfate is at risk for constipation. Thus, the nurse monitors the frequency of bowel movements. The nurse also would monitor the client for hypotension, decreased respirations, and urinary retention. The nurse would plan measures to counteract these expected effects, such as encouraging fluids, coughing and deep breathing, and increasing mobility to the extent tolerated by the client.

The nurse has just admitted to the nursing unit a client with a basilar skull fracture who is at risk for increased intracranial pressure (ICP). Pending specific health care provider prescriptions, the nurse should avoid placing the client in which positions? 1. Head midline 2. Neck in neutral position 3. Flat, with head turned to the side 4. Head of bed elevated 30 to 45 degrees

Rationale: The client who is at risk for or with increased ICP should be positioned so that the head is in a neutral, midline position. The nurse should avoid flexing or extending the client's neck or turning the head from side to side. The head of the bed should be raised to 30 to 45 degrees. Use of proper positions promotes venous drainage from the cranium to keep ICP down.

A client with Guillain-Barré syndrome has ascending paralysis and is intubated and receiving mechanical ventilation. Which strategy should the nurse incorporate in the plan of care to help the client cope with this illness? 1. Giving client full control over care decisions and restricting visitors 2. Providing positive feedback and encouraging active range of motion 3. Providing information, giving positive feedback, and encouraging relaxation 4. Providing intravenously administered sedatives, reducing distractions, and limiting visitors

Rationale: The client with Guillain-Barré syndrome experiences fear and anxiety from the ascending paralysis and sudden onset of the disorder. The nurse can alleviate these fears by providing accurate information about the client's condition, giving expert care and positive feedback to the client, and encouraging relaxation and distraction. The family can become involved with selected care activities and provide diversion for the client as well.

The nurse is admitting a client with Guillain-Barré syndrome to the nursing unit. The client has ascending paralysis to the level of the waist. Knowing the complications of the disorder, the nurse should bring which most essential items into the client's room? 1. Nebulizer and pulse oximeter 2. Blood pressure cuff and flashlight 3. Flashlight and incentive spirometer 4. Electrocardiographic monitoring electrodes and intubation tray

Rationale: The client with Guillain-Barré syndrome is at risk for respiratory failure because of ascending paralysis. An intubation tray should be available for use. Another complication of this syndrome is cardiac dysrhythmias, which necessitates the use of electrocardiographic monitoring. Because the client is immobilized, the nurse should assess for deep vein thrombosis and pulmonary embolism routinely. Although items in the incorrect options may be used in care, they are not the most essential items from the options provided.

The nurse has given instructions to a client with Parkinson's disease about maintaining mobility. Which action demonstrates that the client understands the directions? 1. Sits in soft, deep chairs to promote comfort. 2. Exercises in the evening to combat fatigue. 3. Rocks back and forth to start movement with bradykinesia. 4. Buys clothes with many buttons to maintain finger dexterity.

Rationale: The client with Parkinson's disease should exercise in the morning when energy levels are highest. The client should avoid sitting in soft deep chairs because they are difficult to get up from. The client can rock back and forth to initiate movement. The client should buy clothes with Velcro fasteners and slide-locking buckles to support the ability to dress self.

The nurse is caring for a client with a diagnosis of right (nondominant) hemispheric stroke. The nurse notes that the client is alert and oriented to time and place. On the basis of these assessment findings, the nurse should make which interpretation? 1. Had a very mild stroke 2. Most likely suffered a transient ischemic attack 3. May have difficulty with language abilities only 4. Is likely to have perceptual and spatial disabilities

Rationale: The client with a right (nondominant) hemispheric stroke may be alert and oriented to time and place. These signs of apparent wellness often suggest that the client is less disabled than is the case. However, impulsivity and confusion in carrying out activities may be very real problems for these clients as a result of perceptual and spatial disabilities. The right hemisphere is considered specialized in sensory-perceptual and visual-spatial processing and awareness of body space. The left hemisphere is dominant for language abilities.

A client who has had a stroke (brain attack) has residual dysphagia. When a diet prescription is initiated, the nurse should avoid which action? 1. Giving the client thin liquids 2. Thickening liquids to the consistency of oatmeal 3. Placing food on the unaffected side of the mouth 4. Allowing plenty of time for chewing and swallowing

Rationale: The client with dysphagia is started on a diet only after the gag and swallow reflexes have returned. The client is assisted with meals as needed and is given ample time to chew and swallow. Food is placed on the unaffected side of the mouth. Liquids are thickened to avoid aspiration.

The nurse has provided instructions to a client with a diagnosis of myasthenia gravis about home care measures. Which client statement indicates the need for further instruction? 1. "I will rest each afternoon after my walk." 2. "I should cough and deep breathe many times during the day." 3. "I can change the time of my medication on the mornings when I feel strong." 4. "If I get abdominal cramps and diarrhea, I should call my health care provider."

Rationale: The client with myasthenia gravis and his or her family should be taught information about the disease and its treatment. They should be aware of the side and adverse effects of anticholinesterase medications and corticosteroids and should be taught that timing of anticholinesterase medication is critical. It is important to instruct the client to administer the medication on time to maintain a chemical balance at the neuromuscular junction. If it is not given on time, the client may become too weak to even swallow. Resting after a walk, coughing and deep breathing many times over the day, and calling the health care provider when experiencing abdominal cramps and diarrhea indicate correct understanding of home care instructions to maintain health with this neurological degenerative disease.

A client with myasthenia gravis is having difficulty with airway clearance and difficulty with maintaining an effective breathing pattern. The nurse should keep which items available at the client's bedside? 1. Oxygen and metered-dose inhaler 2. Ambu bag and suction equipment 3. Pulse oximeter and cardiac monitor 4. Incentive spirometer and cough pillow

Rationale: The client with myasthenia gravis may experience episodes of respiratory distress if excessively fatigued or with development of myasthenic or cholinergic crisis. For this reason, an Ambu bag, intubation tray, and suction equipment should be available at the bedside.

At the end of the work shift, the nurse is reviewing the respiratory status of a client admitted with acute brain attack (stroke) earlier in the day. The nurse determines that the client's airway is patent if which data are identified? 1. Respiratory rate 24 breaths/min, oxygen saturation 94%, breath sounds clear 2. Respiratory rate 18 breaths/min, oxygen saturation 98%, breath sounds clear 3. Respiratory rate 16 breaths/min, oxygen saturation 85%, wheezes bilaterally 4. Respiratory rate 20 breaths/min, oxygen saturation 92%, diminished breath sounds in lung bases

Rationale: The client's airway is most protected if all of the respiratory parameters measured fall within normal limits. Therefore the respiratory rate should ideally be 16 to 20 breaths/min, the oxygen saturation should be greater than 95%, and the breath sounds should be clear. The correct option is the only one that meets all three criteria.

The nurse is documenting nursing observations in the record of a client who experienced a tonic-clonic seizure. Which clinical manifestation did the nurse most likely note in the clonic phase of the seizure? 1. Body stiffening 2. Spasms of the entire body 3. Sudden loss of consciousness 4. Brief flexion of the extremities

Rationale: The clonic phase of a seizure is characterized by alternating spasms and momentary muscular relaxation of the entire body, accompanied by strenuous hyperventilation. The face is contorted and the eyes roll. Excessive salivation results in frothing from the mouth. The tongue may be bitten, the client sweats profusely, and the pulse is rapid. The clonic jerking subsides by slowing in frequency and losing strength of contractions over a period of 30 seconds. Options 1, 3, and 4 identify the tonic phase of a seizure.

An adult client has a diagnosis of hydrocephalus. The nurse plans care, knowing that this condition leads to more serious neurological consequences in adults as a result of closure of which structures? 1. Cranial sutures 2. Arachnoid villi 3. Foramen of Monro 4. Aqueduct of Sylvius

Rationale: The closure of cranial sutures during childhood prevents expansion of the cranial vault when hydrocephalus occurs in the adult. This leads to increased neurological changes with lesser degrees of hydrocephalus compared with hydrocephalus during early childhood. The other structures identified are associated with cerebrospinal fluid formation and circulation, but these do not close off.

A client has dysfunction of the cochlear division of the vestibulocochlear nerve (cranial nerve VIII). The nurse should determine that the client is adequately adapting to this problem if he or she states a plan to obtain which item? 1. A walker 2. Eyeglasses 3. A hearing aid 4. A bath thermometer

Rationale: The cochlear division of cranial nerve VIII is responsible for hearing. Clients with hearing difficulty may benefit from the use of a hearing aid. The vestibular portion of this nerve controls equilibrium; difficulty with balance caused by dysfunction of this division could be addressed with use of a walker. Eyeglasses would correct visual problems (cranial nerve II); a bath thermometer would be of use to clients with sensory deficits of peripheral nerves, such as with diabetic neuropathy.

A client is diagnosed with Bell's palsy. The nurse assessing the client expects to note which symptom? 1. A symmetrical smile 2. Difficulty closing the eyelid on the affected side 3. Narrowing of the palpebral fissure on the affected side 4. Paroxysms of excruciating pain in the lips and cheek on the affected side

Rationale: The facial drooping associated with Bell's palsy makes it difficult for the client to close the eyelid on the affected side. A widening of the palpebral fissure (the opening between the eyelids) and an asymmetrical smile are seen with Bell's palsy. Paroxysms of excruciating pain are characteristic of trigeminal neuralgia.

The nurse has completed discharge instructions for a client with application of a halo device. Which action indicates that the client needs further clarification of the instructions? 1. Uses a straw for drinking 2. Drives only during the daytime 3. Uses caution because the device alters balance 4. Washes the skin daily under the lamb's wool liner of the vest

Rationale: The halo device alters balance and can cause fatigue because of its weight. The client should cleanse the skin daily under the vest to protect the skin from ulceration and should avoid the use of powder or lotions. The liner should be changed if odor becomes a problem. The client should have food cut into small pieces to facilitate chewing and use a straw for drinking. Pin care is done as instructed. The client cannot drive at all because the device impairs the range of vision.

The nurse is positioning a client who has increased intracranial pressure. Which position should the nurse avoid? 1. Head midline 2. Head turned to the side 3. Neck in neutral position 4. Head of bed elevated 30 to 45 degrees

Rationale: The head of a client with increased intracranial pressure should be kept in a neutral midline position. The nurse should avoid flexing or extending the client's neck or turning the head from side to side. The head of the bed should be raised to 30 to 45 degrees. Use of proper positions promotes venous drainage from the cranium to keep intracranial pressure down.

The nurse is caring for a client who has just been admitted to the hospital with a diagnosis of a hemorrhagic stroke. The nurse should plan to place the client in which position? 1. Prone 2. Supine 3. Semi-Fowler's with the hip and the neck flexed 4. Head of the bed elevated 30 degrees with the head in midline position

Rationale: The health care provider's prescriptions are always followed with regard to positioning the client after stroke. Clients with hemorrhagic stroke usually have the head of the bed elevated to 30 degrees to reduce intracranial pressure that can occur from the hemorrhage. The head should be in a midline, neutral position to facilitate venous drainage from the brain. Extreme hip and neck flexion should be avoided to prevent an increase in intrathoracic pressure and to promote venous drainage from the brain. For clients with ischemic stroke, the head of the bed usually is kept flat to ensure adequate blood flow and thus oxygenation to the brain. Options 1, 2, and 3 are incorrect positions for clients with hemorrhagic stroke.

A client who suffered a stroke is prepared for discharge from the hospital. The health care provider has prescribed range-of-motion (ROM) exercises for the client's right side. What nursing action should the nurse include in the client's plan of care? 1. Implement ROM exercises to the point of pain for the client. 2. Consider the use of active, passive, or active-assisted exercises in the home. 3. Encourage the client to be dependent on the home care nurse to complete the exercise program. 4. Develop a schedule of ROM exercises every 2 hours while awake even if the client is fatigued.

Rationale: The home care nurse must consider all forms of ROM for the client. Even a client with hemiplegia can participate in some components of rehabilitative care. In addition, the goal in home care nursing is for the client to assume as much self-care and independence as possible. The nurse needs to teach home care measures so that the client becomes self-reliant. Options 1 and 4 are incorrect from a physiological standpoint.

A nurse notes that a client who has suffered a brain injury has an adequate heart rate, blood pressure, fluid balance, and body temperature. The nurse concludes that which area of the client's brain is functioning adequately? 1. Thalamus 2. Hypothalamus 3. Limbic system 4. Reticular activating system

Rationale: The hypothalamus is responsible for autonomic nervous system functions, such as heart rate, blood pressure, temperature, and fluid and electrolyte balance (among others). The thalamus acts as a relay station for sensory and motor information. The limbic system is responsible for emotions. The reticular activating system is responsible for the sleep-wake cycle.

A client who has had a brain attack (stroke) is being managed on the medical nursing unit. At 0800, the client was awake and alert with vital signs of temperature 98° F orally, pulse 80 beats/min, respirations 18 breaths/min, and blood pressure 138/80 mm Hg. At noon, the client is confused and only responsive to tactile stimuli, and vital signs are temperature 99° F orally, pulse 62 beats/min, respirations 20 breaths/min, and blood pressure 166/72 mm Hg. The nurse should take which action? 1. Reorient the client. 2. Retake the vital signs. 3. Call the health care provider (HCP). 4. Administer an antihypertensive PRN.

Rationale: The important nursing action is to call the HCP. The deterioration in neurological status, decreasing pulse, and increasing blood pressure with a widening pulse pressure all indicate that the client is experiencing increased intracranial pressure, which requires immediate treatment to prevent further complications and possible death. The nurse should retake the vital signs and reorient the client to surroundings. If the client's blood pressure falls within parameters for PRN antihypertensive medication, the medication also should be administered. However, options 1, 2, and 4 are secondary nursing actions.

The nurse is reviewing the record for a client seen in the health care clinic and notes that the health care provider has documented a diagnosis of amyotrophic lateral sclerosis (ALS). Which initial clinical manifestation of this disorder should the nurse expect to see documented in the record? 1. Muscle wasting 2. Mild clumsiness 3. Altered mentation 4. Diminished gag reflex

Rationale: The initial symptom of ALS is a mild clumsiness, usually noted in the distal portion of one extremity. The client may complain of tripping and drag one leg when the lower extremities are involved. Mentation and intellectual function usually are normal. Diminished gag reflex and muscle wasting are not initial clinical manifestations.

A client has a neurological deficit involving the limbic system. Which assessment finding is specific to this type of deficit? 1. Is disoriented to person, place, and time 2. Affect is flat, with periods of emotional lability 3. Cannot recall what was eaten for breakfast today 4. Demonstrates inability to add and subtract; does not know who is the president of the United States

Rationale: The limbic system is responsible for feelings (affect) and emotions. Calculation ability and knowledge of current events relate to function of the frontal lobe. The cerebral hemispheres, with specific regional functions, control orientation. Recall of recent events is controlled by the hippocampus.

The home care nurse is making a visit to a client who is wheelchair bound after a spinal cord injury sustained 4 months earlier. Just before leaving the home, the nurse ensures that which intervention has been done to prevent an episode of autonomic dysreflexia (hyperreflexia)? 1. Updating the home safety sheet 2. Leaving the client in an unchilled area of the room 3. Noting a bowel movement on the client progress note 4. Recording the amount of urine obtained with catheterization

Rationale: The most common cause of autonomic dysreflexia is visceral stimuli, such as with blockage of urinary drainage or with constipation. Barring these, other causes include noxious mechanical and thermal stimuli, particularly pressure and overchilling. For this reason, the nurse ensures that the client is positioned with no pinching or pressure on paralyzed body parts and that the client will be sufficiently warm.

A client with a spinal cord injury is prone to experiencing autonomic dysreflexia. The nurse should avoid which measure to minimize the risk of occurrence? 1. Strict adherence to a bowel retraining program 2. Keeping the linen wrinkle-free under the client 3. Preventing unnecessary pressure on the lower limbs 4. Limiting bladder catheterization to once every 12 hours

Rationale: The most frequent cause of autonomic dysreflexia is a distended bladder. Straight catheterization should be done every 4 to 6 hours (catheterization every 12 hours is too infrequent), and Foley catheters should be checked frequently to prevent kinks in the tubing. Constipation and fecal impaction are other causes, so maintaining bowel regularity is important. Other causes include stimulation of the skin from tactile, thermal, or painful stimuli. The nurse administers care to minimize risk in these areas.

A nurse is providing instructions to a client beginning medication therapy with divalproex sodium (Depakote) for treatment of absence seizures. The nurse instructs the client that which represents the most frequent side effect of this medication? 1. Tinnitus 2. Irritability 3. Blue vision 4. Nausea and vomiting

Rationale: The most frequent side effects of medication therapy with divalproex sodium (Depakote) are gastrointestinal (GI) disturbances, such as nausea, vomiting, and indigestion. The items in the other options are incorrect.

A client who had cranial surgery 5 days earlier to remove a brain tumor has a few cognitive deficits and does not seem to be progressing as quickly as the client or family hoped. The nurse plans to implement which approach as most helpful to the client and family at this time? 1. Emphasize progress in a realistic manner. 2. Set high goals to give the client something to "aim for." 3. Tell the family to be extremely optimistic with the client. 4. Inform the client and family of standardized goals of care.

Rationale: The most helpful approach by the nurse is to emphasize progress that is being made in a realistic manner. The nurse does not offer false hope but does provide factual information in a clear and positive manner. The nurse encourages the family to be realistic in their expectations and attitudes. The plan of care should be individualized for each client.

A client who experienced a brain attack (stroke) several months ago still exhibits some difficulty with chewing food. The nurse plans care, knowing that the client has residual dysfunction of which cranial nerve? 1. Vagus (cranial nerve X) 2. Trigeminal (cranial nerve V) 3. Hypoglossal (cranial nerve XII) 4. Spinal accessory (cranial nerve XI)

Rationale: The motor branch of cranial nerve V is responsible for the ability to chew food. The vagus nerve is active in parasympathetic functions of the autonomic nervous system. The hypoglossal nerve aids in swallowing. The spinal accessory nerve is responsible for shoulder movement, among other things.

The nurse caring for a client with a head injury is monitoring for signs of increased intracranial pressure. The nurse reviews the record and notes that the intracranial pressure (cerebrospinal fluid) is averaging 8 mm Hg. The nurse plans care, knowing that these results are indicative of which condition? 1. Normal condition 2. Increased pressure 3. Borderline situation 4. Compensating condition

Rationale: The normal intracranial pressure is 5 to 10 mm Hg. A pressure of 8 mm Hg is within normal range.

The nurse has determined that a client with a neurological disorder also has difficulty breathing. Which activities would be appropriate components of the care plan for this client? Select all that apply. 1. Keep suction equipment at the bedside. 2. Elevate the head of the bed 30 degrees. 3. Keep the client lying in a supine position. 4. Keep the head and neck in good alignment. 5. Administer prescribed respiratory treatments as needed.

Rationale: The nurse maintains a patent airway for the client with difficulty breathing by keeping the head and neck in good alignment and elevating the head of bed 30 degrees unless contraindicated. Suction equipment is kept at the bedside if secretions need to be cleared. The client should be kept in a side-lying position whenever possible to minimize the risk of aspiration.

A client is receiving anticonvulsant therapy with phenytoin (Dilantin). To monitor for adverse effects of this medication, the nurse assesses the results of which laboratory test? 1. Serum sodium 2. Serum potassium 3. Blood urea nitrogen 4. Complete blood count (CBC)

Rationale: The nurse monitors the CBC because hematological effects of this therapy include aplastic anemia, agranulocytosis, leukopenia, and thrombocytopenia. Other test results that warrant monitoring include serum calcium levels, urinalysis, and hepatic and thyroid function tests. Electrolyte results and renal function tests are not a concern with this medication

A client is newly admitted to the hospital with a diagnosis of brain attack (stroke) manifested by complete hemiplegia. Which item in the medical history of the client should the nurse be most concerned? 1. Glaucoma 2. Emphysema 3. Hypertension 4. Diabetes mellitus

Rationale: The nurse should be most concerned about emphysema. The respiratory system is the priority in the acute phase of a brain attack (stroke). The stroke client is vulnerable to respiratory complications such as atelectasis and pneumonia. Because the client has complete hemiplegia (is unable to move) and has emphysema, these risks are very significant. Although options 1, 3, and 4 are important, they are not as significant as option 2.

An older man is brought to the hospital emergency department by a neighbor who heard him talking and found him wandering in the street at 3 am. The nurse should first determine which data about the client? 1. His insurance status 2. Blood toxicology levels 3. Whether he ate his evening meal 4. Whether this is a change in his usual level of orientation

Rationale: The nurse should first determine whether this behavior represents a change in the client's neurological status. The next item to determine is when the client last ate. Blood toxicology levels may or may not be needed, but the health care provider would likely prescribe these. Insurance information must be obtained at some point but is not the priority from a clinical care viewpoint.

A client is admitted with an exacerbation of multiple sclerosis. The nurse is assessing the client for possible precipitating risk factors. Which factor, if reported by the client, should the nurse identify as being unrelated to the exacerbation? 1. Annual influenza vaccination 2. Ingestion of increased fruits and vegetables 3. An established routine of walking 2 miles each evening 4. A recent period of extreme outside ambient temperatures

Rationale: The onset or exacerbation of multiple sclerosis can be preceded by a number of different factors, including physical stress (e.g., vaccination, excessive exercise), emotional stress, fatigue, infection, physical injury, pregnancy, extremes in environmental temperature, and high humidity. No methods of primary prevention are known. Intake of fruits and vegetables is an unrelated item.

The nurse is providing instructions to the client with trigeminal neuralgia regarding measures to take to prevent the episodes of pain. Which should the nurse instruct the client to do? 1. Prevent stressful situations. 2. Avoid activities that may cause fatigue. 3. Avoid contact with people with an infection. 4. Avoid activities that may cause pressure near the face.

Rationale: The pain that accompanies trigeminal neuralgia is triggered by stimulation of the trigeminal nerve. Symptoms can be triggered by pressure such as from washing the face, brushing the teeth, shaving, eating, or drinking. Symptoms also can be triggered by stimulation by a draft or cold air. Options 1, 2, and 3 are not associated with triggering episodes of pain.

A client with Parkinson's disease is at risk for falls because of an abnormal gait. The nurse assesses the client, expecting to observe which type of gait? 1. Unsteady and staggering 2. Shuffling and propulsive 3. Broad-based and waddling 4.Accelerating with walking on the toes

Rationale: The parkinsonian gait is characterized by short, accelerating, shuffling steps. The client leans forward with the head, hips, and knees flexed and has difficulty starting and stopping. An ataxic gait is unsteady and staggering. A dystrophic gait is broad-based and waddling. A festinating gait is accelerating with walking on the toes.

A client with trigeminal neuralgia asks the nurse what causes the painful episodes associated with the condition. The nurse's response is based on an understanding that the symptoms can be triggered by which process? 1. A local reaction to nasal stuffiness 2. A hypoglycemic effect on the cranial nerve 3. Release of catecholamines with infection or stress 4. Stimulation of the affected nerve by pressure and temperature

Rationale: The paroxysms of pain that accompany this neuralgia are triggered by stimulation of the terminal branches of the trigeminal nerve. Symptoms can be triggered by pressure from washing the face, brushing the teeth, shaving, eating, or drinking. Symptoms also can be triggered by thermal stimuli, such as a draft of cold air. Options 1, 2, and 3 are incorrect.

The nurse is reviewing a discharge teaching plan for a postcraniotomy client that was prepared by a nursing student. The nurse would intervene and provide teaching to the student if the student included which home care instruction? 1. Sounds will not be heard clearly unless they are loud. 2. Obtain assistance with ambulation if client is lightheaded. 3. Tub bath or shower is permitted, but the scalp is kept dry until the sutures are removed. 4. Use a check-off system for administering anticonvulsant medications to avoid missing doses.

Rationale: The postcraniotomy client typically is sensitive to loud noises and can find them excessively irritating. Control of environmental noise by others will be helpful for this client. Seizures are a potential complication that may occur for up to 1 year after surgery. For this reason, the client must diligently take anticonvulsant medications. The client and family are encouraged to keep track of the doses administered. The family should learn seizure precautions and should accompany the client during ambulation if dizziness or seizures tend to occur. The suture line is kept dry until sutures are removed to prevent infection.

The nurse is assessing a child with increased intracranial pressure. On assessment, the nurse notes that the child is now exhibiting decerebrate posturing. The nurse should modify the client's plan of care based on which interpretation of the client's change? 1. An insignificant finding 2. An improvement in condition 3. Decreasing intracranial pressure 4. Deteriorating neurological function

Rationale: The progression from decorticate to decerebrate posturing usually indicates deteriorating neurological function and warrants health care provider notification. Options 1, 2, and 3 are inaccurate interpretations.

The nurse has a prescription to administer diazepam (Valium) 5 mg by the intravenous (IV) route to a client. The nurse should administer the medication over a period of at least how long? 1. 15 seconds 2. 30 seconds 3. 1 minute 4. 5 minutes

Rationale: The recommended rate of infusion of diazepam is to give each 5 mg of the medication over at least 1 minute. This will prevent adverse effects including apnea, bradycardia, hypotension, and possibly cardiac arrest.

A client is anxious about an upcoming diagnostic procedure. The client's pupils are dilated, and the respiratory rate, heart rate, and blood pressure are increased from baseline. The nurse plans care, knowing that these changes are the result of which response? 1. Vagal response 2. Peripheral nervous system response 3. Sympathetic nervous system response 4. Parasympathetic nervous system response

Rationale: The sympathetic nervous system is responsible for the so-called fight or flight response, which is characterized by dilated pupils, increases in heart rate and cardiac output, and increases in respiratory rate and blood pressure. The sympathetic nervous system response affects some type of change in most systems of the body. The responses stated in the other options do not produce these effects.

After review of the client's laboratory values, the nurse notes that a phenytoin (Dilantin) level for a client receiving phenytoin (Dilantin) is 7 mcg/mL. The nurse makes which interpretation regarding this laboratory result? 1. The level is within the expected therapeutic range. 2. The level indicates the medication should be stopped. 3. The level is lower than the expected therapeutic range. 4. The level is higher than the expected therapeutic range.

Rationale: The target range for a therapeutic serum level of phenytoin is between 10 and 20 mcg/mL. Levels below 10 mcg/mL are too low to control seizures. At levels above 20 mcg/mL, signs of toxicity begin to appear. This client has a low serum level, and the dosage is likely to be increased.

A client is taking the prescribed dose of phenytoin (Dilantin) to control seizures. Results of a phenytoin blood level study reveal a level of 35 mcg/mL. Which finding would be expected as a result of this laboratory result? 1. Hypotension 2. Tachycardia 3. Slurred speech 4. No abnormal finding

Rationale: The therapeutic phenytoin level is 10 to 20 mcg/mL. At a level higher than 20 mcg/mL, involuntary movements of the eyeballs (nystagmus) occur. At a level higher than 30 mcg/mL, ataxia and slurred speech occur.

The nurse in the health care provider's office is reviewing the results of a client's phenytoin (Dilantin) level determination performed that morning. The nurse identifies that a therapeutic drug level has been achieved if which result is noted? 1. 3 mcg/mL 2. 8 mcg/mL 3. 15 mcg/mL 4. 24 mcg/mL

Rationale: The therapeutic range for serum phenytoin levels is 10 to 20 mcg/mL in clients with normal serum albumin levels and renal function. A level below this range indicates that the client is not receiving sufficient medication and is at risk for seizure activity. In this case, the medication dose should be adjusted upward. A level above this range indicates that the client is entering the toxic range and is at risk for toxic effects of the medication. In this case, the dose should be adjusted downward.

A client who has been taking phenytoin (Dilantin) for seizure control has a serum phenytoin drug level of 8 mcg/mL. The nurse should make which interpretation about this value? 1. A toxic level 2. An inadequate drug level 3. The low end of therapeutic range 4. The high end of therapeutic range

Rationale: The therapeutic serum drug level range for phenytoin is 10 to 20 mcg/mL. A laboratory value of 8 mcg/mL is below the therapeutic range, indicating an inadequate drug level.

A client is taking phenytoin (Dilantin) for seizure control. A blood sample is drawn to determine the serum drug level, and the nurse reviews the results. Which would indicate a therapeutic serum drug range? 1. 5 to 10 mcg/mL 2. 10 to 20 mcg/mL 3. 20 to 30 mcg/mL 4. 30 to 40 mcg/mL

Rationale: The therapeutic serum drug level range for phenytoin is 10 to 20 mcg/mL; therefore the remaining options are incorrect.

The nurse is administering mouth care to an unconscious client. The nurse should perform which actions in the care of this person? Select all that apply. 1. Position the client on his or her side. 2. Use products that contain alcohol. 3. Brush the teeth with a small, soft toothbrush. 4. Cleanse the mucous membranes with soft sponges. 5. Use lemon glycerin swabs when performing mouth care.

Rationale: The unconscious client is positioned on the side during mouth care to prevent aspiration. The teeth are brushed at least twice daily with a small toothbrush. The gums, tongue, roof of the mouth, and oral mucous membranes are cleansed with soft sponges to avoid encrustation and infection. The lips are coated with water-soluble lubricant to prevent drying, cracking, and encrustation. The use of products with alcohol should be avoided because they have a drying effect.

A client who is experiencing an inferior wall myocardial infarction has had a drop in heart rate into the range of 50 to 56 beats/minute. The client also is complaining of nausea. The nurse understands that these symptoms are caused by stimulation of which cranial nerve (CN)? 1.Vagus (CN X) 2.Hypoglossal (CN XII) 3.Spinal accessory (CN XI) 4.Glossopharyngeal (CN IX)

Rationale: The vagus nerve is responsible for sensations in the thoracic and abdominal viscera. It also is responsible for the decrease in heart rate because approximately 75% of all parasympathetic stimulation is carried by the vagus nerve. CN XII is responsible for tongue movement. CN XI is responsible for neck and shoulder movement. CN IX is responsible for taste in the posterior two thirds of the tongue, pharyngeal sensation, and swallowing.

A client who is experiencing an inferior wall myocardial infarction has had a drop in heart rate into the 50 to 56 beats/min range. The client is also complaining of nausea. The nurse interprets that these symptoms are because of stimulation of which cranial nerve (CN)? 1. Vagus (CN X) 2. Hypoglossal (CN XII) 3. Spinal accessory (CN XI) 4. Glossopharyngeal (CN IX)

Rationale: The vagus nerve is responsible for sensations in the thoracic and abdominal viscera. It is also responsible for the decrease in heart rate because approximately 75% of all parasympathetic stimulation is carried by the vagus nerve. CN IX is responsible for taste in the posterior two thirds of the tongue, pharyngeal sensation, and swallowing. CN XI is responsible for neck and shoulder movement. CN XII is responsible for tongue movement.

A client with Parkinson's disease has been prescribed benztropine (Cogentin). The nurse should assess for which gastrointestinal (GI) problems as a side effect of this medication? 1. Diarrhea 2. Dry mouth 3. Increased appetite 4. Hyperactive bowel sounds

Rationale: This medication is classified as an anticholinergic medication and is used to treat Parkinson's disease. Common GI side effects of benztropine therapy include constipation, dry mouth, and nausea. An adverse effect is ileus. These effects are the result of the anticholinergic properties of the medication.

The nurse is teaching a client with paraplegia measures to maintain skin integrity. Which instruction will be most helpful to the client? 1. Shift weight every 2 hours while in a wheelchair. 2. Change bed sheets every other week to maintain cleanliness. 3. Place a pillow on the seat of the wheelchair to provide extra comfort. 4. Use a mirror to inspect for redness and skin breakdown twice a week.

Rationale: To maintain skin integrity, the client should shift weight in the wheelchair every 2 hours and use a pressure relief pad. A pillow is not sufficient to relieve the pressure. While the client is in bed, the bottom sheet should be free of wrinkles and wetness. Sheets should be changed as needed and more frequently than every other week. The client should use a mirror to inspect the skin twice daily (morning and evening) to assess for redness, edema, and breakdown. General additional measures include a nutritious diet and meticulous skin care.

The home health nurse has been discussing interventions to prevent constipation in a client with multiple sclerosis. The nurse determines that the client is using the information most effectively if the client reports which action? 1. Drinking a total of 1000 mL/day 2. Giving herself an enema every morning before breakfast 3. Taking stool softeners daily and a glycerin suppository once a week 4. Initiating a bowel movement every other day, 45 minutes after the largest meal of the day

Rationale: To manage constipation, the client should take in a high-fiber diet, bulk formers, and stool softeners. A fluid intake of 2000 mL/day is recommended. The client should initiate the bowel program on an every-other-day basis. This should be done approximately 45 minutes after the largest meal of the day to take advantage of the gastrocolic reflex. A glycerin suppository, bisacodyl suppository, or digital stimulation may be used to initiate the process. Laxatives and enemas should be avoided whenever possible because they lead to dependence.

The nurse is assessing for Kernig's sign in a child with a suspected diagnosis of meningitis. Which action should the nurse perform for this test? 1. Tap the child's facial nerve and assess for spasm. 2. Compress the child's upper arm and assess for tetany. 3. Bend the child's head toward the knees and hips and assess for pain. 4. Raise the child's leg with the knee flexed and then extend the leg at the knee and assess for pain.

Rationale: To test for Kernig's sign, the client's leg is raised with the knee flexed and then extended at the knee. If any resistance is noted or pain is felt, the result is a positive Kernig's sign. This is a common finding in meningitis. Chvostek's sign, seen in tetany, is a spasm of the facial muscles elicited by tapping the facial nerve in the region of the parotid gland. Trousseau's sign is a sign for tetany in which carpal spasm can be elicited by compressing the upper arm and causing ischemia to the nerves distally. Brudzinski's sign occurs when flexion of the head causes flexion of the hips and knees.

A nurse should question a prescription for which medication in the client concurrently receiving tramadol (Ultram)? 1 β-Blockers 2. Histamine 2 antagonists 3. Calcium channel blockers 4. Monoamine oxidase inhibitors (MAOIs)

Rationale: Tramadol can precipitate a hypertensive crisis if combined with an MAOI. The combination is contraindicated. Its use is not contraindicated with beta blockers, calcium channel blockers, or histamine 2 antagonists.

The nursing student is caring for a child with increased intracranial pressure. On review of the chart, the student nurse notes that a transtentorial herniation has occurred. The nursing instructor asks the student about this type of herniation. Which statement by the student indicates a need for further research about this condition? 1. "The herniation can be unilateral or bilateral." 2. "It involves only anterior portions of the brain." 3. "It can cause death if large amounts of tissue are involved." 4. "The brain herniates downward and around the tentorium cerebelli."

Rationale: Transtentorial herniation occurs when part of the brain herniates downward and around the tentorium cerebelli. It can be unilateral or bilateral and may involve anterior or posterior portions of the brain. If a large amount of tissue is involved, the risk of death is increased because vital brain structures are compressed and become unable to perform their function.

The home care nurse is preparing to visit a client with a diagnosis of trigeminal neuralgia (tic douloureux). When performing the assessment, the nurse should plan to ask the client which question to elicit the most specific information regarding this disorder? 1. "Do you have any visual problems?" 2. "Are you having any problems hearing?" 3. "Do you have any tingling in the face region?" 4. "Is the pain experienced a stabbing type of pain?"

Rationale: Trigeminal neuralgia is characterized by spasms of pain that start suddenly and last for seconds to minutes. The pain often is characterized as stabbing or as similar to an electric shock. It is accompanied by spasms of facial muscles that cause twitching of parts of the face or mouth, or closure of the eye. Options 1, 2, and 3 do not elicit data specifically related to this disorder.

The nurse is performing an assessment on a client suspected of having trigeminal neuralgia (tic douloureux). Which assessment question would elicit data specific to this disorder? 1. "Have you had any facial paralysis?" 2. "Have you noticed that your eyelid has been drooping?" 3. "Have you had any numbness and tingling in your face?" 4. "Have you had any sharp pain or any twitching in any part of your face?"

Rationale: Trigeminal neuralgia is characterized by spasms of pain that start suddenly and last from seconds to minutes. The pain often is described as either stabbing or similar to an electric shock. It is accompanied by spasms of the facial muscles that cause twitching of parts of the face or mouth, or closure of the eye.

A client is taking trihexyphenidyl hydrochloride for the treatment of Parkinson's disease. The nurse should assess for which adverse effect of this medication? 1. Diarrhea 2. Urinary retention 3. Urinary incontinence 4. Excessive perspiration

Rationale: Trihexyphenidyl is an anticholinergic medication. Thus it can cause urinary hesitancy and retention, constipation, dry mouth, and decreased sweating.

The nurse has given medication instructions to a client receiving phenytoin (Dilantin). Which statement indicates that the client has an adequate understanding of the instructions? 1. "Alcohol is not contraindicated while taking this medication." 2. "Good oral hygiene is needed, including brushing and flossing." 3. "The medication dose may be self-adjusted, depending on side effects." 4. "The morning dose of the medication should be taken before a serum drug level is drawn."

Rationale: Typical anticonvulsant medication instructions include taking the prescribed daily dosage to keep the blood level of the drug constant and having a sample drawn for serum drug level determination before taking the morning dose. The client is taught not to stop the medication abruptly, to avoid alcohol, to check with a health care provider before taking over-the-counter medications, to avoid activities in which alertness and coordination are required until medication effects are known, to provide good oral hygiene, and to obtain regular dental care. The client should also wear a Medic-Alert bracelet.

The nurse is assessing a client who is experiencing seizure activity. The nurse understands that it is necessary to determine information about which items as part of routine assessment of seizures? Select all that apply. 1. Postictal status 2. Duration of the seizure 3. Changes in pupil size or eye deviation 4. Seizure progression and type of movements 5. What the client ate in the 2 hours preceding seizure activity

Rationale: Typically seizure assessment includes the time the seizure began, parts of the body affected, type of movements and progression of the seizure, change in pupil size or eye deviation or nystagmus, client condition during the seizure, and postictal status. Option 5 is not a component of seizure assessment.

The nurse is caring for a client with Parkinson's disease who is taking oral benztropine mesylate (Cogentin) daily. What is the priority nursing assessment for the client? 1. Intake and output 2. The prothrombin time 3. The pupillary response 4. The partial thromboplastin time

Rationale: Urinary retention is a side effect of benztropine mesylate. The nurse needs to observe for dysuria, distended abdomen, infrequent voiding of small amounts, and overflow incontinence. Options 2, 3, and 4 are unrelated to the side effects of this medication.

The nurse has just admitted to the nursing unit a client with a basilar skull fracture who is at risk for increased intracranial pressure. Pending specific health care provider prescriptions, the nurse should safely place the client in which positions? Select all that apply. 1. Head midline 2. Neck in neutral position 3. Head of bed elevated 30 to 45 degrees 4. Head turned to the side when flat in bed 5. Neck and jaw flexed forward when opening the mouth

Rationale: Use of proper positions promotes venous drainage from the cranium to keep intracranial pressure from elevating.The head of the client at risk for or with increased intracranial pressure should be positioned so that the head is in a neutral, midline position. The head of the bed should be raised to 30 to 45 degrees. The nurse should avoid flexing or extending the neck or turning the head from side to side.

A client has a prescription to receive valproic acid (Depakene) daily. To maximize the client's safety, when is the best time for the nurse to schedule the administration of this medication? 1. After lunch 2. After breakfast 3. Before bedtime 4. Before breakfast

Rationale: Valproic acid is an anticonvulsant that causes central nervous system (CNS) depression. For this reason, the side effects include sedation, dizziness, ataxia, and confusion. When the client is taking this medication as a single daily dose, administering it at bedtime negates the risk of injury from sedation and enhances client safety. The medication also should be administered at the same time each day.

The nurse is caring for a child after surgical removal of a brain tumor. The nurse should assess the child for which sign that would indicate that brainstem involvement occurred during the surgical procedure? 1. Inability to swallow 2. Elevated temperature 3. Altered hearing ability 4. Orthostatic hypotension

Rationale: Vital signs and neurological status are assessed frequently after surgical removal of a brain tumor. Special attention is given to the child's temperature, which may be elevated because of hypothalamic or brainstem involvement during surgery. A cooling blanket should be in place on the bed or readily available if the child becomes hyperthermic. Inability to swallow and altered hearing ability are related to functional deficits after surgery. Orthostatic hypotension is not a common clinical manifestation after brain surgery. An elevated blood pressure and widened pulse pressure may be associated with increased intracranial pressure, which is a complication after brain surgery, but is not related to brainstem involvement.

A client has sustained damage to Wernicke's area in the temporal lobe from a brain attack (stroke). Which should the nurse anticipate when caring for this client? 1. The client will be unable to recall past events. 2. The client will have difficulty understanding language. 3. The client will demonstrate difficulty articulating words. 4. The client will have difficulty moving one side of the body.

Rationale: Wernicke's area consists of a small group of cells in the temporal lobe the function of which is the understanding of language. The hippocampus is responsible for the storage of memory (the client will be unable to recall past events). Damage to Broca's area is responsible for aphasia (the client will demonstrate difficulty articulating words). The motor cortex in the precentral gyrus controls voluntary motor activity (the client will have difficulty moving one side of the body).

A client has sustained damage to Wernicke's area in the temporal lobe from a stroke (brain attack). The nurse anticipates that the client will have difficulty with which function? 1. Articulating words 2. Understanding language 3. Moving one side of the body 4. Recalling events in the remote past

Rationale: Wernicke's area consists of a small group of cells in the temporal lobe whose function is the understanding of language. Damage to Broca's area is responsible for aphasia. The motor cortex in the precentral gyrus controls voluntary motor activity. The hippocampus is responsible for the storage of memory.

A girl who is playing in the playroom experiences a tonic-clonic seizure. During the seizure, the nurse should take which actions? Select all that apply. 1. Remain calm. 2. Time the seizure. 3. Ease the child to the floor. 4. Loosen restrictive clothing. 5. Keep the child on her back. 6. Place a pillow under the child's head.

Rationale: When a child is having a seizure, the nurse should remain calm, time the seizure, ease the child to the floor if the child is standing or seated, keep the child on the side, loosen restrictive clothing, and place a pillow under the child's head.

The nurse has applied a hypothermia blanket to a client with a fever. The nurse should inspect the skin frequently to detect which condition that is a complication of hypothermia blanket use? 1. Frostbite 2. Skin breakdown 3. Arterial insufficiency 4. Venous insufficiency

Rationale: When a hypothermia blanket is used, the skin is inspected frequently for pressure points, which over time could lead to skin breakdown. The hypothermia blanket decreases the blood flow to pressure areas and can cause numbness, making it so that the client is not aware of damage to the skin. The temperature of the blanket is not cold enough to cause frostbite. Arterial insufficiency and venous insufficiency are not complications of hypothermia blanket use.

A nurse is reviewing the results of a test on a sample drawn from a child who is receiving carbamazepine (Tegretol) for the control of seizures. The results indicate a serum carbamazepine level of 10 mcg/mL. The nurse analyzes the results and anticipates that the health care provider (HCP) will note which prescription? 1. Discontinuation of the medication 2. An increased dose of the medication 3. A decreased dose of the medication 4. Continuation of the presently prescribed dosage

Rationale: When carbamazepine (Tegretol) is administered, blood levels need to be tested periodically to check for the child's absorption of the medication. The amount of the medication prescribed is based on the blood level achieved. Carbamazepine's therapeutic serum range is 3 to 14 mcg/mL. Therefore, the nurse anticipates that the HCP will continue the presently prescribed dosage.

The nurse is told that the result of a serum carbamazepine (Tegretol) level for a child who is receiving the medication for the control of seizures is 10 mcg/mL. Based on this laboratory result, the nurse anticipates that the health care provider (HCP) will document which prescription? 1. Discontinuation of the medication 2. A decrease of the dosage of the medication 3. An increase of the dosage of the medication 4. Continuation of the presently prescribed dosage

Rationale: When carbamazepine is administered, blood levels need to be monitored periodically to check for the child's absorption of the medication. The amount of the medication prescribed is based on the blood level achieved. The therapeutic serum range of carbamazepine is 4 to 12 mcg/mL. The nurse would anticipate that the HCP will continue the presently prescribed dosage.

The nurse is caring for a client with severe back pain. Codeine sulfate has been prescribed for the client. Specific to this medication, which intervention should the nurse include in the plan of care while the client is taking this medication? 1. Monitor radial pulse. 2. Monitor bowel activity. 3. Monitor apical heart rate. 4. Monitor peripheral pulses.

Rationale: While the client is taking codeine sulfate, the nurse would monitor vital signs and assess for hypotension. The nurse also should increase fluid intake, palpate the bladder for urinary retention, auscultate bowel sounds, and monitor the pattern of daily bowel activity and stool consistency because the medication causes constipation. The nurse should monitor respiratory status and initiate deep-breathing and coughing exercises. In addition, the nurse monitors the effectiveness of the pain medication.

The nurse is caring for a client who is on bed rest as part of aneurysm precautions. The nurse should avoid doing which action when giving respiratory care to this client? 1. Encourage hourly coughing. 2. Assist with incentive spirometer. 3. Encourage hourly deep breathing. 4. Reposition gently side to side every 2 hours.

Rationale: With aneurysm precautions, any activity that could raise the client's intracranial pressure (ICP) is avoided. For this reason, activities such as straining, coughing, blowing the nose, and even sneezing are avoided whenever possible. The other interventions (repositioning, deep breathing, and incentive spirometry) do not provide added risk of increasing ICP and are beneficial in reducing the respiratory complications of bed rest.

A client receiving therapy with carbidopa/levodopa (Sinemet) is upset and tells the home health nurse that his urine has turned a darker color since he started taking this medication. The client wants to discontinue its use. In formulating a response to his concerns, the nurse interprets that this change is indicative of which condition? 1. Developing toxicity 2. A harmless side effect of the medication 3. A result of taking the medication with milk 4. A sign of interaction with another medication

Rationale: With carbidopa/levodopa therapy, a darkening of the urine or sweat may occur. The client should be reassured that this is a harmless effect of the medication, and its use should be continued. Darkened urine is not indicative of carbidopa/levodopa toxicity, the result of taking the medication with milk, or a sign of interaction with another medication.

A client with insomnia has been started on zolpidem (Ambien). After instructing the client in how to obtain the maximal effect of zolpidem, the nurse determines that the client demonstrates understanding of correct administration of the medication by which statement? 1. "I take the medication just after my evening meal." 2. "I need to take the medication with a full glass of water." 3. "I wait until bedtime and take the medication with a snack." 4. "I take the medication with milk or an antacid to avoid stomach upset."

Rationale: Zolpidem is a hypnotic. The client should be instructed to take the medication at bedtime and to swallow the medication whole with a full glass of water. For faster onset of sleep, the client should be instructed not to take the medication with food or immediately after a meal. Antacids affect the absorption of the medication.


Kaugnay na mga set ng pag-aaral

Completing the Application, Underwriting, and Delivering Policy

View Set

ACCT 3306: Chapter 5: Cost-Volume-Profit Relationships

View Set

Nutrition in Health: Chapter 11: Water and Major Minerals.

View Set

Health Promotions and Psychosocial

View Set